0% found this document useful (0 votes)
518 views

Engineering Bridging Program Kit

Copyright
© © All Rights Reserved
Available Formats
Download as PDF, TXT or read online on Scribd
0% found this document useful (0 votes)
518 views

Engineering Bridging Program Kit

Copyright
© © All Rights Reserved
Available Formats
Download as PDF, TXT or read online on Scribd
You are on page 1/ 222

ENGINEERING

BRIDGING PROGRAM
ALGEBRA
TRIGONOMETRY
ANALYTIC GEOMETRY
PLANE AND SOLID GEOMETRY
PART 1:
ALGEBRA
CIVIL ENGINEERING BOARD EXAM PROBLEMS PHILIPPINES

ALGEBRA MODULE 1 August 24, 2020

INTRODUCTION

Algebra is a branch of mathematics dealing with symbols and the rules for manipulating those symbols. In elementary algebra, those symbols (today
written as Latin and Greek letters) represent quantities without fixed values, known as variables.

The word algebra comes from the Arabic ‫( ﺍﻟﺟﺑﺭ‬al-jabr lit. "the restoring of broken parts") from the title of the early 9th century book cIlm al-jabr wa l-muqābala
"The Science of Restoring and Balancing" by the Persian mathematician and astronomer al-Khwarizmi. In his work, the term al-jabr referred to the operation of
moving a term from one side of an equation to the other, ‫ ﺍﻟﻣﻘﺎﺑﻠﺔ‬al-muqābala "balancing" referred to adding equal terms to both sides. Shortened to just algeber
or algebra in Latin, the word eventually entered the English language during the fifteenth century, from either Spanish, Italian, or Medieval Latin.

FIELDS OF ALGEBRA

1. Elementary algebra is the most basic form of algebra. It is taught to students who are presumed to have no knowledge of mathematics beyond the
basic principles of arithmetic. In arithmetic, only numbers and their arithmetical operations (such as +, −, ×, ÷) occur. In algebra, numbers are often
represented by symbols called variables (such as a, n, x, y or z).
2. Abstract algebra extends the familiar concepts found in elementary algebra and arithmetic of numbers to more general concepts.
3. Linear algebra, in which the specific properties of linear equations, vector spaces and matrices are studied.
4. Boolean algebra, a branch of algebra abstracting the computation with the truth values false and true.

REAL NUMBER SYSTEM

The real numbers is the set of numbers containing all of the rational numbers and all of the irrational numbers. The real numbers are “all the numbers” on the
number line. There are infinitely many real numbers just as there are infinitely many numbers in each of the other sets of numbers.

Figure 1 : The complex number system © CK12

Natural numbers – numbers that is used for counting that starts from 1,2,3,4 and so on.
Whole numbers – numbers that includes zero.
Integers – numbers that now includes negative numbers.
Rational numbers – consists of all the numbers that can be expressed as a quotient of two integers .
Parts: Numerator – the dividend of a number
Denonminator – the divisor of a number

Rational numbers as decimals


TERMINATING DECIMALS – decimals have exact value. Ans. 1/4 = 0.25
REPEATING DECIMALS – decimals have repeating its digits. Ans. 1/3 = 0.333333

Irrational numbers - set of all numbers whose decimal representations are neither terminating nor repeating. Irrational numbers cannot be expressed as a quotient
of integers. Examples: √3 , e, π

Imaginary numbers- these numbers having the negative square root or even roots.

Complex numbers – these having in the form of a + bi


Absolute value - is the distance from 0 to a on a number line. Because absolute value describes a distance, it is never negative. Ex. |-1| = 1

PROPERTIES OF REAL NUMBERS

Commutative property - adding or multiplying numbers, order is neglected.

a + b = b + a ; ab = ba

Associative property – adding or multiplying three numbers, no effect on the resulting sum of product in any of two numbers to be operated.

(a + b) + c = a + (b + c) ; ab(c) = a(bc)

Distributive property – multiplying a number by a sum of two numbers, gets the same result. As getting multiplied by the number each of the terms.

a (b + c) = ab + ac ; (b+c)a = ab + bc

THE PEMDAS

P – Parentheses , Brackets and the like


E – Exponents and Roots
M – Multiplication
D – Division
A - Addition
S – Subtraction

Examples:

1. 1 + 2 × 3

1+6=7 .
SOLUTION:

2. 10 − 3 × 6 + 10 + 6 + 1 × 4
SOLUTION:

3. 5 + 6 2 3 + 9
SOLUTION:

4. 6 − 12 ÷ −13 + 14 ×2
EXERCISES: Evaluate the following numbers:

−5√30 − 5 + −2 −21 −7 − −9
1. . 11. . 2.5
4 + |7 − 10| 19 6 −3 − 1 −2

5 + −2 2
2. . ∞ 12. √36 × 3 − √16 ÷ 2 . 53
6 ⋅ √9 − 9 ⋅ 2

3. 1 + 3|−4 − −8 | + 2|3 + −5 | . 81 13. 2 − 4|−4 − 3| + −2 !


. −10

5 + 12 6 −6
4. + . − 16 14. 2 ⋅ 5 + 7 ⋅ 4 − 20 . 26
−13 −2 5

5 −4
5. 32 − 3 ÷ 9 × 10 " −4 ÷8+3 . 39 15. − √11 − 2 . 0
7

6. 40 − 4 + 6 ÷ 2 + 3 . 38 16. 7 ⋅ 6 − 4 + 1" . 27

4 7+5
7. 8−3 +2⋅5 −7+4⋅6 . 52 17. . 16
2+1

16 − 4 |−2| + 1
8. 6+ −2 . 6 18. . −1
2 +2 −7 − √4

3 +3
9. # $ +1 . 26 19. 18 + 2 ∙ 3 − 10 . 14
−2 −3

10 . 2 ⋅ 3 − 5 + 8 ⋅ 2 − 1 . 11 20. 4 15 − 9 + 8 2 . 88

EXPONENTS AND RADICALS

EXPONENTS - a quantity representing the power to which a given number or expression is to be raised, usually expressed as a raised symbol beside the number
or expression.

RADICALS - is the mathematical opposite of an exponent, in the same sense that addition is the opposite of subtraction. The smallest radical is the square root,
represented with the symbol .

LAWS OF EXPONENTS
LAWS OF RADICALS

CONJUGATE – set of two numbers that has the same quantity but the opposite sign in the middle. (More important when studying limits of functions)
EXAMPLES

1. &' (
)

SOLUTION:

2. ' " * '* +


SOLUTION:

−2 -!
3,
−2 .
SOLUTION:

= -32

3/ 0" 1 2
4.
4/ 0 1 0

SOLUTION:

4'"
5. 3 (
*
SOLUTION
6. &32/-2 1 ( 4 +
5

SOLUTION:

7. √10 − 2√10 + 3√10


6 3

SOLUTION:

→ Simplified

√5
8.
√2 − √7
SOLUTION:

9. &'* &' *
) 6

SOLUTION:

EXERCISES - Simplify the following:

√24' * " 3* )√'


)

1. & 7 ∙ 6&7 . &7+ 11. .


89

√16' *
)
2
3

/ 0 1! 1- 7 35 − 7√3
2. : ; . 12. .
5 125/ . 5 + √3 22

0+
40 / 1 ! / 2ℎ ? 0 @ ! 2ℎ @
3. < = . 13. .
- " 3?@ 3? !
2 / 0 1! 81

4 5 GH- − 10 5 G0- −2 −2 + 1
4. ABA CD'EF .9 14. .
2 5 G 3× 2 ( 12

5√3/
5. &75/ 1 0! . 15. I&/ ( 1 J . /1 √/
1
6. &/ − 6/ + 9 . |/ − 3| 16. √6 √15 √10 . 30

- - −5/1 0 0 1-2
7. 277 K( E 0- . 37K E 0 17. .
5/ ! 1 0! 125/ (

/1 !
8. −2 ⋅5 . −200 18. 2 /1 0 0
4/ 0 1 0-
.
2

/ + &1 L/ + &1M
9. 2√40 + √135 . 7√5 19. .
/ − &1 / −1
3 3 3

" 64/-
10. √' ⋅ &' . '! 20. 4/ ! 1 0! .
1-
4

CONVERSION OF UNITS, SCIENTIFIC NOTATION AND NUMERICAL PREFIXES

CONVERSION FACTOR - a number used to change one set of units to another, by multiplying or dividing. When a conversion is necessary, the appropriate
conversion factor to an equal value must be used.

Note: MKS and CGS system will be discussed further in Engineering Mechanics.

INTERNATIONAL SYSTEM OF UNITS (SI UNITS) or METRIC SYSTEM - a system of physical units ( SI units ) based on the meter, kilogram, second, ampere,
kelvin, candela, and mole, together with a set of prefixes to indicate multiplication or division by a power of ten.

SI SYSTEM and ENGLISH SYSTEM ENGINEERNG BASE UNITS

IMPERIAL OR ENGLISH UNITS or US CUSTOMARY UNITS - are the units of measurement used in England up to 1826 (when they were replaced by Imperial
units), which evolved as a combination of the Anglo-Saxon and Roman systems of units. Various standards have applied to English units at different times, in
different places, and for different applications.

CONVERSION FACTORS
I. LINEAR , DEPTHS AND DISTANCES
1 fathom = 6 feet = 1.8288 m
1 nautical mile = 6076.1155 feet = 1.852 meters
METRIC CONVERSION
1 meter = 1000 mm = 100 cm

II. TIME
1 hour = 60 minutes
1 minute = 60 seconds
1 hour = 60 minutes = 3600 seconds (60 × 60)
1 day = 24 hours = 1440 minutes = 86400 seconds
1 week = 7 days
1 month = 30 or 31 days
1 year = 365 days = 52 weeks = 12 months
1 decade = 10 years
1 score = 20 years
1 century = 100 years (21st century starts at year 2000)
1 millennium = 1,000 years (Note: third millennium starts at year 2000)

III. AREA
1 square meter = 1,000,000 square millimeter
1 hectare = 10,000 m^2
1 square kilometer = 100 hectares = 1,000,000 m^2
1 square inch = 645.16 mm^2
1 square feet = 144 in^2 = 92903.04 mm^2
1 square yard = 1296 in^2 = 836,127.36 mm^2
1 acre = 0.40469 ha = 4046.856 m^2 = 43560 ft^2
1 square mile = 2.59 km^3

IV. MASS
1 gram = 1000 milligrams
1 dram = 1.77185 grams
1 ounce = 28.3 grams = 16 drams
1 pound = 454 grams = 16 ounces
1 kilogram = 1000 grams = 2.20462 pounds
1 hundredweight = 100 pounds = 50.8023 kg
1 slug = 14.5939 kg = 32.174 lbs
1 Metric ton = 1000 kg = 2204.62 lbs
1 short ton = 2000 lbs = 907.185
1 long or Imperial ton = 2240 lbs = 1016.05

V. WEIGHT AND FORCE


1 Newton = 100,000 dyne = 0.2248 lb
1 kilogram = 9.80665 N (widely used is 9.81)
1 slug = 143.1173 N or 0.143117 kN
1 pound = 4.448 N

VI. VOLUME
1liter = 1000 mm = 1000 cc
1 cubic inch = 16.387064 mL
1 cubic feet = 28.31685 L = 1728 cu in.
1 fluid ounce = 29.5737 mL = 1.80469 cu in.
1 pint = 16 fl oz. = 473.176 mL
1 gallon = 3.78541 L = 8 pints = 128 fl oz
1 quart = 32 fl oz = 946.353 mL
1 barrel = 158.9873 liters = 42 gallons

VII. SPEED
1 feet per second = 0.3048 meters per second
1 meter per second = 3.6 kilometers per hour
1 mile per hour = 1.46667 feet per seconds
1 meter per second = 2.23694 miles per hour
1 mile per hour = 1.60934 kilometers per hour
1 knot = 1.852 kilometers per hour = 1.15078 miles per hour
VIII. ACCELERATION
1 foot per second squared = 0.3048 meter per second squared
1 gravity = 9.81 m/s^2 = 32.2 ft/s^2

IX. WORK ,MOMENT, ENERGY AND HEAT


1 foot pound = 1.35582 joule
1 calorie = 4184 joules = 4.1868 watts = 3.08596 ft lb
1 watt = 3,600 Joules
1 horsepower = 746 watts = 550 ft lb = 2545 BTU
1 British thermal unit = 252.164 calories = 1055.06 Joules

X. PRESSURES AND STRESSES


1 Pound per square inch = 6.8948 kilo Pascals
1 atmosphere = 101.3 kPa = 760 mm Hg = 14.6959 Psi
1 MPa = 1,000 kPa
1 kPa = 20.88543 pound per square foot
1 kilogram per square centimeter = 98.0665 kPa = 14.2233 pounds per square inch
1 MPa = 145.038 pounds per square inch

XI. STRUCTURAL CONVERSIONS AND UNIT WEIGHTS (DENSITIES)


1 kilonewton per cubic meter = 6.36588 pounds per cubic feet
1 kilonewton per cubic meter = 101.97177 kg/m^3
1 gram per cubic cm = 1000 kg/m^3
1 Newton meter = 8.85075 pound inch
1 pound foot = 1.35582 N.m
1 kilonewton meter = 737.562
1 kilopound = 4.448 kN
1 pound per inch = 175.126835 N/m
1 kilonewton per meter = 68.5218 pound per foot

XII. TEMPERATURE
1 Fahrenheit = 1.8C +32
1 Degree = (F - 32)/1.8
1 Kelvin = 273.15 + C (For Fahrenheit , convert to Celsius)
1 Rankine = F + 459.67 = 0.555556 Kelvin
1 Reaumur = 0.8C (For F Scales, convert to Celsius)

XIII. ANGULAR UNITS


1 radian = 57.296 degrees
1 revolution = 360° = 2π radians = 400 gradians = 6400 mils
1 gradian = 0.9 degrees = 16 mils

XIV. MISCELLANEOUS
1 dozen = 12 units
1 baker’s dozen = 13 units
1 gross = 144 units
1 small gross = 10 dozens = 120 units
1 great gross = 1728 units
1 teaspoon = 4.92892 ml
1 tablespoon = 3 teaspoons
1 cup = 236.588 mL
1 byte = 8 bits
1 MbPS = 1000 KBPs
1 carat = 0.2 gram

PHILIPPINE SYSTEM OF MEASUREMENTS

A. MASS
1 punto = 1/3 catty
1 chinanta = 12 catty
1 lachsa = 48 catty
1 caban = 97 catty
1 pecul = 100 catty
1 jeydon = 1,000 catty
1 kaban = 60.33 (nowadays 50 kg)
B. AREA
1 loan = 0.1 balita
1 quignon = 10 balita
1 stub = 100 balita
1 balita = 2795 m^2

C. VOLUME
1 kaban = 25 ganta
1 ganta = 8 chupa
1 chupa = 3.75 mL
1 apatan = 1⁄4 chupa

PREFIXES OF SI UNITS

SCIENTIFIC NOTATION

A positive number is written in scientific notation if it is written as the product of a number a , where 1 ≤ a ≤ 10 and an integral units r of 10:

a × 10^r

Steps:
1. Move the decimal point in the original number until the new number has a value between 1 and 10.
2. Count the number of decimal places the decimal point was moved in Step 1. If the original number is 10 or greater, the count is positive. If the original
number is less than 1, the count is negative.
3. Write the product of the new number in Step 1 by 10 raised to an exponent equal to the count found in Step 2.
EXAMPLES – CONVERSION OF UNITS

1. Convert -25° C to Kelvin.


SOLUTION:

K = 273.15 + °C = 273.15 – 25 = 248.15 K. Ans.

2. There are only 14 peaks whose summits are over 8000 m above sea level. They are the tallest peaks in the world and are referred to as “eight-
thousanders.” What is the elevation, in feet, of an elevation of 8000 m?
SOLUTION:

3. Convert 0.02515 ft^3 to cm^3.


SOLUTION:

4. How many kilonewton per cubic meters are equivalent to 95.05 pounds per cubic feet?
SOLUTION:

5. Convert 0.357 atm to torr (1 atm = 760 torr)


SOLUTION:
6. Convert 20 dram to milliliters.
SOLUTION

7. On an interstate highway in a rural region of Wyoming, a car is traveling at a speed of 38.0 m/s. Is this car exceeding the speed limit of 75.0 mi/h?
SOLUTION

SCIENTIFIC NOTATION – Convert the following numbers into scientific notation.

1. 444
SOLUTION:
Move the decimal points two places leftwards. Therefore, 444 = 4.44 x 10^2 Ans.

2. 0.0068
SOLUTION:
Move the decimal points three place rightwards . The position of the non zero digit is the negative power of 10.
0.0068 = 6.8 x 10^-3 Ans.

2,400,000,000 × 0.0000045
3. Evaluate:

0.00003 × 1500

SOLUTION

4. The amount of water surface area on the Earth is 140 million square miles. Convert in scientific notation.
SOLUTION:

140,000,000 move eight decimal places to the left. → 1.4 x 10^8 square miles. Ans.

EXERCISES A
Convert the following system of units :

1. 84 inches to feet Ans. 7 feet


2. 1 hectoliter to cubic meter Ans. 100 m^3
3. 7 kilograms to grams Ans. 7,000 g
4. 23 square nano meters to square decimeter Ans. 2.3 (10^-15) sq decimeter
5. 24 carats to grams Ans. 0.48 grams
6. 1.65 gigaliters to megaliters Ans. 1650 megaliters
7. 450 GB to kb. 4.5 x 10^8 kilobytes
8. 2.5 cavans to kilograms Ans. 125 kgs
9. 4 cups and 3 tablespoons to mL. Ans. 990.712228
10. Convert 0.9 pg to fg. Ans. 9 x 10^-10 fg
11. Convert 7 days 6 hours into hours. Ans. 174 hours
12. Ron covers a distance of 1710 m in 3 minutes. Find his speed in km/hr. Ans. 34.2 km/hr
13. The distance from the university to home is 10 mi and it usually takes 20 min to drive this distance. Calculate the average speed in meters per second
(m/s). Ans. 13m/s
14. The volume of a tetrachloride having a mass of 1200 kg is 0.952 m^3. Specific weight. Ans. 12.36 kN/m^3
15. 15 Reaumur to Fahrenheit . Ans. 65.75° F

EXERCISES B – Write the following into scientific notation.

1. 0.00000032 Ans. 3.2 x 10^-7


2. At their closest points, Mars and Earth are approximately 75,000,000 kilometers apart . Express the distance in meters. Ans. 7.5 x 10^10 m
3. According to state census information, the population of LaVergne, Tennessee in 2012 was 3.3777 * 104. The population of Murfreesboro, Tennessee
in 2012 was 1.14038 * 105. What was the combined population of both cities? Ans. 1.47815 x 10^5
4. Simplify: (3.4 x 10^6) ÷ (2 x 10^-10 ) Ans. 1.7 × 10^16
5. As of December 2011, the national debt was $15.2 trillion, or 15.2 * 1012 dollars.
6. At that time, the U.S. population was approximately 312,000,000 (312 million), or 3.12 * 10^8 If the national debt was evenly divided among every
individual in the United States, how much would each citizen have to pay? Ans. 4.87 x 10^4
7. 5 megavolts to volts . Ans. 5 x 10^6 volts
8. The Memphis Grizzlies play in the FedEx Forum. The FedEx Forum can hold approximately x 1.5 × 10^4 people. If the Grizzlies play 41 games in the
FedEx Forum and all of the seats were full for all of the games, how many people were there total? Ans. 6.15× 10^5 people
9. Simplify: (7.245 x 10^9) ÷ (2.1× 10^-3) Ans. 3.45 x 10^12
10. A computer was used to draw a rectangle with an area of 0.000007 square meter. Find the area in square centimeters. Ans. 7 x 10^-2 cm^2

NEXT TOPICS ON AUGUST 26 , 2020:

1. Operations on Algebraic Expressions


2. Special Products and Factoring
3. Rational Expressions
References:

https://www.livescience.com/50258-algebra.html#:~:text=Algebra%20is%20a%20branch%20of,fixed%20values%2C%20known%20as%20variables.
https://en.wikipedia.org/wiki/Algebra#Different_meanings_of_%22algebra%22
https://www.varsitytutors.com/hotmath/hotmath_help/topics/number-
systems#:~:text=The%20real%20numbers%20is%20the,the%20other%20sets%20of%20numbers.
Introductory Algebra by Blitzer
Algebra and Trigonometry by Stewart
Elementary Algebra by Kaufmann
Pre Algebra for Dummies
http://www.wallace.ccfaculty.org/book/0.3%20Order%20of%20Operations.pdf
https://www.rit.edu/studentaffairs/asc/sites/rit.edu.studentaffairs.asc/files/docs/services/resources/handouts/A3_OrderofOperations_BP_9_22_14.pdf
https://www.chilimath.com/lessons/introductory-algebra/order-of-operations/
https://www.purplemath.com/modules/orderops3.htm
College Algebra by Sullivan
http://www.jaconline.com.au/mathsquestvic/mq-vic-first-ed/number/whole/skillbuilders/MS-order-ops-squares.pdf
https://saylordotorg.github.io/text_elementary-algebra/s04-07-order-of-operations.html
Algebra for Dummies
Intermediate Algebra by Martin-Gay
Introductory and Intermediate Algebra by Blitzer
http://www.columbia.edu/itc/sipa/math/exponents.html
http://www.mathwords.com/r/radical_rules.htm
https://www.purplemath.com/modules/exponent.htm
Algebra and Trigonometry by OpenStax
https://www.mesacc.edu/~scotz47781/mat120/notes/exponents/review/review.html
https://www.lonestar.edu/departments/learningcenter/exponents_and_radicals.pdf
https://www.intmath.com/exponents-radicals/exponent-radical.php
Glencoe’s Advanced Mathematical Concepts
https://mcckc.edu/tutoring/docs/bt/exp_rad_log/Radical_Workshop.pdf
Precalculus by Blitzer
https://cdn.kutasoftware.com/Worksheets/Alg1/Properties%20of%20Exponents.pdf
https://math.libretexts.org/Courses/Mission_College/Math_C_Intermediate_Algebra_(Carr)_Spring_2020/03%3A_Radical_Functions_and_Equations/3.02%3A_
Adding%2F%2FSubtracting%2F%2FMultiplying_Radicals
https://en.wikipedia.org/wiki/English_units
https://en.wikipedia.org/wiki/SI_base_unit
https://en.wikipedia.org/wiki/Philippine_units_of_measurement
https://cds.cern.ch/record/1501100/files/978-3-642-23026-4_BookBackMatter.pdf
https://metricunitconversion.globefeed.com/area_conversion_table.asp
https://en.wikipedia.org/wiki/Mass
https://metricunitconversion.globefeed.com/volume_conversion_table.asp
https://www.smartconversion.com/unit_conversion/acceleration_conversion_table.aspx
https://en.wikipedia.org/wiki/Gross_(unit)
https://en.wikipedia.org/wiki/R%C3%A9aumur_scale#:~:text=The%20R%C3%A9aumur%20scale%20(French%20pronunciation,0%20and%2080%20degrees%
20respectively.
https://mathworld.wolfram.com/SIPrefixes.html
Physics Principles with Applications by Giancoli
https://www.chemteam.info/Metric/Metric-English-Conv.html
https://www.centrallyon.org/pages/uploaded_files/Chapter%2010%20Worked%20Examples.pdf
Physics For Scientists and Engineers by Serway
Precalculus by Ron Larson
https://examples.yourdictionary.com/scientific-notation-examples.html
Schaum’s Outlines of Basic Mathematics
https://phys.libretexts.org/Bookshelves/University_Physics/Book%3A_University_Physics_(OpenStax)/Map%3A_University_Physics_I_-
_Mechanics%2C_Sound%2C_Oscillations%2C_and_Waves_(OpenStax)/01%3A_Units_and_Measurement/1.04%3A_Unit_Conversion
Fluid Mechanics and Hydraulics by Besavilla
https://cpb-us-e1.wpmucdn.com/blogs.ccsd.edu/dist/6/634/files/2015/11/7A-scientific-notation-multiply-and-divide-word-problems-1o2dn88.pdf
https://ideagalaxyteacher.com/teaching-operations-scientific-notation/
https://cpb-us-east-1-juc1ugur1qwqqqo4.stackpathdns.com/blogs.ccsd.edu/dist/6/282/files/2012/10/Day-14-Scientific-Notation-Applications-2c2nw7g.pdf
CIVIL ENGINEERING BOARD EXAM PROBLMEMS PHILIPPINES - AUGUST 28 , 2020

ALGEBRA 2

OPERATIONS OF POLYNOMIALS

POLYNOMIALS - a single term or the sum of the two or more terms containing variables with whole number exponents.

Considering this polynomial

7 9 13 6

7x^3 - is considered as the highest term . The degree (highest exponent) is 3.

MONOMIAL – single terms


BINOMIAL – two terms
TRINOMIAL – three terms
MULTINOMIAL OR POLYNOMIAL – four or more terms.

OPERATIONS ON POLYMOIALS

1. Add and subtract by combining like or similar terms. Follow operations on integers.
2. Multiply by the polynomials by following laws of exponents. Combine like terms when necessary.
3. Divide polynomials using by either long division or synthetic division.

EXAMPLES:

1. 7 3 4 4 6 7
SOLUTION:

2. 4 5 2 3 6
SOLUTION:

3. 2 4 2 4
SOLUTION:

4. 2 3 1

5. 3 5 4 1
SOLUTION:
6. 2 11 34 2
SOLUTION:

7. 4 2 3 2

8.
SOLUTION:

9. 2 7 4 4
SOLUTION:

EXERCISES : Evaluate the following:

1. 2 3 1 3 5 4 . 2 3

6 4 10
2. .3 2 5
2
3. 1 5 4 3 2 2 . 5 9 7 . 10 12 13

4. 6 4 2 4 .2 8 32 16

5. 3 2 5 4 3 . 6 8 3

6. 7 8 1 .7 8

7. 5 7 4 2 . 35 20 10

8. 7 10 2 . 5

9. 6 11 31 15 3 2 .2 5 7 .1

10. 3 4 5 2 . 2 . 11 9

11. 1 1 1 1 .2

12. 4 10 3 5 . 2

13. 5 6 3 10 6 2 4 4 . 8 14

14. 2 6 3 3 5 6 5 . 5 6 4 5

15. 6 8 4 8 8 . 48 16 32 32

SPECIAL PRODUCTS AND FACTORING

1. Common Monomial Factors → =


2. Difference of Two Squares → " = " "
3. Sum and Difference of Two Cubes → " = ±" ∓ " "
4. Perfect Square Trinomial → ±" = ±2 " "
5. Square of a Trinomial → ±"±% = " % ± 2 " ± 2"% ± 2 %
For the formula of the number of terms when squaring polynomials:
& & 1
=
2

n - no of terms of a squared polynomial


t – no of terms in an entire polynomial to be squared

6. Factorable Trinomials → ' ( '( = ' (


7. Factoring by Grouping – Can be factored using synthetic division - will be discussed on Polynomial Functions
8. Suitable Terms – known as Sophie – Germain Factoring method. The exponents should be multiple of 4. → "
9. Odd Primes – both should be an odd exponent.
)
± ") = ±" )*+
± )*
" )*
" ±⋯ " )*+

EXAMPLES :
1. 20 " 8 12 "
SOLUTION:
GCF (20a^3 ,8a , -12a^5) = 4a
Then isolate,

4 5 " 2 3 "

2. 7 6
SOLUTION:
3. 2 1
SOLUTION:

4. 216 - 8
SOLUTION:
8 27 - = 8 3 = 8 3 9 3

5. 9 10 1
SOLUTION:

6. 2 . / .+-
SOLUTION:
2 ./ = 2 ./ 4 ./

Rearranging:
2 . / .+- 4 ./ = ./ 2 .
./ 2 . ./ 2 . = 2 . ./ 2 . ./ = . . .

7.
SOLUTION:

EXERCISES – Factor the Following expressions :


1. 3 2 6 . 3 2 11. 16 40 25 . 5 4
2. 16 8 1 . 4 1 12. 64 . 4 8 4 8
3. 120 360 108 . 12 0 30 9 13. 5 10 5 . 5 1
4. 6 25 . 5 2 5 2 14. 64 . 8 8
5. 216 125 . . 6 5 . 36 30 . 25 . 15. 12 22 8 = 2 2 1 3 4
6. 4 4 1 . 2 1 16. .
7. 6 . 12 . 8. . 2. 17. ) 8 ) 16 . ) 4
8. 270 125 . 30 5 90 150 25 18. 4 4 . 2 2
9. 2 3 . 3 19. 5 2 10 . 5 2
) 1
10. 243 . 3 3 9 27 81 20. . ) 1 ) ) 1 1

RATIONAL ALGEBRAIC EXPRESSIONS

RATIONAL ALGEBRAIC EXPRESSIONS - the ratio or quotient of two polynomials .

Examples:

2 1
; ≠ 4.
4

GREATEST COMMON FACTOR - for the monomial ax^k ,

a - biggest integer factor common to all of polynomial coefficients


k – smallest exponent found in all of the terms for a polynomial.

LEAST COMMON MULTIPLE OR LEAST COMMON DENONMINATOR – smallest real number that all of the denonminators or multiples divide evenly.

OPERATIONS of RATIONAL ALGEBRAIC EXPRESSIONS


1. Add or subtract by combining like terms and same denonminator (use LCD for distinct ones)
2. Multiply or divide (reciprocate) the terms.
3. Answers should be shown to simplified terms by factoring.
COMPLEX FRACTIONS – fractions have rational expressions in numerator or denonminator or both.

Examples:

EXAMPLES – Simplify the following :

5 7 2
1.
2 6 6

SOLUTION:

3 2 4 8
2. ∙
2 1 3 2
SOLUTION:

4 12 9
3.
16 6

4 6 3 5
4.
3 2 1 2
SOLUTION:
"
5.
% 5 "% "5 % 5 "% "5
SOLUTION:

4 10 6 15
6.
3 9

SOLUTION:

EXERCISES – Simplify the following rational expressions

7 10 4 21 5 5 3 35 6
1. ∙ . 9. .
5 14 9 18 6 4 14 28

27 2 4 3 9 6 11 35 4 49 4 49
2. ⋅ . 10. ⋅ .
4 6 2 8 22 21 9 25 4 3 3 5

2 2 2 2 8 50 12
3. . 11. .
4 16 4 4 3 2 3 4

2 6 4 14 10 9 5
4. . 12. ⋅ .
2 1 2 1 2 1 3 9 4 12 6

1 3 4 2 1 23 1
5. . 13. .
121 8 33 121 3
3 2 6 4 1 5 2
6. . 14. 1 .
1 2 1 2 2 2

30 6 5 2 3 3 12 3
7. . 15. ⋅ .
4 12 2 6 8 16 1 2 1

2 1 3 2
8. .
4 2 2 4

COMPLEX FRACTIONS

COMPLEX/COMPOUND FRACTIONS – fractions that have rational expressions in numerator, denominator or both.

Methods:
1. Traditional way of combining terms into single fraction
2. LCD Method- getting LCD of all denominators.

EXAMPLES:

2
1. 32
1
16 2

SOLUTION:

2% % 1
2. % 2 % 1
2% 1
% 1

SOLUTION:
EXERCISES – Simplify the following :

25 2 1
20 1125 2 2 3 2
1. 9 4 . 6. .
4 720 36 6 6
5 4

1 1 1 1
1 3 2 ℎ 1
2. . 7. .
1 1 3 ℎ ℎ

2 1
1 1 1
3. . 8. .
4 2 1
1 1

2
3 2 3 1
4. 3 * * . 9. .
1 3 1 2
1
1

5 6
2 2 1
5. . 10. . 6
10 2 2 1
2

TOPICS TO BE DISCUSSED ON SEPTEMBER 4 , 2020:

EQUATIONS – PART 1
1. Linear and Rational Equations
2. Quadratic and Rational Equations (Up to two variables)
3. Systems of Equations in Two and Three Variables

REFERENCES:
Introductory and Intermediate Algebra by Blitzer
http://www.mesacc.edu/~scotz47781/mat120/notes/polynomials/multiplying/multiplying_poly.html
https://www.chilimath.com/lessons/intermediate-algebra/multiplying-polynomials/
Intermediate Algebra by Elayn Martin
https://www.allmathtricks.com/dividing-polynomials/
Schaum’s Outlines of College Algebra
Algebra and Trigonometry by Stewart
https://www.allmathtricks.com/dividing-polynomials/
College Algebra and Trigonometry by Swokowski
https://cdn.kutasoftware.com/Worksheets/Alg1/Multiplying%20Polynomials.pdf
https://78bbm3rv7ks4b6i8j3cuklc1-wpengine.netdna-ssl.com/wp-content/uploads/tutoring/handouts/Long-and-Synthetic-Division.pdf
Precalculus by Openstax
https://www.pinterest.ph/pin/431712314261749310/
https://trans4mind.com/personal_development/mathematics/series/multiNomialExpansion.htm#Squaring_The_Multinomial
MTAP Reviewers
https://www.wtamu.edu/academic/anns/mps/math/mathlab/col_algebra/col_alg_tut7_factor.htm
https://cdn.kutasoftware.com/Worksheets/Alg2/Factoring%20A%20Sum%20and%20Difference%20of%20Cubes.pdf
Algebra for Dummies
Cliff’s Quick Review of Algebra I
Beginning and Intermediate Algebra by Gustafson
https://socratic.org/questions/how-do-you-factor-x-4-64
Algebra and Trigonometry Problem Solvers
https://themathpage.com/Alg/perfect-square-trinomial.htm
Algebra and Trigonometry by Larson
https://mcckc.edu/tutoring/docs/bt/algebra/Multiplying_Rational_Expressions.pdf
https://www.wtamu.edu/academic/anns/mps/math/mathlab/col_algebra/col_alg_tut9_mulrat.htm
https://www.mesacc.edu/~scotz47781/mat120/notes/rational/multiplying/mult_rational_practice.html
Precalculus by Sullivan
https://www.deanza.edu/faculty/balmcheryl/114worksheets.pdf
https://mcckc.edu/tutoring/docs/bt/algebra/Multiplying_Rational_Expressions.pdf
http://www.montereyinstitute.org/courses/DevelopmentalMath/COURSE_TEXT2_RESOURCE/U15_L1_T3_text_final.html
College Algebra with Trigonometry by Barnett
https://tutorial.math.lamar.edu/classes/alg/RationalExpressions.aspx
Algebra and Trigonometry by Openstax
https://www.lavc.edu/math/library/math125/Worksheets/simplifycomplexfract.pdf
http://www.teaching.martahidegkuti.com/shared/lnotes/3Algebra2/complexFractions.pdf
Precalculus by Ron Larson
CIVIL ENGINEERING BOARD EXAMS PROBLEMS PHILIPPINES - SEPTEMBER 4 ,2020

EQUATIONS – statements in algebra that represents equality between two expressions called members.

CONDITIONAL EQUATION – equation have one or some true values of the variables.
IDENTITY EQUATION – permissible to all values.

OPERATIONS OF TRANSPOSITION OF EQUATIONS:


If equals are added to equals, the results are equal.
If equals are subtracted from equals, the results are equal.
If equals are multiplied by equals, the results are equal.
If equals are divided by equals , the results are equal as long as divisor is not zero.
The same powers or roots of equals are equal
Reciprocals of equals are equal, provided the reciprocal zero does not occur.

LINEAR EQUATIONS FOR ONE VARIABLE


A linear equation in one variable is an equation that can be written in the form

0.

LITERAL EQUATIONS – used to determine or derive the a value other than the standard one.

1. 2 5 9
EXAMPLES

SOLUTION:

2.

SOLUTION:

3. 4 2 3 2 4 3 3 1
SOLUTION:
3 3
4. 2
1 1

SOLUTION:

But x ≠ 1 , therefore, no solution.

3 7
5. 1 0
4 8

SOLUTION:

1
6. ∶ !
2

SOLUTION:
EXERCISES - Solve the indicated value asked.

4 2 5 5
1. 7 "# . 13 11. 2 3 4 1 "# .
5 13 2
11 2 3
2. 3 2 12 5 1 "# . 12. 4 "# . $ !% #
2 3 3
& 2 3
3. & & !, . "# . 13. 3 2 1 "# .
&!

1 5 + ) 2
4. ( )* , , . "# . 14. 20- . 8! , .. "# . . 30- 12!
) ( 5 + 3

17 /
5. 3 34 13 2 "# . 15. / 3 5 , . "# .
5 3 5

2 3 7 1 1 1
6. "# . 1 16. "# . $ !% #
2 2 1 1

3 1
7. 0.25 60 0.10 0.15 60 "# . 120 17. 4 3 16 "# . %# %#%! !% #
4 4

1 8
8. 5|1 4 | 15 0 "# . 1 , 12 18. 3 2 1 3 1 "# .
2 5

28 3 62
9. 5 9 3 19 "# . 19. 23 1.24 "# . 3
8 0.4 75

4 6 4
10. 4 " 5 6, . "# . 20. 1 4 15 "# . 4 50
" 5 5

RATIONAL EQUATIONS

Rational equations are equations that involves at least one rational equation. These equations can be resulting into linear, quadratic or polynomial .

3 1 1
EXAMPLES:
1.
2 5

SOLUTION:

2. .
1
2. 1 ;
SOLUTION:

1 1
* , * , ;

5 3 2
3.
2 4 8 3

SOLUTION:

1 10 1
4.
5 8 5 25 5
SOLUTION:

6 3
5. 9
3 3

SOLUTION:

9 21 3 ; 9 18 , 2
EXERCISES – Solve the value of the equation indicated.

8 8 3 2 4 2
1. 4 "# . $ !% # 9. "# . 90, 4:
1 1 4 2 4

) 8 ) 8 2 3
2. ; ; +. "# . + 10. "# . 11
+ ; 3 2 1

3 5 5 1 4 7 5 10- 2
3. 2 "# . 10 11. "# . - 1
5 5 7 7 1 - 2 - 2 -8 4

1 1 1 +< +8 9
4. +. "# . + 12. 2 "# . 99, 3:
+ +< +8 +< +8 3

5 10 6 3 6
5. 7 "# . 1 , 32 13. "# . 3
2 2 7 4 6

3 3 "6 2= "6
6. 2 "# . $ !% # 14. " 5 , = "# . =
1 1 = " 5 2

1
1 2 1
7. > ; . "# . 15. 3 "# . 4
> 2 1
1

1 3
8. 3 2 "# . 4
2 4

QUADRATIC EQUATIONS

For a quadratic equation in one unknown x is in the form


8 0 , 0

METHODS OF SOLVING:

1. Extraction of roots
2. Factoring
3. Completing the square
4. Quadratic Formula

±√ 8 4
, )- .
2

DISCRIMINANT – part of quadratic formula to determine if the roots of the values.

= 8 4

If b^2 – 4ac > 0 , real and distinct values is expected


If b^2 – 4ac = 0 , real and equal values is expected.
If b^2 - 4ac < 0 , imaginary values is expected.

SUM AND PRODUCT OF THE ROOTS


Sum of the roots - x1 + x2 = -b/a
Product of the roots – x1⋅ x2 = c/a
1. 8 2 1 0
EXAMPLES:

SOLUTION:
By factoring:

1 1 0 ; 1 . "# .

2. 2 8 2 7 0
SOLUTION:

3. 4 8 7 2 0
SOLUTION:

4. Bℎ ! !ℎ . !% # % 4 ± 5%. = ! )%# !ℎ . 3 !% . !% #.
SOLUTION:

5. 6 3 8 4 0
SOLUTION:
6. Find the value of k for each of the following quadratic equations so that they have two equal roots. : 2x^2 + kx + 3 = 0 .
SOLUTION:

EXERCISES A: Solve the following quadratic equations.

3
1. 5 8 6 1 0 "# . 9 0.2 , 1: 9. 2 8 3 "# . 10, 2
2

5 3 2
2. 8 8 2 15 0 "# . 1 , 2 10. 49)8 28) 4 0 "# . )
4 2 7

3. 4! 8 11! 3 0. "# . ! 92.443, 0.307: 11. 8


2 √6 2 0 "# . √6 ± 2√2

4. 8
18 45 0 "# . 915 , 3: 12. 8
8 4 "# . 2 ± 2%

2 % 7 ± √103
5. 6D 5 3 5 8 1 4 "# . ± 13. 6 8 9 14 "# .
5 5 6

3 1 3 1 8
25 4 2
6. * , 2* , 6 0 "# . 3 1 , 192 14. 15 8
2 8 0 "# . 1 , 2
9 9 2 5 3

15 ± 7√5 5 7 ± 3√41
7. 1048 304 2 "# . 4 15. 4 5 "# .
10 3 8

7 ± √305
8. 8#8 7# 15 7 "# . #
16

EXERCISES B – Answer the following questions


1. Describe the discriminant of the equation x2 +4x -5 . Ans. Real and distinct
2. Write the quadratic equation given the [± 5i] Ans. x2 +25 = 0
3. The quadratic equation 2x2 – 7x – 5 = 0 has the roots a and b. Find a2 + b2 . Ans. 17.25
4. If the root of the equation x2 – 6x + k = 0 is 4, find the value of k. Ans. 8
5. If one root of the 5x2 + 13x + k = 0 is reciprocal of the other, find the value of k. Ans. 5

RATIONAL EQUATIONS - the resulting equation contains a variable in the radicand . The resulting equation can be linear, quadratic or polynomial.

EXAMPLES
1. √ 2
SOLUTION:
2. 4 √ 6 2
E

SOLUTION:

2
3. √ 1

SOLUTION:

4. 11 1.69√ 4.45 3.49


SOLUTION:

5. F G 3 8 1
3

SOLUTION:

6. 1 √1 √2 4
SOLUTION:
EXERCISES - Solve the following equations .

1. √3 4 1 √2 1 "# . 90,4: 9. √ 3 5 112 "# . 11446

8 I<
2 √ 10H √8 9H "# . H 8 10. I
J 3 J 1 0 "# . 122.99186

5 √ 4
3. √ 1 √2 1 "# . 8 11. 2√ 1 "# . 3
√ 1 2

4. 4 √ 2 "# . 7 12. √2K 3 K 1 "# . $ !% #

169 3√
5. L√ 3 √ 3 "# . 13. 6 "# . 4
9 1 √

6. 3 √2 3 "# . 2 14. √ 1 7 "# . 10

4 2 1 10 M1
7. M "# . 15. L1 √1 ⋯ "# . 0.618
4 2 2 3

8. √3! 14 ! 6 "# . ! 5

SYSTEM OF EQUATIONS IN TWO AND THREE VARIABLES

DEPENDENT - all values of two unknowns are permissible when eliminating the system , lines are coincided.
CONSISTENT – distinct value of two unknowns are permissible during elimination process , lines are intersecting.
INCONSISTENT – doesn’t have solution if constant is not equal to zero during elimination process., lines are parallel

METHODS:
1. Substitution
2. Elimination
3. Cramer’s Determinants

1. 44 8 , 5 204 40
EXAMPLES:

SOLUTION:

2. 3 4 7 ; 2 34 1
All values are permissible. Ans.

SOLUTION:
3. H 8 , 4 2H 17 , 24 H 16
SOLUTION:

23 1 4
4. 7 , 2
4 4
SOLUTION:

5. 24 H 4 , 44 2H 6 ,2 34 H 3
SOLUTION:
SYSTEM OF NON LINEAR EQUATIONS IN TWO UNKNOWNS

Methods : Elimination and Substitution

EXAMPLES:

1. 2 4 3, 8
4 2
SOLUTION:

2. G 4 0 , 2 8 4 0
SOLUTION:

3. 4 3 , 4 2
SOLUTION:
4. 8 3 4 24 8 3, 8
5 4 64 8 15
SOLUTION:

5. 4 3 , 8
48 12
SOLUTION:
EXERCISES - A , Solve the following systems of linear equations .

5 11
1. 4 , 24 7 5 "# . 3,4
7 7

2. 7 54 H 0 , 34 2H 16 , 64 H 18 "# . 2,2,4

3. 2 54 22 , 54 4 "# . 6, 2

62 12
4. 3 44 6 ,2 54 8 "# . * , ,
23 23

5. 5 24 12 , 2 34 1 "# . 2, 1
4 4
6. 3 , 1 "# . 7.5 , 7
3 6 2 4

7. 4 H 6, 4 H 0, 4 H 7 "# . $ !% #

1 4 1 4 7
8. , "# . *5, ,
4 27 2 4 33 4

9. 94 7 , 184 4 26 "# . 2 ,1

2 4 4 H 1 4 1 H 9 5 4 1 H 2 19
10. 0 , , "# . 4,8,6
6 3 2 2 2 4 2 4 3 2 4
1 5
11. 3 54 2, 4 34 1 "# . * , ,
11 11
1 4
12. 4 1 ,2 4 2, 4 4 H 8 * , , 8,
3 3

13. 5 34 10 , 10 64 0 "# . $ !% #

14. 2 4 3, 4 2 "# . 1, 1

72 15
15. 34 9 ,2 74 3 "# . * , ,
13 13

16. 2 4 1 ,4 2 5 "# . $ !% #

4 3 2 5 1 1
17. 18 , 4 "# . * , ,
3 2

3 1 1 3 1
18. 4 , 4 "# . * 1, ,
4 2 2 2 2

19. 12 34 15 , 2 34 13 "# . 2, 3

3 12
20. 34 7H 13 , 4 H 1, 24 3H 4 "# . * 2, , ,
5 5
EXERCISES B - Solve the system of non linear equations.

1. 4 1 ,4 8 1 "# . 1,2 , 0,1

2. 8 48 64 7, 8
4 1
Ans:

3. 2 8 4 1 8 1 , 4 3 "# . $ !% #

4. √ 4, 8
48 6 "# . $ !% #

5. 4 8
2 4 ,4 8
2 6 "# . 1,7 , 1 ,3

6. 8
24 10 , 3 4 9 "# . 4,3 , 2, 3

7. 24 0, 8
48 5 "# . 2, 1 , 2,1

1
8. 4 8
48 5, 4 "# .

9. 2 8
48 18 , 4 4 "# .

10 . 8 48 26 , 3 8 254 8 100 "# . 5,1 , 5,1 , 5, 1 , 5, 1

NEXT TOPIC ON SEPTEMBER 7 - WORD PROBLEMS


REFERENCES

Scahum’s Outlines of College Algebra


Intermediate Algebra by Elayn Martin
Engineering Mathematics by John Bird
Algebra and Trigonometry by Cynthia Young
Algebra and Trigonometry Problem Solvers
http://www.mathcentre.ac.uk/resources/Algebra%20leaflets/mc-simplelinear-20090-1.pdf
Algebra and Trigonometry by Openstax
Introductory and Intermediate Algebra by Blitzer
https://tutorial.math.lamar.edu/classes/alg/solvemultivariable.aspx
https://www.math-only-math.com/method-of-solving-a-linear-equation-in-one-variable.html
https://opentextbc.ca/collegealgebraopenstax/chapter/linear-equations-in-one-variable/
https://www.mcckc.edu/tutoring/docs/br/math/equat_inequ/Solving_Linear_Equations.pdf
https://byjus.com/maths/linear-equation-in-one-variable/#:~:text=In%20One%20Variable-
,Linear%20Equation%20in%20One%20Variable,a%20single%20variable%20in%20it.
https://www.myprivatetutor.my/prime/documents/ppts/details/53/ppt-on-linear-equation-with-one-variable
College Algebra with Trigonometry by Barnett
Schaum’s Outlines of College Mathematics
Schaum’s Outlines of Elementary Mathematics
https://www.mcckc.edu/tutoring/docs/br/math/equat_inequ/Solving_Literal_Equations_Methods.pdf
Glencoe Mathematics Pre Algebra
https://www.mathbootcamps.com/how-to-solve-linear-equations/
https://www.chilimath.com/lessons/advanced-algebra/solving-rational-equations/
PreCalculus by Ron Sullivan
https://mcckc.edu/tutoring/docs/br/math/equat_inequ/Practice_Solving_Literal_Equations.pdf
https://www.mesacc.edu/~scotz47781/mat120/notes/rational/solving/solving_practice.html
Precalculus by Cynthia Young
https://courses.lumenlearning.com/intermediatealgebra/chapter/read-solve-rational-equations/
Engineering Mathematics by John Bird
http://www.montereyinstitute.org/courses/DevelopmentalMath/COURSE_TEXT2_RESOURCE/U15_L2_T1_text_final.html
https://www.mathbootcamps.com/the-quadratic-formula/
http://www.wallace.ccfaculty.org/book/9.5%20Quadratic%20from%20Roots.pdf
College Algebra in Context by Yocco
https://www.mathsisfun.com/algebra/quadratic-equation.html
Higher Engineering Mathematics by John Bird
https://www.mathworksheets4kids.com/quadratic-equation/square-difficult-2.pdf
https://sccollege.edu/Faculty/epham/Documents/IntroAlgManuscript/Chapter%2013%20-Quadratic%20equations.pdf
Glencoe’s Advanced Mathematical Concepts
Licensure Examinations by Besavilla
http://www.mesacc.edu/~scotz47781/mat150/notes/solve_quads/Solving_Quads_Quad_Formula_Practice.pdf
https://www.intmath.com/quadratic-equations/sum-product-roots-quadratic-equation.php
Intermediate Algebra by Kaufmann
http://www.montereyinstitute.org/courses/DevelopmentalMath/COURSE_TEXT2_RESOURCE/U16_L3_T1_text_final.html
https://www.wtamu.edu/academic/anns/mps/math/mathlab/col_algebra/col_alg_tut19_radeq.htm
https://www.ixl.com/math/algebra-1/solve-radical-equations-ii
Engineering Mathematics vol. 1 by Gillesania
PreCalculus by Blitzer
https://www.chilimath.com/lessons/advanced-algebra/systems-non-linear-equations/
Schaum’s Outlines of Advanced Math
https://people.ucsc.edu/~miglior/chapter%20pdf/Ch02_SE.pdf
https://courses.lumenlearning.com/wmopen-collegealgebra/chapter/introduction-systems-of-linear-equations-two-variables/
https://sccollege.edu/Faculty/epham/Documents/IntroAlgManuscript/Chapter%206%20-
%20Systems%20of%20two%20linear%20equations%20with%20two%20variables.pdf
https://www.wtamu.edu/academic/anns/mps/math/mathlab/col_algebra/col_alg_tut49_systwo.htm
http://www.mesacc.edu/~scotz47781/mat150/notes/cramers_rule/Cramers_Rule_2_by_2_Notes.pdf
PreAlgebra by Glencoe Mathematics
https://www.yumpu.com/en/document/read/10778988/systems-of-two-equations-cramers-rulepdf-kuta-software
https://math.libretexts.org/Bookshelves/Precalculus/Book%3A_Precalculus_(OpenStax)/09%3A_Systems_of_Equations_and_Inequalities/909%3A_Solvin
g_Systems_with_Cramers_Rule
https://openstax.org/books/college-algebra/pages/7-3-systems-of-nonlinear-equations-and-inequalities-two-variables
Precalculus by Openstax
https://www.wtamu.edu/academic/anns/mps/math/mathlab/col_algebra/col_alg_tut52_nonlinear_sys.htm
https://courses.lumenlearning.com/precalctwo/chapter/systems-of-nonlinear-equations-and-inequalities-two-variables/
Intermediate Algebra by Blitzer
College Algebra Demystified
http://teachers.dadeschools.net/lberkson/Documents/Ch7_Section4.pdf
CIVIL ENGINEERING BOARD EXAMS PROBLEMS PHILIPPINES – SEPTEMBER 7, 2020

WORD PROBLEMS

To prepare for problem solving, we practice writing algebraic expressions that can be simplified.

General Strategy for Problem Solving

1. UNDERSTAND the problem. During this step, become comfortable with the problem. Some ways of doing this are to:
Read and reread the problem.
Propose a solution and check.
Pay careful attention to how you check your proposed solution. This will help when writing an equation to model the problem.
Construct a drawing.
Choose a variable to represent the unknown. (Very important part)
2. TRANSLATE the problem into an equation.
3. SOLVE the equation.
4. INTERPRET the results: Check the proposed solution in the stated problem and state your conclusion.

CLASSIFICATION OF WORD PROBLEMS (in one or more variables, can be linear, quadratic or polynomial)

1. Digit and Numerical Problems


2. Age Problems
3. Clock Problems
4. Mixture Problems
5. Motion and Distance Problems
6. Rate and Work Problems
7. Investment Problems
8. Miscellaneous Problems (optional)

Note: Geometric Problems will be discussed on Solid Mensuration.

DIGIT AND NUMERICAL PROBLEMS

DIGIT PROBLEMS – involves individual digits in integers and how digits are related according to the question.
NUMBER OR NUMERICAL PROBLEMS- involves numbers that can be consecutive or related to each other.

Examples:
1. Twice the larger of two numbers is three more than five times the smaller, and the sum of four times the larger and three times the smaller is 71.
What are the numbers?
SOLUTION:
2. The sum of the seven consecutive integers is 77. Find the smallest integer.
SOLUTION:

Alternate: (x -3) + (x-2 ) + (x-1) +x + (x – 1 ) +(x +2 ) + (x+3) = 77 . x = 11 . , x -3 =8.

3. The sum of two numbers is 27 and their product is 50. Find the numbers.
SOLUTION:

4. The tens digit of a number is twice the units digit. If the digits are reversed, the new number is 27 less than the original. Find the original number.
SOLUTION:

EXERCISES – Solve the following problems


1. When a two-digit number is divided by the product of the two digits, the quotient is 2. If 27 is added to the number, the original number turns into
a new number with the digits being swapped around. Find the number. Ans. 36
2. Only once, in 1991, were the winning and losing scores in the Super Bowl consecutive integers. The New York Giants beat the Buffalo Bills in a
nearly error-free game. If the sum of the scores was 39, determine the points scored by the losing team, the Bills, and the winning team, the Giants.
Ans. Giants 20 , Bllls 19
3. Find three consecutive even integers so the sum of the first three is 2 more than the twice the fourth. Ans. 8, 10 and 12
4. Twice the sum of a number and 3 is the same as five times the number, minus 1, minus four times the number. Find the number. Ans. 5
5. Divide 25 into two parts so that the sum of the reciprocals is 1/6. Ans. 10 and 15
6. The sum of the two numbers is 12 and the sum of their squares is 80. Find the numbers. Ans. 4 and 8
7. The sum of the digits of a three digit number is 21. If the digits are reversed , the new number is 198 greater than the original. The sum of the first
and third digits is one more than three times the second digit. Find the number. Ans. 759

AGE PROBLEMS

Algebra age problems contain the problems based on present age or some year hence.

EXAMPLES:

1. In January of the year 2000, I was one more than eleven times as old as my son Will. In January of 2009, I was seven more than three times as old
as him. How old was my son in January of 2000?
SOLUTION :
2. Mary is 24 years old. Mary is twice as old as Anna was when Mary was as old as Anna is now. How old is Anna now?
SOLUTION:

3. Al's father is 45. He is 15 years older than twice Al's age. How old is Al?
SOLUTION:
2x+ 15 = 45 , x = 15 Ans.

EXERCISES – Solve the following problems

1. Two years ago, a man was six times as old as his daughter. In 18 years, he will be twice as old as his daughter. Determine the present ages.
Ans. Man – 32 and daughter – 7
2. Yasir is fifteen years elder than Mujtaba. Five years ago, Yasir was three times as old as Mujtaba. Then Yasir’s present age will be ___.
Ans. 29
3. Five years ago, John’s age was half of the age he will be in 8 years. How old is he now? Ans. 18
4. A wool tapestry is 32 years older than a linen tapestry. Twenty years ago, the wool tapestry was twice as old as the linen tapestry. Find the present
age of each. Ans. Wool – 84 and linen -52
5. The ratio of John’s age to David’s age is 6:5. In 7 years, the ratio will be 7:6. What are their ages now? Ans. John 42 and David 35
6. Danielle's mom is 3 less than four times her daughter's age. If M is her mother’s age, what is Danielle’s age in terms of M? Ans. D = (M+3)/4
7. A woman’s age is 120% of what is was 15 years ago but 75% of what will be after 15 years. What is her present age? Ans. 65

CLOCK PROBLEMS

These problems relate two distinct measurements: time (can be in seconds, minutes and hours) and angles (can be in degrees, radians or revolutions) . The
angle is measured in degrees from the mark of 12 clockwise. The time is usually based on a 12 hour clock (can be 24 hour clock)
For clocks that is superimposed:

EXAMPLES
1. At what time between 10 and 11 will the minute and hour hand be at right angles for the first time?
SOLUTION:
Right angles will be 15 minutes. But since 10 pm is 60° ,

15 10 , 5.45454545 .
12

2. The hands on the clock shows the time 12 hours and 2 minutes. Calculate the size of a sharp angle between clock hands three hours later.
SOLUTION:

3. A clock is set right at 8 a.m. The clock gains 10 minutes in 24 hours. What will be the true time when the clock indicates 1 p.m. on the following
day?
SOLUTION:

EXERCISES – Solve the following equations


1. At what time between 2 and 3 o’clock will the hands of a clock together? Ans. 2:10 10/11
2. At what time between 4 and 5 are the minute and hour hand at right angles for the second time? Ans. 4:38 2/11
3. (CE Board) What time between 2 and 3 o’clock will the angle between the hands of the clock be bisected by the line connecting the center of the
clock and the 3 o’clock mark? Ans. 2:18 6/13
4. How many times in a day, the hands of a clock are straight? Ans 44
5. My watch, which gains uniformly, is 2 min, & show at noon on Sunday, and is 4 min 48 seconds fast at 2 p.m on the following Sunday when was it
correct? Ans. Tuesday 2 pm
6. When the minute hand and the hour hand coincide (have the same direction) for the first time after the noon? Ans. 1:05:27
MIXTURE PROBLEMS

These are problems that involves creating a mixture from two or more things and then determining some quantity (such as percentage, price, etc.. ) of the
resulting mixture.

EXAMPLES :

1. How many pounds of cream containing 12% butterfat must be added to 1800 lbs of milk containing butterfat to obtain a milk having 3% butterfat?
SOLUTION:

2. You need to mix a solution that is 70% antifreeze with one that is 30% antifreeze of a mixture to obtain 20 liters of a mixture that is 60% antifreeze.
How many liters of each liters of the solutions must be used?
SOLUTION:

0.7x + 0.3(20 – x ) = 20(0.60) , upon solving equations results to x = 15 liters for the 70% and 5 liters for the 30%. Ans.

3. The manual for your new car suggests using gasoline that is 89 octane. In order to save money, you decide to use some 87 octane and some 93
octane in combination with the 89 octane currently in your tank in order to have an approximate 89 octane mixture. Assuming you have 1 gallon of
89 octane remaining in your tank (your tank capacity is 16 gallons), how many gallons of 87 and 93 octane should be used to fill up your tank to
achieve a mixture of 89 octane?
SOLUTION:

EXERCISES – Solve the following equations.


1. A mechanic has 340 gallons of gasoline and 10 gallons of oil to make gas/oil mixtures. He wants one mixture to be 4% oil and the other mixture to
be 2.5% oil. If he wants to use all of the gas and oil, how many gallons of gas and oil are in each of the resulting mixtures? Ans. Gasoline 83.333
gallons (4%) and 266.67 gallons (2.5% mixture) and Oil – 3.3333 (4%) and 6.6667 (2.5%)
2. Rabbits in a lab are to be kept on a strict daily diet that includes 30 grams of protein, 16 grams of fat, and 24 grams of carbohydrates. The scientist
has only three food mixes available with the following grams of nutrients per unit.

Find how many units of each mix are needed daily to meet each rabbit’s dietary need. Ans. A = 2 units, B = 3 units , C = 1 unit
3. The gasoline tank of a car contains 50 liters of gasoline and alcohol, the alcohol comprising 25%. How much of the mixture must be drawn off and
replaced by alcohol so that the tank contain a mixture of which 50% is alcohol? Ans. 16.67 liters.
4. A coffee mix is to be made that sells for $2.50 by mixing two types of coffee. The cafe has 40 mL of coffee that costs $3.00. How much of another
coffee that costs $ 1.50 should the cafe mix with the first? Ans. 20 ml
5. How many pounds of chocolate worth $ 1.20 a pound must be mixed with 10 pounds of chocolate worth 90 cents a pound to produce a mixture
worth $ 1.00 a pound? Ans 5 pounds
6. A fruit salad mixture consists of apples, banana and peaches in the ratio 6:5:4 respectively by weight. If 225 grams of mixture is prepared, how
many more grams of apples than peaches? Ans. 30 g.

MOTION AND DISTANCE PROBLEMS

Formula for motion:

Distance equals rate(speed) times time.

EXAMPLES:
1. It takes Andrew 30 min to drive to work in the morning. He drives home using the same route, but it takes 10 min longer, and he averages 10 mi/h
less than in the morning. How far does Andrew drive to work?
SOLUTION:

Now we have to solve for the distance covered:

2
10 , 40 ℎ.
2 3
Solving the distance: 40(1/2) = 20 miles. Ans.

2. Pawnee and Springfield are 420 km apart. A train leaves Pawnee heading toward Springfield at the same time a train leaves Springfield heading
toward Pawnee. One train is moving at a speed of 45 kph, and the other is moving 60 mph. How long will they travel before they meet?
SOLUTION:

Then, the time is to be solved.

, 420 45 60 , 4ℎ . .

3. An airplane flying into a head wind travels the 1800-mile flying distance between two cities in 3 hours and 36 minutes. On the return flight, the same
distance is traveled in 3 hours. Find the ground speed of the plane and the speed of the wind, assuming that both remain constant.
SOLUTION:
Let x – ground speed, and y – wind speed., Then, negative value resisting the wind.

D = rt

1800 = 3.6(x – y) for the inbound flight


1800 = 3(x + y) for the return flight.

Solve the system in two variables:


1800 = 3.6 x – 3.6y , 1800 = 3x + 3y , Using substitution method, Use Eq. 2 , y = 600 – x (* Divide by 3 as common factor)

Then,

1800 = 3.6x – 3.6(600 – x) , 1800 = 3.6x – 2160 + 3.6x , 3960 = 7.2x , x = 550 mph Ground speed. Ans.

For wind speed, y = 600 – x = 600 – 550 = 50 mph wind speed. Ans.

4. Suppose you are at a river resort and rent a motor boat for 5 hours starting at 7 A.M. You are told that the boat will travel at 8 miles per hour
upstream and 12 miles per hour returning. You decide that you would like to go as far up the river as you can and still be back at noon. At what time
should you turn back, and how far from the resort will you be at that time?
SOLUTION:
x - Approaching destination time
5 – x - returning time

Then , d = rt , 8x = 12(5 – x) , x = 3 hours,. So you should go back at 10 AM. Ans.


For distance: 8(3) = 24 miles. Ans.

EXERCISES – Solve the following problems.

1. A boat can travel 16 miles up a river in 2 hours. The same boat can travel 36 miles downstream in 3 hours. What is the speed of the boat in still
water? What is the speed of the current? Ans. 2 mph for current and 10 mph for boat
2. A car passes an intersection heading north at 40 kph. Another car passes the same intersection 15 minutes later heading north traveling at 45 kph.
How long will it take for the second car to overtake the first? Ans. 2 hours
3. A messenger travels from points A to B. If he will leave A at 8 AM and travel at 2 kph, he will arrive at B 3 mins earlier than the expected time of
arrival. However, if he will leave at 8:30 AM and travel at 3 kph, he will arrive 6 mins later than the expected time. What is the expected time of
arrival? Ans. 9:06 AM
4. When a small plane flies with the wind, it can travel 800 miles in 5 hours. When the plane flies in the opposite direction, against the wind, it takes 8
hours to fly the same distance. Find the rate of the plane in still air and the rate of the wind. Ans. 130 mph – still air and 30 mph – wind
5. Two friends, Sherry and Fredrick, live 2000 miles apart. They want to meet each other and decided to both travel so that they can see each other
sooner. If they both drive directly toward each other and Sherry travels at 50 mph and Fredrick travels at 60 mph, how much time will pass before
they meet each other? Ans. 18.18 miles
6. Two motorists start toward each other at 4:30 PM from towns 255 km. apart. If their respective average speeds are 40 and 45 kph, at what time will
they meet? Ans 7:30 PM
7. The distance between two towns is 380 km. At the same moment, a passenger car and a truck start moving towards each other from different towns.
They meet 4 hours later. If the car drives 5 km/hr faster than the truck, what are their speeds? Ans. Car = 50 kph and truck = 45 kph
8. Two trains travel toward each other from points which are 195 miles apart. They travel at rate of 25 and 40 miles an hour respectively. If they start
at the same time, how soon will they meet? Ans. 3 hours

RATE AND WORK PROBLEMS

A rate is a mathematical way of relating two quantities, which are usually measured in different units. A favorite type of a rate problem in algebra
courses sends two hypothetical trains rushing towards each other at different speeds, and asks you to determine when they will meet. Less exciting, but also
common, rate situations involve calculating wages or determining the time it takes for a container to fill or empty. The secret to solving all rate problems is
creating a system of equations to represent the situation.

Work problems have direct real-life applications. We often need to determine how many people are needed to complete a task within a given time.
Alternatively, given a limited number of workers, we often need to determine how long it takes to finish a project. Here we deal with the basic math concepts of
how to handle these types of problems.

EXAMPLES:
1. Sandra , who is paid time and a half for hours worked in excess for 40 hours, had gross weekly wages of $ 442 for 48 hours worked. What is her
regular hourly rate:
SOLUTION:
Time and a half for hours means overtime pay. The value is 1.5 times. So that,

40x + 8(1.5x) = 442., upon direct solving, x = $ 8.50 Ans.

2. One pipe can fill a tank in 5 hours less than another. Together they can fill the tank in 5 hours. How long would it take each alone to fill the tank?
SOLUTION:
The work rate of pipe A is 1/x and the other is 1/x – 5 . Then the rate of fill the tank together is 1/5 (negative sign if drained)
The working equation is to be:

1 1 1
;
5 5
Solve next the equation using quadratic equations. (get the LCD first)

Using quadratic formula gives ,

X = 13.09 or 1.909 but 13.09 is only accepted value.

X – 13.09 hours for first tank and x – 5 = 8.09 hours for the second tank. Ans.

3. Suppose one painter can paint the entire house in twelve hours, and the second painter takes eight hours to paint a similarly-sized house. How long
would it take the two painters together to paint the house?
SOLUTION:
x/12 - painter 1 and x/8 – painter 2 Then,

1 , 4.8 " .
8 12

4. Combined, Megan and Kelly worked 60 hours. Kelly worked twice as many hours as Megan. How many hours did they each work?
SOLUTION:
M – megan , k – Kelly

# 60 , 2#

Using substitution methods leads to

2# # 60 , # 20. .

2 20 40 .

EXERCISES – Solve the following problems.


1. (CE Board Exam) Melissa Scarlatti can clean the house in 4 hours whereas her husband Jack can do the same job in 5 hours. They agreed to clean
together so that they can finish in time to watch a movie on TV that starts in 2 hours. How long will it take to clean the house together? Ans. 20/9
hours
2. If Art and Rita can do a job in 8 hours (working together at their respective constant rates) and Art can do the job alone in 12 hours. In how many
hours can Rita do the job alone? Ans. 24 hours
3. A tank can be emptied by any one of three caps. The first can empty the tank in 20 minutes while the second takes 32 minutes. If all three working
together could empty the tank in 8 8/59 minutes, how long would the third take to empty the tank? Ans. 24 minutes
4. Together, an experienced word processor and an apprentice word processor can create a word document in 6 hours. Alone, the experienced word
processor can create the document 2 hours faster than the apprentice word processor can. Find the time in which each person can create the word
document alone. Ans. 13.1 hours for apprentice and 11.1 hours for experienced
5. Jay and Morgan work in the summer for a landscaper. It takes Jay 3 hours to complete the company’s largest yard alone. If Morgan helps him, it
takes only 1 hour. How much time would it take Morgan alone? Ans. 1.5 hours
6. Working together, Esteban and Ray-J can clean up a room in 8 hours. If Esteban were working alone it will take him 5 hours to complete the task.
How long will it take Ray-J to clean up the room by himself? Ans. 13 hrs 20 mins
7. Rodney and Tomaso can build a cabinet together in 12 hours. Rodney can build the cabinet alone in 16 hours. How long will it take Tomaso to
build the cabinet alone? Ans. 48 hours
8. A pipe can fill the tank in 4 hours. Due to leakage at the bottom it is filled in 5 hours. When the tank is full, in how much time will it be emptied by
the leak?

INVESTMENT PROBLEMS

Investment problems usually involve simple annual interest (as opposed to compounded interest), using the interest formula I = Prt, where I stands
for the interest on the original investment, P stands for the amount of the original investment (called the "principal"), r is the interest rate (expressed in decimal
form), and t is the time.

For annual interest, the time t must be in years. If they give you a time of, say, nine months, you must first convert this to 9/12 = 3/4 = 0.75 years.
Otherwise, you'll get the wrong answer. The time units must match the interest-rate units. If you got a loan from your friendly neighborhood loan shark, where
the interest rate is monthly, rather than yearly, then your time must be measured in terms of months. (This topic will be discussed further in Engineering
Economics)
EXAMPLES
1. You have $50,000 to invest, and two funds that you'd like to invest in. The You-Risk-It Fund (Fund Y) yields 14% interest. The Extra-Dull Fund (Fund
X) yields 6% interest. Because of college financial-aid implications, you don't think you can afford to earn more than $4,500 in interest income this
year. How much should you put in each fund?
SOLUTION:

2. (ECE Board) If you owned a sari sari store in Kuwait at what price will you mark a small camera for sale that cost P 600 in order that you may offer
20% discount on the marked price and still makes a profit of 25% on the selling price?
SOLUTION:

3. As an incentive plan, a company stated that employees who worked for four years with the company would receive $516 and a laptop computer.
Mr. Rodriquez worked for the company for 3.5 years. The company pro-rated the incentive plan, and he still received the laptop computer, but only
$264. What was the value of the laptop computer?
SOLUTION:

Note: Some value related topics will be discussed further on Engineering Economics.

EXERCISES – Solve the following problems.


1. Danny invested $11,000. Part of his money is invested in bonds which yield 8% and the remainder is invested in bonds which yield 10%. His total
annual income from these bonds is $1,020. Find the amount he has invested in each kind of bond. Ans. $ 4,000 at 8% and $7,000 for 10%
2. By selling 5 php per dozen, a vendor gains 20%. The cost of the eggs rise by 12.5% , If he sells at same price as before, find his new gain in
percent. Ans. 6.67%
3. A man has invested $ 2,000 invested at 7% and $5000 at 4% simple interest. What additional sum must he invest at 6% to give him an overall return
of 5%? Ans. $ 1,000
4. Suppose you have P 12,000 to invest. If part is invested at 10% and the rest is 15%, how much should be invested at each rate to yield 12% on the
total amount invested? Ans. P 7,200 at 10% and P 4,800 at 15%
5. A recently retired couple need $ 12,000 per year to supplement their social security. They have $ 150,000 to invest to obtain this income. The have
decided on two investment options: AA bonds yielding 10% per annum and a Bank Certificate yielding 5%. How much should be invested in each
to realize exactly $ 12,000? Ans. $ 90,000 for the AA Bonds and $ 60,000 for Bank Certificates
6. A man leaves P 60,000 to his children and grandchildren seven in all. The children received 1/3 of it, which is P 2,000 more apiece than what the
grandchildren get. How much is the amount received by each grandchildren? Ans. P 8,000

MISCELLANEOUS PROBLEMS (Geometric related problems are excluded and to be discussed on solid mensuration)
EXAMPLES:

1. The price of gasoline increased by 25% between January and March. If the price per gallon in March was $1.15, what was the price per gallon in
January?
SOLUTION:

2. In the United States, 7 out of every 25 homes are heated by electricity. At this rate, how many homes in a community of 36,000 homes would you
predict are heated by electricity?
SOLUTION:

3. Peter has six times as many dimes as quarters in her piggy bank. She has 21 coins in her piggy bank totaling $2.55 How many of each type of coin
does she have?
SOLUTION:
0.10x – no. of dimes. , 0.25(21 – x) – no. of quarters.
Then,
0.10x + 0.25(21 – x) = 2.55 , x = 18 – dimes, 21 – x = 3 quarters. Ans.

4. Ahmed buys some shoes online for $32.00 per pair including tax, and the shipping is $4.00. If he decides to buy more pairs of those same shoes
and it doesn't change the shipping cost, how many can he buy with $100.00?
SOLUTION:
Let x – no. of pairs of shoes, then ,
32x + 4 = 100 , x = 3 pairs Ans.
EXERCISES – Solve the following problems

1. The body surface area can be modeled in the function

'ℎ
$% &
3600

Where w – weight and h – height.


The BSA of a male 177 cm. tall is 2.1 . Find his weight. Ans. 89.69 kg.

2. An investor purchased a group of lots for P 8.4 M. All but 4 lots were later sold for a total of P 8.4 M. The selling price of each lot was P 350,000
greater than the cost. How many lots were originally purchased? Ans. 12
3. (EE Board Exam) A student has test scores of 75,83, and 78. The final has a weight equal to one third of the total grade, what should the student
strive for a minimum final test score so that he gets a passing minimum average of 80? Ans. 82.66
4. The cost of admission to a popular music concert was $162 for 12 children and 3 adults. The admission was $122 for 8 children and 3 adults in
another music concert. How much was the admission for each child and adult? Ans. $ 10 per child and $ 14 per adult
5. A group of 266 persons consists of men, women, and children. There are four times as many men as children, and twice as many women as
children. How many of each are there? Ans 38, 152 and 76
6. The PAL aircraft has 104 seats more than the Cebu Pacific aircraft. If their total number of seats is 328 , find the number of seats for the PAL aircraft.
Ans. 216
7. A company produces Italian sausages and bratwursts at plants in Green Bay and Sheboygan. The hourly production rates at each plant are given
in the table. How many hours should each plant be operated to exactly fill an order for 62,250 Italian sausages and 76,500 bratwursts?
Ans. Green Bay = 60 hours, Sheboygan = 28.5 hours
8. At a restaurant four people order fried crab claws and four people order a cup of gumbo, with a total bill of $31. If only two people had ordered the
crab claws and one person ordered the gumbo, the bill would have been $12.25. How much is each order of fried crab claws and each cup of
gumbo? Ans. $ 4.50 for crab claws and $ 3.25 for gumbo
9. Mr. Phillips has a total of 26 dimes and quarters in his desk drawer, totaling $3.65. How many of each type of coin does he have? Ans. 19 dimes
and 7 quarters
10. A certain farmer only raises chickens and pigs. Altogether, the animals have 57 heads and 158 legs. How many chickens does he have? Ans. 35

NEXT TOPIC ON SEPTEMBER 11, 2020: - FUNCTIONS AND INEQUALITIES


REFERENCES

Intermediate Algebra by Elayn Martin


Engineering Mathematics by Gillesania
https://www.onlinemathlearning.com/digit-word-problems.html
https://www.purplemath.com/modules/numbprob.htm
Cayley Math Olympiad
https://www.vivaxsolutions.com/maths/gcseqdrtcwrdproblems.aspx
http://webhost.trafalgar.qc.ca/Math/flashphp/form%20III/Quadratic%20Equations/Factorable/Word%20Problems/Sheet%203/QUAD%20EQN%20WORD%20
-%20WKS%203%20-%20SOLN.pdf
https://library.vcc.ca/learningcentre/pdf/vcclc/MathProb-TwoAndThreeVariableWordProblems.pdf
https://www.purplemath.com/modules/ageprobs.htm
https://mathalino.com/reviewer/algebra/example-03-age-related-problem-
algebra#:~:text=Mary%20is%2024%20years%20old,old%20as%20Anna%20is%20now.
Schaum’s Outlines of College Algebra
https://www.onlinemathlearning.com/age-problems.html
http://www.wallace.ccfaculty.org/book/1.9%20Age.pdf
http://testprepshsat.com/wp-content/uploads/2017/01/L7_Word-Problems_Age_Time.pdf
http://bankersdaily.in/practice-difficult-problems-ages-questions-asked-competitive-exams/
https://www.hitbullseye.com/Quant/Problems-on-Clocks.php
https://www.hackmath.net/en/math-problem/1594
Engineering Mathematics Vol. 1 by Besavilla
https://gradeup.co/short-tricks-to-solve-clock-questions-in-reasoning-i-df122ec3-c8dd-11e5-b53a-34fcf7b52438
https://www.mbarendezvous.com/cat/quantitative-ability/important-concepts-and-formulas-on-clock-problems/
Algebra and Trigonometry by Cynthia Young
http://www.wallace.ccfaculty.org/book/4.6%20%20Mixture.pdf
https://www.onlinemathlearning.com/math-mixture-problems.html
UPCAT Math Reviewer
Introductory Algebra by Blitzer
https://www.onlinemathlearning.com/distance-problems.html
College Algebra and Trigonometry by Barnett
http://www.umsl.edu/~defreeseca/intalg/ch8extra/rate.htm
Algebra Demystified
http://www.mathguide.com/lessons2/Distance.html
http://www.wallace.ccfaculty.org/book/1.10%20Distance%20Practice.pdf
College Algebra by Ron Sullivan
https://www.onlinemathlearning.com/work-problems.html
https://content.nroc.org/Algebra.HTML5/U06L2T1/TopicText/en/text.html#:~:text=Rate%20Problems&text=A%20rate%20is%20a%20mathematical,determine
%20when%20they%20will%20meet.
https://www.cliffsnotes.com/study-guides/algebra/algebra-i/word-problems/number-problems-with-two-variables
Licensure Examinations by Besavilla
https://www.hitbullseye.com/Time-and-Work-Problems.php
http://www.wallace.ccfaculty.org/book/9.8%20Teamwork.pdf
http://ww.mathgotserved.com/22-combined-work-word-problems-college-algebra-2-practice-worksheet-pdf-doc.html
https://www.math-only-math.com/worksheet-on-pipes-and-water-tank.html
https://www.purplemath.com/modules/investmt.htm
Glencoe’s Advanced Mathematical Concepts
https://hsorkin.com/Algebra/Investment_Mixture/Investment-Mixture.htm
Precalculus by Blitzer
https://www.varsitytutors.com/hotmath/hotmath_help/topics/writing-systems-of-linear-equations-from-word-problems
https://themathpage.com/Alg/word-problems.htm
https://mgccc.edu/learning_lab/math/alg/sysword.pdf
MTAP Reviewer
CIVIL ENGINEERING BOARD EXAMS PROBLEMS PHILIPPINES – SEPTEMBER 11, 2020

FUNCTIONS AND RELATIONS

Relations – set of ordered pairs


Domain – set of all first components of the ordered pairs.
Range – set of all second components of the ordered pairs
Functions – relations do correspond to only one second component.

Ways of determining the functions


1. Ordered pairs – The domain should have the range only once.
2. One to One Chart – it can be one to one and no more than one element in the domain. It can be one to one or many to one. Many to many may vary.
3. Vertical Line Test – the graph should pass the y ordinates once.
4. Solving for Y – the value of y should be no more than one value

EXAMPLES
1. f = {(0,0) , (1,1) , (1 , -1)}
SOLUTION:

2. y^2 = 2x
y has two values, it remains a relation.

3.
SOLUTION: Using Vertical Line test, the graph is a function.

4.
Using the many to one correspondence, The graph is a function.

EXERCISES - Determine if the ordered pairs, graph or equation is a relation or function.

1. Equation of a circle - Ans. Relation


2. {(3,0) , (4,2) , (2, -6)} - Ans. Function

3. Ans. Function
4. Ans. Relation
5. g(x) = √(x^2 – 9) Ans. Relation
6. {(1,b) , (1,c)} Ans. Relation

7. Ans. Function

8. Ans. Relation

9. Ans Function

10. Ans. Relation

DETERMINING DOMAIN AND RANGE OF THE FUNCTIONS

Another way to identify the domain and range of functions is by using graphs. Because the domain refers to the set of possible input values, the domain of a graph
consists of all the input values shown on the x-axis. The range is the set of possible output values, which are shown on the y-axis. Keep in mind that if the graph continues
beyond the portion of the graph we can see, the domain and range may be greater than the visible values.

In this section, we will practice determining domains and ranges for specific functions. Keep in mind that, in determining domains and ranges, we need to consider what is
physically possible or meaningful in real-world examples, such as tickets sales and year in the horror movie example above. We also need to consider what is mathematically
permitted.

EXAMPLES:

1.
SOLUTION:
Domain = all real values

2. √ 2
SOLUTION:
15
3.
3

SOLUTION:

EXERCISES – Determine the following domain and range.

2 5
1. . , 0
√2 5 2

2. . 3,
√ 3

2 3
3. . 4 ! " ",
4
1 1
4. # . 9 ! " " ,% 0 , # &0, # %
3 √# 3

5. 3 6 1 . "

6. . (1,
1
3
7. , . (2, (3
2

4
8. , . (0,

9. 2 8 3 . "

10. √2 . +2, +0

TYPES AND EVALUATION OF FUNCITONS

Note: Even and Odd Functions will be discussed on trigonometry.

1. Constant function - functions have constant values of its range and the line is parallel to x – axis.
2. Polynomial Functions – these functions are in the form with degree n.
, -
.
/ .01 .01 / ⋯ / 1 3

Where a_n , a_n-1 , … , a_1 and a_0 are real numbers and x ≠ 0.

3. Linear Functions – these functions have degree of 1 and straight lines


4. Quadratic Functions – these functions have degree of 2 and they’re in form of parabola.
5. Exponential Functions – these functions that involves exponents
6. Logarithmic Functions – involves logarithms whether if it is natural or common logarithms
7. Absolute Value Function – f(x) = |x| , wherein the range should be minimum of zero.
8. Identity functions – Let f : a → b be the function f.
9. Trigonometric Functions – involves trigonometric identities
10. Greatest Integer or Floor Function – these have greatest integer less than or equal to the designated variable.
EVALUATION OF FUNCTIONS – Same approach in solving algebra .

Let f and g be two functions. All operations of functions have all real numbers in its domains.

1. 4 5 6 " ! /7 /7

2. , 6" !# 7 ⋅7

3. 9 6# # : ; ,7 (0
7 7

4. <65=6 # 6 7 ∘7

1. ? # "7 . @ " #ℎ 6 6B 7
a. 7 4
b. (fg)(x)
c. (f/g) (x)
SOLUTION:

4
2. ? # "7 .@ " 3 .
1 3 7

SOLUTION:
By Using Complex Fractions,

Then,

3. C #ℎ !# 6 "7 3, " #ℎ 6 6B 7:
. E7 1 F " G. 7E 1 F
SOLUTION:
4. @ " ∘ 7 6 #ℎ !# 6 1 "7 √2
SOLUTION:

EXERCISES – Evaluate the following functions

1. 2 3 8 "7 5 6 , " ∘7 . . 50 135 98

1 1
2. 1 "7 @ " 7∘ 2 .
1 4

3. 2 , " 7 7 8 . 75

4. "7 3 , " ! " 7 ! " . ! 2!" " 3! 3"

5. ,7 2, " ∘7 2 . 16

6. ℎ 2 4 ,H 3 1, " ℎH 5 . 84

7. 5 6 ,7 3 4 8, " 7 .3 9 2

8. 4 "7 2 , " : ; 3 . 21
7

9. 3 7 4, " ∘ . 27 126 I
54 217 72

10. 15 1J
"7 3 K
, " : ; .5 L
7

11. 3, " ∘ ∘ . 9

12. 6 "7 2 1 , " ∘7 . 4 4 7

2 ℎ 2
13. 3 , " . 3

1 1
14. 7 , " 7 . 10 24
4 6

15 . 2 3 ,7 2 1 , " : ; 0 . 2
7

LINEAR, QUADRATIC , and POLYNOMIAL FUNCTIONS

Note: The further information about the lines and parabolas, kindly refer to Analytic Geometry Subject.

A linear equation has the form Ax + By + C = 0, where A and B are not both zero. Its graph is a straight line. The solutions of a linear equation are the ordered
pairs for the points on its graph. An ordered pair corresponds to a point in the coordinate plane. Since two points determine a line, only two points are needed to graph a linear
equation. Often the two points that are easiest to find are the x-intercept and the y-intercept. The x-intercept is the point where the line crosses the x-axis, and the y-intercept
is the point where the graph crosses the y-axis.

Slope of the line


1
5 1 (
1

Note: See Differential Calulus for the information of the slope of the other curves.
Slope – Intercept Form:

5 G

For information about midpoints and other elements, see Analytic Geometry Subject as well as Translation of axis.

Zeros of the Linear Function - Usually an x-intercept.

QUADRATIC FUNCTIONS

The graph of the quadratic function y = ax^2 + bx + c , a ≠ 0. If a ≥ 1 , the parabola opens upward otherwise opens downwards. When the parabola opens upwards, the
lowest point is known as the vertex, while the highest point is the vertex if the parabola opens downwards.

For detailed information about the parabolas, see analytic geometry . Kindy refer to the previous topic quadratic equations about the zeros of the quadratic function.

The graph of

ℎ H , (0

is a parabola whose vertex is the point (h,k) . The parabola opens upward for a ≥ 1 other wise downward except a = 0.
Note: Finding the x – intercept of the quadratic functions is same process of solving quadratic equations . Find the y- intercept is setting x = 0 .

MAXIMUM AND MINIMUM VALUES OF THE QUADRATIC FUNCTIONS

For the detailed information about the maximum and minimum values of the other functions, see Differential Calculus under the topic Maxima and Minima.

POLYNOMIAL FUNCTIONS

If n is a non negative integer or natural numbers,a function that can be written in the form

, .
.
/ .01
.01
/ ⋯/ 1 3 , . (0

is called a polynomial function of degree n. The numbers that involves variable a are the coefficients of P(x)

ZEROS OR ROOTS – P(x) = 0 , can be solved by synthetic division or substituting values of x . The most n zeros is also the degree n.

DIVISION ALGORITHM - For each polynomial P(x) of degree greater than 0 and each number r, there exists a unique polynomial Q(x) of degree 1 less than P(x) and a unique
number R such that

, 9

The polynomial Q(x) is called the quotient, x - r is the divisor, and R is the remainder. Note that R may be 0.

REMAINDER THEOREM – If R is the remainder after dividing the polynomial P(x) by x – r , then P(r) = R.
FACTOR THEOREM – If r is a zero of the polynomial P(x) , then , x – r is a factor of P(x) . Conversely, if x – r is a factor of P(x) , the r is a zero of P(x)

RATIONAL ROOT THEOREM – If the polynomial

, .
.
/ .01
.01
/ ⋯/ 1 3 , . (0

has any rational roots , then they must be in the form

!#6 6 3
/
!#6 6 .
DESCARTES’ RULE OF SIGNS – determines the real zeros of the polynomial functions. It tells us that the number of positive real zeroes in a polynomial function f(x) is the
same or less than by an even numbers as the number of changes in the sign of the coefficients. The number of negative real zeroes of the f(x) is the same as the number of
changes in sign of the coefficients of the terms of f(-x) or less than this by an even number.

UPPER BOUND - If you divide a polynomial function f(x) by (x - c), where c > 0, using synthetic division and this yields all positive numbers, then c is an upper bound to the
real roots of the equation f(x) = 0. Note that two things must occur for c to be an upper bound. One is c > 0 or positive. The other is that all the coefficients of the quotient as
well as the remainder are positive.

LOWER BOUND – . If you divide a polynomial function f(x) by (x - c), where c < 0, using synthetic division and this yields alternating signs, then c is a lower bound to the real
roots of the equation f(x) = 0. Special note that zeros can be either positive or negative. Note that two things must occur for c to be a lower bound. One is c < 0 or negative.
The other is that successive coefficients of the quotient and the remainder have alternating signs.

INTERMEDIATE VALUE THEOREM - If f(x) is a polynomial function and f(a) and f(b) have different signs, then there is at least one value, c, between a and b such that f(c) =
0.

TURNING POINT – A point on a continuous graph that separates an increasing portion from a decreasing portion or vice versa.

For other graphing elements and higher fundamentals, see Differential Calculus.

EXAMPLES
1. Determine the zero of y = 1/2x + 2 .
SOLUTION:

2. Find the y-intercept of the graph of f(x) = 1/2 x + 3/7 .

3. Determine the appropriate vertex form of the quadratic function y = 3x^2 +15x +32 .
SOLUTION:

4. Determine the vertex of the quadratic function f(x) = -2x^2 + 4x + 5 .


SOLUTION:

5. Compute the factors of the polynomial function:

, I
6 30

SOLUTION:
6. Find the upper bound and lower bound of the function :
I
3 5 10

SOLUTION:

7. Given that one root x^3 + 2x^2 -23x – 60 = 0 is 5 , solve the equation.
SOLUTION:

EXERCISES – Solve the following questions.

1. Find the vertex of the quadratic function f(x) = (x + 3)^ 2 +1 Ans. ( - 3 , 1)


2. Find the y-intercept of the quadratic function f(x) = -4(x + 3)^2 +7 Ans. – 29
3. Consider the linear function x + 4y = -4 , find the x and y intercepts. Ans. x – intercept = -4 , y-intercept = -1
4. For the quadratic function y = x^2 +4x , determine the minimum value. Ans. – 4 .
5. Determine the real roots of the function f(x) = x^5 + 5x^4 – 2x^3 – 28x^2 – 8x + 32. Ans. x = ± 2 , -4
6. Find the upper bound and the lower bound of the polynomial function 9x^4 – 9x^3 – 58x^2 + 4x + 24 Ans. lower bound = - 3 , upper bound = 4
7. Write a linear function that have slope of 1/2 and passes through (6, 4) Ans. y = 1/2 x + 1
8. Determine the remainder of P(x) = 3x^3 – 2x^2 + x – 5 . Ans. -39
9. If P(x) is divided by 3x – 4 , the quotient is x^2 - x + 2 and the remainder is 7. Find P(x) Ans. P(x) = 3x^2 + x^2 + 2x +15
10. Given that the expression px^3 – qx + 4 leaves a remainder of 10 and -2 when divided by (x + 1) and (x – 2 ) respectively, find the values of p and q.
Ans. p =1 , q = 7
11. Rewrite in standard form:

1
8 12
2

. 8 20
2

12. Find the zeros of the linear function y = 2x – 4. Ans. x = 2.


13. Determine the x – intercept of the function f(x) = x2 +3x Ans. x = [0 . -3]
14. Determine the positive real roots of P(x) = x^3 – 12x -16 Ans. 1
15. Find the maximum value of the function f(x) = -2(x – 1)2 - 3. Ans. – 3
16. Determine the remainder of the function P(x) = 2x3 – 3x2 – 8x - 3 when divided by x – 3 . Ans. 0
17. Find the slope of the this linear function:

2
4
3

Ans. 2/3

18. Find the vertex of the quadratic function f(x) = 2x2 – 6x +7 . Ans. (3/2 , 5/2)
19. Find the rational roots of this polynomial equation:
J
2 M
18 I
8 41 30 0

Ans. 2,3,-1 and 5


20. Find the real zeros of this function given the interval [1,4] :
I
5 3 6

Ans. x = 2

INVERSE FUNCTIONS

If y is a function of x, the graph of y against x can be used to find x when any value of y is given. Thus the graph also expresses that x is a function of y. Two such functions are
called inverse functions.

In general, given a function y = f(x) , its inverse may obtained by interchanging the variables the transposing for the designated variable. The inverse function is denoted by y
= f-1(x) .

EXAMPLES:

1. 2 1
SOLUTION:

2. √ 1
SOLUTION:

2
3.
5

SOLUTION:
EXERCISES -Determine the inverse of the following functions.

1. 2 I
. 01

3
2 6. 4 . 01
√ 4

2 1 1
2. 3 2 . 01
7. . 01
2
3 2

3. I
6 . 01
√ 6 8. 1, %0 . 01
/√ 1
3

5 1 1 3
4. 3 5 . 01
9. . 01
3 2 3 2 1
1
5. 2 .
N
I 01
√ 2 10. 3 1 . 01
3

APPLICATIONS OF FUNCTIONS

The slope of the function is the same as the rate of change for the dependent variable . More detailed topics on the rate of change will be discussed further in Calculus.
Mathematical model describes a system using mathematical concepts and language.

EXAMPLES:
1. Initially, trains A and B are 325 miles away from each other. Train A is travelling towards B at 50 mph and train B is traveling towards A at 80 miles per hour. At
what time will the two trains meet? At this time how far did the trains travel?
SOLUTION:
The two lines are y = 50x and y = -80x +325 , equating two terms gives you

50 80 325 , 6 6 7 2.5 ℎ . → # 5 B 5 #

For the distance, y = 50(2.5) = 125 miles. Ans.

2. A rock is thrown upward from the ground. Its height in feet above ground after t seconds is given by the function f(x) = -16t^2 +20t. Find the maximum height of the
rock and the number of seconds it took for the rock to reach its maximum height.
SOLUTION:

Note: Geometric related applications wil be discussed further in Solid Mensuration.

EXERCISES – Solve the following questions.

1. The polynomial function P(x) = 45x - 100,000 models the relationship between the number of computer briefcases x that a company sells and the profit the
company makes, P(x) . Find (4000), the profit from selling 4000 computer briefcases. Ans. 80,000
2. A meteorologist sends a temperature probe on a small weather rocket through a cloud layer. The launch pad for the rocket is 2 feet off the ground. The height of
the rocket after launching is modeled by the equation h = - 16t^2 + 232t +2 , where h is the height of the rocket in feet and t is the elapsed time in seconds. When
will be the rocket 114 ft above the ground? Ans. 0.5 and 14 seconds.
3. The profit P (in $ 1M) for a Blu Ray Manufacturer can be modeled by P = -6x^3 +72x , where x is the no. of Blu Rays produced per million. For a Profit of $ 96M ,
how many Blu Rays can be produced? Ans. 2 million
4. If f(x) is a linear function and (3, - 2) and (8, 1) are the points of the line, is this function increasing or decreasing? Ans. Increasing
5. A person standing near the edge of a cliff 100 feet above a lake throws a rock upward with an initial speed of 32 feet per second. The height of the rock above the
lake at the bottom of the cliff is a function of time and is described by

ℎ # 16# 32# 100

How many seconds will take until the rock reaches its maximum height? Ans. 1 second
6. A phone company charges $0.39 per minute for the first 10 mins. of an international call ans $0.12 per minute after that. Find the cost function C(x) as a function
of the length of the phone call in x minutes. Ans. C(x) = 0.39x + 0.12(x – 10) if x > 10 , C(x) = 0.39x for x ≤ 10.
INEQUALITIES

For two real numbers a and b, we say that a is less than b , and write a < b , if there is a positive real number p so that a + p = b . The statement b > a , read b is greater that
a , means exactly same as a < b .

When we write a ≤ b we mean a < b or a = b and say a is less than or equal to b. When we write a ≥ b we mean a > b or a = b and say a is greater than or equal to b.

INEQUALITY PROPERTIES

EXAMPLES:

2 3 4
1. 6%2
4 3
SOLUTION:

2. % 0 , % 0 , 2 +4
SOLUTION:

3. 6 3&0
SOLUTION:
4. 2&3 4 + 16
SOLUTION:

5. In order to get a bonus this month, Leon must sell at least 120 newspaper subscriptions. He sold 85 subscriptions in the first three weeks of the month. How many
subscriptions must Leon sell in the last week of the month?
SOLUTION:

EXERCISES – Solve the following inequalities . (11 to 15 are applications)

1. 2 3 5 +8 1 . %2 6. 2 3 3 1 6 . &3

1 1 7 3 10
2. 2% : 9; 1 . +4 7. 2 . &2, 4
2 2 4 4

3. 2 5& 36 6 &0 . 16 &0 8. P 2 8 2 . & 2

5 3
4. 4 % 14 . + 3√2 26 % 3√2 2 9. 2 3 4 5 +0 . + +
4 2

5. 3 6 6 &4 1 . 1 10. %0 . + 16 % 0

11. The total profit function P(x) for a company producing x thousand units is given by:

, 2 26 44

Find the values for x which the company makes a profit. Ans. x = between 2 and 11

12. The elevator in Yehire’s apartment building has a sign that says the maximum weight is 2,100 pounds. If the average weight of one person is 150 pounds, how
many people can safely ride the elevator? Ans. 14
13. An Internet provider charges P30 per day for 1000 MB of mobile data plus P 0.08 for each additional MB. In a week of month of data consumption, his spendings
ranged from P 36.40 to P 47.20. What were the most and least MB used? Ans. range from 1080 to 1215 MB
14. A stuntman will jump off a 20 m building. A high-speed camera is ready to film him between 15 m and 10 m above the ground. When should the camera film him?
Ans. Around 1 to 1.4 seconds
15. A company manufactures cassettes. Its cost and revenue functions are C(x) = 26,000 + 30x and R(x) = 43x, respectively, where x is the number of cassettes
produced and sold in a week. How many cassettes must be sold by the company to realise some profit? Ans. 2,000 cassettes

NEXT TOPIC ON SEPTEMBER 14 , 2020:

1. Exponential and Logarithmic Functions and Equations


2. Ratio and Proportion and Variation
REFERENCES

https://sciencing.com/ways-tell-something-function-8602995.html
Algebra and Trigonometry by Cynthia Young
https://byjus.com/maths/one-to-one-function/
Glencoe’s Advanced Mathematical Concepts
https://www.dublin.k12.ca.us/cms/lib/CA01001424/Centricity/Domain/172/Function%20worksheet.pdf
Schaum’s Outlines of Calculus
Schaum’s Outlines of Algebra
https://www.chilimath.com/lessons/intermediate-algebra/relations-and-functions/
College Algebra with Trigonometry by Barnett
Introductory and Intermediate Algebra by Blitzer
http://www.montereyinstitute.org/courses/DevelopmentalMath/COURSE_TEXT2_RESOURCE/U17_L2_T3_text_final.html
https://www.leonschools.net/cms/lib7/FL01903265/Centricity/Domain/4838/Ch%202.1%20%20Functions%20Domain%20and%20Range%20Review%20WKSH%20PDF.pdf
PreCalculus by OpenStax
https://www.emathzone.com/tutorials/calculus/types-of-functions.html
https://brilliant.org/wiki/floor-function/#:~:text=The%20floor%20function%20(also%20known,than%20or%20equal%20to%20x.
https://www.chilimath.com/lessons/introductory-algebra/evaluating-a-function/
https://www.cliffsnotes.com/study-guides/algebra/algebra-ii/relations-and-functions/compositions-of-functions
https://nool.ontariotechu.ca/mathematics/functions/composition-of-functions.php
https://www.mathworksheets4kids.com/function/evaluate/quadratic-functions-easy-1.pdf
Beginning Algebra by Miller
Algebra for Dummies
http://www.montereyinstitute.org/courses/DevelopmentalMath/COURSE_TEXT2_RESOURCE/U17_L3_T1_text_final.html
https://www.chilimath.com/lessons/advanced-algebra/composition-of-function/
https://www.onlinemathlearning.com/composite-functions.html
Algebra and Trigonometry by OpenStax
Schaum’s Outlines of College Mathematics
https://www.mathplanet.com/education/algebra-2/polynomial-functions/descartes-rule-of-
sign#:~:text=Descartes'%20rule%20of%20sign%20is,the%20sign%20of%20the%20coefficients.
https://www.varsitytutors.com/precalculus-help/express-a-quadratic-function-in-vertex-form
Precalculus by Cynthia Young
https://courses.lumenlearning.com/ivytech-collegealgebra/chapter/use-the-factor-theorem-to-solve-a-polynomial-equation/
UPCAT Reviewers
https://www.shsu.edu/~kws006/Precalculus/2.1_Quadratic_Functions_files/WS_Soln_2_1A_QuadraticFunctions.pdf
Algebra Essentials For Dummies
https://www.cbsd.org/cms/lib/PA01916442/Centricity/Domain/2746/PC%20-%20Section%202.6%20-%20Upper%20Lower%20Bounds%20-%20NOTES.pdf
Algebra the Easy Way by Barron’s
Engineering Mathematics by Bird
http://www.mash.dept.shef.ac.uk/Resources/A26remainder.pdf
https://www.storyofmathematics.com/inverse-functions
http://dl.uncw.edu/digilib/Mathematics/Algebra/mat111hb/functions/inverse/inverse.html
Higher Engineering Mathematics by Bird
https://mathinsight.org/inverse_function_examples
https://math.libretexts.org/Courses/Borough_of_Manhattan_Community_College/MAT_206_Precalculus/0%3A_Review_-
_Linear_Equations_in_2_Variables/0.1%3A_Linear_Functions
https://students.ga.desire2learn.com/d2l/lor/viewer/viewFile.d2lfile/1798/12938/Algebra_ReasoningwithEquationsandInequalities7.html
https://www.mathsisfun.com/algebra/inequality-rational-solving.html
https://www.chilimath.com/wp-content/uploads/2017/02/worksheets-solving-rational-inequalities-version-2.pdf
https://math.libretexts.org/Bookshelves/Algebra/Book%3A_Elementary_Algebra_(OpenStax)/03%3A_Math_Models/3.06%3A_Solve_Applications_with_Linear_Inequalities
CIVIL ENGINEERING BOARD EXAMS PROBLEMS PHILIPPINES , SEPTEMBER 14, 2020

EXPONENTIAL AND LOGARITHMIC FUNCTIONS AND EQUATIONS


Note: Laws of Exponents are discussed in the Module 1.

EXPONENTIAL FUNCTION
A function of the form

is called exponential function if b > 0 ,b is not equal to 1 and x is a real number.

LOGARITHMIC FUNCTION

If b > 0 and b ≠ 1 , then

log

for every x > 0 and every real number y.

LOGARTIHMS PROPERTIES

If b is a real number, b > 0 , and b ≠ 1, then,

log 1 0 1

log 2

log log log 4

log log log 5

log "
# log 6

COMMON OR BRIGGSIAN LOGARITHMS – logarithms with base 10. Denoted as “log x”


NATURAL OR NAPERIAN LOGARITHMS – logarithms with base e (e = 2.71828) , denoted as “ln x” . e refers to the Euler’s number.

CHANGE OF BASE
If a,b, and c are positive real numbers and neither b nor c is 1 , then

log %
log
log %

EXAMPLES:

Determine the value of 3^x ranging from 1 to 4.


SOLUTION:

f(x) = 3^x = f(1) = 3^1 = 3


f(2) = 3^2 = 9 , f(3) = 3^3 = 27 , f(4) ^ 3^4 = 81

Solve for 8x = 2y + 2 and 163x – y = 4y


SOLUTION:
Evaluate this logarithm:

log & 8

SOLUTION:

Evaluate this logarithm:

log 5 2 (

SOLUTION:

Evaluate this natural logarithm :


ln
SOLUTION:

Evaluate log5 140.


SOLUTION:

Express as single logarithm :

3 ln 7 ln

SOLUTION:

Simplify this logarithm : log 64 – log 128 + log 32


SOLUTION:
EXERCISES – Evaluate the following expressions.
,
1. log 2 log log , - . log .

2. 3 ln /&
- . 1

(
√ . 1 1
3. log ( - . 3 log ( log ( . 1 7 log ( 2
2 1 2

4. 2 223 - . 18

1 1
log 16 log 8 1
5. 2 3 - .
log 4 2

7 (
6. 3 ln 7 ln - . ln 3 4

7. log . 56.25 - . 5.814

8. ln /1
- . 7

9. 3 2 ⋅ 3( 1- . 39,365

10. - .7
81 9 .

11. log . - . log 2 log

12. log ( 7 - . 1.771244

3
13 . 4 3 4 /
- .
256

14. : 0 5 ;/
4 - . 9.5914
( 2 1
15. log 3 4 - . 3 log log 2 1 4 log 2 <# - . 3 log log 2 1 2 log 4
16

EXPONENTIAL AND LOGARITHMIC EQUATIONS

STEPS ON SOLVING EXPONENTIAL EQUATIONS

EXAMPLES
1. 3 . 2 7
SOLUTION:
2. 4. /; 64
SOLUTION:
4. /; 64
4. /; 4(
2x – 1 = 3
2x = 4 , x = 2 Ans.

3. √729 √9
=>?.@ =A?.@

SOLUTION:

LOGARITHMIC EQUATIONS
STEPS ON SOLVING LOGARITHMIC EQUATIONS:

EXAMPLES

1. log 3 log log 4


SOLUTION:
2. log 3 log 27
SOLUTION:

3. log & 2 2
SOLUTION:

EXERCISES – Solve the following equations

1. 5( 16 - . B 0.5742 9. 2 (
3 15 - . B 0.73241

2. .
3 2 0 - . 0 C ln 2 10. 3./( 4. D;
- . B 0.13

3. ln .
16 ln 4 32 - . E< <FGHI< 11. ln .
9 2 - . JK 9 .

1
4. log . log . 1 - . 4 12. ⋅ - . M6,8N
9 ;&
&L

5. 2 log ( log O 10 - . 81 13. 5 D; 0.1 - . B 2.431

6. log P 3 .
1.96 - . B J2.80 14. log ( .
4 - . J9

7. 0.027 3.26 - . B 0.32717 15. P /;


20 - . B 7.99

1
8. ln 3 ln 4 1 ln 1 - .
2

APPLICATIONS OF LOGARITHMIC AND EXPONENTIAL FUNCTIONS

1. EXPONENTIAL GROWTH AND DECAY – given by the this function

H -Q JRS

Negative values – decay


Positive values – growth

Examples: Population growth, substance decay ,

2. Compound interest
# WS
- T U1 V

Where
A = sum of money , P = principal , t = years , r = annual percentage rate r , n = compounded n times a year.
For more information about this, see Engineering Economics. This formula may be used or not in this topic.

3. Miscellaneous – earthquakes , pH levels of a substance , others.


EXAMPLES:
1. The formula A = 22.9e^0.0183t models the city covid cases, A in thousands , t days after latest count. When will total cases reach 27,000?
SOLUTION:

- 22.9 X.X;L(S
, 27 22.9 X.X;L(S
;

27 27
X.X;L(S
; ln 0.0183H ; H 9C . - .
22.9 22.9
.

2. How long will it take for 10% of a 1000 g sample of uranium-235 to decay? The uranium-235 has a decay rate of 703.8 million years.
SOLUTION

3. Find an earthquake’s magnitude on the Richter scale if a recording station measures an amplitude of 300 micrometers and 2.5 seconds between
waves. Assume that B is 4.2. The magnitude of an earthquake is calculated by

Z log U V \
[

T – time in seconds and a – length of the amplitude.


SOLUTION:

EXERCISES – Answer the following word problems.

1. The magnitude R of the earthquake intensity is given by

]
Z log
]Q

R – magnitude , I – actual intensity and I_o is the base intensity.


Masbate records magnitude 6.6 earthquake and it was 10^2.7 stronger than in Batanes. Determine the earthquake strength in Batanes. Ans.
Magnitude 3.9
2. It is estimated that “t” years from now , the population of a certain place will be

20
T H IFFI< .
2 3 /X.X^S

What will be the population after 50 years? Ans. 9.31 million


3. After 5 years of interest payments of 5.5% compounded quarterly, an account has P 5046.02. What was the principal ? Ans. P 3840.00
4. The hydrogen ion concentration of a substance is related to its acidity and basicity. Because hydrogen ion concentrations vary over a very wide
range, logarithms are used to create a compressed pH scale, which is defined as follows:

_` logM` D N

Where [H^+] is the hydrogen ion concentration , in moles per liter. Pure water has pH of 7 (neutral) while acidic substances have the pH level of
below 7. Basic substances have the pH level of greater than 7. Find the classification of seawater with hydrogen ion concentration of 4.63(10^-9) .
Ans. Basic
5. Average national teachers’ salaries can be modeled using the equation y = 9.25(1.06)^n , where y is the salary in thousands of pesos per year and
n is in number of years starting 2000. How much is the salary of the year 2020 ? Ans. P 29,666
6. Refer to (1) . The earthquake happened in Bohol was 7.2 while in Japan is 9.1. How many times that the Japan has stronger compared to Bohol?
Hint: Use I_o = 1 as base. Ans. 79 times
7. A form of a nickel has a half life of 96 years. How much of a 30 gram sample is left after 250 years? Ans. 4.933 grams.
8. In 2004, Kazakhstan’s population is 15 million. How many people will be 20 years from now if the population doubles every 30 years? Ans. 23.8110
million
9. The half-life of caffeine in your body is about 6 hours. If you had 1 cup of coffee 9 hours ago how much is left in your system? Ans. 0.35
10. The function

90
1 270 /X.;..

Where f(x) – persons with high risk of coronavirus. And x – is in ages.


Determine f(70) Ans. 86 persons have high risk of coronavirus.

RATIO, PROPORTION and VARIATION


RATIO – comparison by division of two quantities in the congruent units.
PROPORTION – statement of two ratios are equal.

Inner values of the proportion are MEANS while outer values are known as EXTREMES.

DIRECT PROPORTION – when a given quantity is changed in some ratio and another quantity is always changed in the same ratio or by same factor.
INVERSE PROPORTION – when a given quantity in some ratio and another quantity is changed in the inverse ratio.

EXAMPLES

1. A man who is 6 feet tall casts a shadow 5 feet long. If a building at the same time has a shadow 225 feet long, how high is the building?
SOLUTION:

2. If the ratio of girls to boys in Math Club is 5:3 , and there are 32 members of the club, how many boys are members?
SOLUTION:

5 32
, 5 3 32 , 12 # # < . - .
3

3. 4 people can paint a fence in 3 hours. How long will it take 6 people to paint it? (Assume everyone works at the same rate)
SOLUTION:
EXERCISES – Solve the following questions.

1. Find the mean proportion between 4 and 36. Ans. 12


2. 6 men, working 8 hours a day, earn $ 8400 per week. What will be the earning per week of 9 men who work for 6 hours a day? Ans. $ 9450
3. Bleaching powder is obtained through the reaction of chlorine and slaked lime, 74.10 kg of lime and 70.91 kg of chlorine producing 127.00 kg of
bleaching powder and 18.01 kg of water. How many kg of lime will be required to produce 1000 kg of bleaching powder? Ans. 583.46 kg
4. It takes 4 men 6 hours to repair a road. How long will it take 8 men to do the job if they work at the same rate? Ans. 3 hours
5. If 1 person takes 7 days to pick the pears from a tree, how many days will it take 7 people to do the same job? Ans. 1 day
6. Out of the total students in a class, if the number of boys is 5 and the number of girls being 3, then find the ratio between girls and boys. Ans. 3:5
7. Solve for x : x:4 = 6:8 Ans. x =3
8. In a bag of red and green sweets, the ratio of red sweets to green sweets is 3:4. If the bag contains 120 green sweets, how many red sweets are
there? Ans. 90
9. Four pipes can fill a tank in 70 minutes. How long will it take to fill the tank with 7 pipes? Ans. 40 mins.
10. Pediatricians prescribe 5 milliliters (ml) of acetaminophen for every 25 pounds of a child’s weight. How many milliliters of acetaminophen will the
doctor prescribe for Emilia, who weighs 60 pounds? Ans. 12 mL.

VARIATION

Variation – a mathematical function that relates the values of one variable to those of other values.

DIRECT VARIATION – y varies directly as x or y is directly proportional to x, if there is a nonzero constant k such that y = kx . k stands for constant of
variation.

INVERSE VARIATION - y varies inversely as x or y is inversely proportional to x, if there is a nonzero constant k such that y = k/x .

JOINT VARIATION - If the ratio of a variable y to the product of two or more variables is constant, then y varies jointly as, or is jointly proportional to, the
other variables. If y = kxz. It can be direct and inverse proportional for at least two variables.

Note: For geometric problems, see Solid Mensuration.

EXAMPLES:

1. Suppose that y varies directly as x. If y is 5 when x is 30, find the constant of variation and the direct variation equation.
SOLUTION:

2. The electrical resistance R of a piece of wire is inversely proportional to the crosssectional area A. When A=5 mm^2, R =7.02 ohms. Determine
the cross sectional area when the resistance is 4 ohms.
SOLUTION:

3. Z varies directly as x and varies inversely as the square of y. If z = 7/2 when x = 14 and y = 6, find z when x = 27 and y = 9.
SOLUTION:
4. PARTIAL VARIATION - has an initial value that is not zero and a constant rate of change. It can be solved using systems of equations. Y is partly
constant and partly varies directly with x. When y = 4, x = 2, and when y = 16, x = 4. Find y when x = 6.
SOLUTION:

5. The frequency f of a vibrating guitar string is directly proportional to the square root of the tension T and inversely proportional to the length L.
What is the effect on the frequency if the length is doubled and the tension is quadrupled?
SOLUTION:

Remains the same.

EXERCISES – Solve the following problems indicated.


1. The rate of vibration of a string under constant tension varies inversely with the length of the string. If a string is 24 inches long and vibrates 128
times per second, what is the length of a string that vibrates 64 times per second? Ans. 48 inches
2. Z varies jointly with x and y . If x =3 , y = 8, and z = 6, find z when x = -2 and y = 10. Ans. z = – 5
3. A lumber company needs to estimate the volume of wood a load of timber will produce. The supervisor knows that the volume of wood in a tree
varies jointly as the height h and the square of the tree’s girth g. The supervisor observes that a tree 40 meters tall with a girth of 1.5 meters produces
288 cubic meters of wood. What volume of wood can the supervisor expect to obtain from 50 trees averaging 75 meters in height and 2 meters in
girth? Ans. 48,000 m^3
4. The expense of a boarding house is partly fixed and partly vary with the number of boarders. Each boarder pays $ 390 a month. The profits are $
54 per head per month when there are 50 boarders and $ 64 when there are 60 boarders. What is the profit per head per month when there are 80
boarders? Ans. $ 76.50
5. (CE Board) Given that w varies directly as the product of x and y and inversely as the square of z and that w = 4 when x = 2 , y = 6 and z = 3. Find
w when x = 1, y =4 and z =2. Ans. w = 3
6. (EE Board) A storage battery discharges at a rate which is proportional to the charge. If the charge is reduced by 50% of its original value at the
end of 2 days, how long will it take to reduce the charge to 25% of its original charge? Ans. 3 days
7. y varies directly with x. If y = 15 when x = -18, find y when x = 1.6. Ans. -4/3
8. The distance that a spring will stretch varies directly as the force applied to the spring. A force of 12 pounds is needed to stretch a spring 9 inches.
What force is required to stretch the spring 15 inches? Ans. 20 pounds
9. If x varies inverse as y, and x = 7 when y = 3, find y when x = 9. Ans. 7/3
10. If the horsepower P required to drive a speedboat through water is directly proportional to the cube of the speed v of the boat, what change in
horsepower is required to double the speed of the boat? Ans. Eight times horse power.
11. The quantity y varies directly with the cube of x. If y = 25 when x = 2, find y when x is 6. Ans. 675
12. The volume of gas in a container at a constant temperature varies inversely as the pressure. If the volume is 32 cubic centimeters at a pressure of
8 pounds, find the pressure when the volume is 60 cubic centimeters. Ans. 32/15 pounds
13. A is in joint variation with B and square of C. When A = 144, B = 4 and C = 3. Then what is the value of A when B = 6 and C = 4? Ans. A = 24
14. If y varies indirectly as x and the constant of variation is 2, find y when x is 6. Ans. Y = 1/3
15. X is partly constant and partly varies as y . when y =2,x=15 and y =4, x=27 , find x when y=7. Ans. x = 45

NEXT TOPIC ON SEPTEMBER 18,2020

1. Sequences and Series


2. Mathematical Induction
REFERENCES:

Intermediate Algebra by Elayn Matin


Engineering Mathematics by Besavilla
College Algebra with Trigonometry by Barnett
Higher Engineering Mathematics by Bird
Introductory and Intermediate Algebra by Blitzer
Engineering Mathematics by Bird
Scahum’s Outlines of College Mathematics
Algebra Essentials for Dummies
https://www.siyavula.com/read/maths/grade-11/functions/05-functions-05
Precalculus by Blitzer
https://math.colorado.edu/math1300/resources/Exercises_LogarithmicFunction.pdf
College Algebra By Barnett
Algebra and Trigonometry by Cynthia Young
http://www.sosmath.com/algebra/logs/log4/log43/log43.html
1001 Practice Problems in Precalculus
https://www.chilimath.com/lessons/advanced-algebra/solving-logarithmic-equations/
Cliff’s Quick Review Precalculus
https://www.analyzemath.com/high_school_math/grade_12/how_to_solve_exponential_equations.html
Essentials Engineering Mathematics by Alan Jeffrey
Fundamental Engineering Mathematics Workbook by Challis
https://courses.lumenlearning.com/waymakercollegealgebra/chapter/exponential-growth-and-decay/
https://www.softschools.com/math/algebra/topics/exponential_equations_compound_interest_application/
Engineering Mathematics Vol. 1 by Besavilla
Algebra and Trigonometry by Openstax
Advanced Mathematical Concepts by OpenStax
https://www.mathsisfun.com/algebra/exponential-growth.html
Schaum’s Outlines of Basic Mathematics
Idiot’s Guide to Basic Mathematics and Pre Algebra
https://www.math-only-math.com/worksheet-on-inverse-variation.html
http://www.mathster.com/worksheets/KS3%20&%20KS4%20Free%20Maths%20Worksheets/Inverse%20Proportion.pdf
https://byjus.com/maths/ratios-and-proportion/
Schaum’s Outlines of Elementary Algebra
https://www.onlinemathlearning.com/ratio-problems.html
https://www.emathzone.com/tutorials/everyday-math/inverse-proportion.html
Pre Algebra By OpenStax
Engineering Mathematics Vol. 1 by Gillesania
https://www.shelovesmath.com/algebra/beginning-algebra/direct-inverse-and-joint-variation/
https://www.math-only-math.com/problems-on-variation.html
https://algebraelisara.weebly.com/uploads/5/8/7/3/58736993/direct_inverse_variation_words_graphs_ws.pdf
https://www.mesacc.edu/~scotz47781/mat120/notes/variation/inverse/inverse.html
PreCalculus by OpenStax
https://www.jiskha.com/questions/1512656/x-is-partly-constant-and-partly-varies-as-y-when-y-2-x-15-and-y-4-x-27-a-find-the
CIVIL ENGINEERING BOARD EXAMS PROBLEMS PHILIPPINES – SEPTEMBER 18, 2020

SEQUENCES AND SERIES

SEQUENCE OR PROGRESSION – sets of numbers in a specific order that can be used for all real numbers.

SERIES – the combination of the sets of numbers in a progression or sequence.

Series are represented in a compact called summation notation using the capital letter Sigma Σ .

EXAMPLES:

1. Evaluate this term:

SOLUTION:

2. Evaluate this term:

SOLUTION:
For n = 1: 1 – 1 + 2 = 2
For n = 2: 2^2 – 2 + 2 = 4 – 2 + 2 = 4
For n = 3: 3^2 – 3 + 2 = 9 – 3 + 2 = 8
Adding the terms: 2 + 4 +8 = 14 Ans.

ARITHMETIC PROGRESSION AND SERIES

2 1
2 2

an = nth term of the artimetic progression


Sn = sum of the terms in an arithmetic progression
a1 = first term n = no of terms d = common difference

GEOMETRIC PROGRESSION AND SERIES

1
, 1
1

INFINITE GEOMETRIC SERIES

1
r – common ratio

EXAMPLES

1. Find the 10th term of the arithmetic progression 1, 3.5, 6, 8.5,...


SOLUTION:

2. How many terms must be added in an arithmetic sequence whose first term is 11 and whose common difference is 3 to obtain a sum of 1092?
SOLUTION:

2 1 ; 1092 2 11 1 3
2 2

Using quadratic equations:

22 3 3
1092 ; 2184 19 3 ; 3 19 2184 0
2

19 # $ 19 4 3 2184
, 24 % &.
2

Note: -91/3 is not accepted because n should be integers.

3. Find the eighth term of the sequence 1,3,9 …


SOLUTION:

, 3

1 3( 2187 % &.

4. Find the tenth partial sum of the geometric sequence 24,12,6, …


SOLUTION:

5. At the beginning of Claudia Schaffer’s exercise program, she rides 15 minutes on the Lifecycle. Each week, she increases her riding time by 5 minutes.
Find her riding time after 7 weeks.
SOLUTION

15 , 5, 7

Then,

1 15 7 1 5 45 +, -./&. % &.

6. The sides of a square, l, have lines drawn between them connecting adjoining sides with their midpoints. This creates another square within the original
and this process is continued indefinitely. Calculate the sum of the areas of the infinite squares.
SOLUTION:
EXERCISES – Solve the following problems. (13 to 20 are applications)
1. Find the ninth term of the arithmetic progression whose first three terms are 3,9,and 15. Ans. 51
2. The second term of a geometric progression is 8 and the sixth term is 128. Find the 11th term. Ans. 4096
3. (ECE Board) Determine the sum of the infinite series : S = 1/3 , 1/9 , 1/27 … Ans. 1/2
4. Find the sum of the multiples of 3 between 28 and 112. Ans. 1974
5. If a_n = -5 , 10 , -20 , 40 and b_n = 10 , -5 , -20 -35, … Find a_10 + b_10 Ans. 2435
6. Find the explicit equation of the terms 0.9,0.09,0.009, etc… Ans. a_n = (9/10)(1/10)^n-1
7. Find the sum of the positive terms of the arithmetic sequence 85, 78, 71, … 1 Ans. 559
8. Determine the number of terms of the series 5,8,11… of which the sum is 1025. Ans. 25 terms
9. Find the first three terms of the geometric progression whose first term is 3 and a common ratio of 3. Ans. 6,18,54
10. Determine the sum of this series:

05 $4 1 2
1 3
Ans. 35
11. Find the summation :

, 3
4
1 3

Ans. -1/2
12. Find the sixth term of the sequence r – 1 , -3r +3 , 9r – 9 … Ans. -243r +243
13. An auditorium has 20 seats on the first row, 24 seats on the second row, 28 seats on the third row, and so on and has 30 rows of seats. How many
seats are in the theatre? Ans. 2340 seats
14. In 2013, the number of students in a small school is 284. It is estimated that the student population will increase by 4% each year. Estimate the student
population in 2020. Ans. 374
15. If the shading process shown in the figure is continued indefinitely, what fractional part of the largest square will eventually be shaded?

Ans. 4/3
16. A side of a square is 12 cm. The midpoints of its sides are joined to form an inscribed square, and this process is continued. Find the sum of the
perimeters of the squares if this process is continued without end (round answer to two decimal places). Ans. 163.88 cm
17. A five-year old child receives an allowance of $1 each week. His parents promise him an annual increase of $2 per week. What will the child’s
allowance be when he is 16 years old? Ans. $ 23
18. The sum of three consecutive integers is - \,90−90. What is the largest integer? Ans. -29
19. A company offers a starting yearly salary of $33,000 with raises of $2500 per year. Find the total salary over a ten-year period. Ans. $ 55,500
20. In the recent COVID responses, there are 6400 frontliners are deployed in the city. 216 meals are provided assuming 3 meals per day to be consumed.
The meals were provided for 9 more days due to daily deaths from COVID -19. At an average , determine the number of deaths from COVID -19. Ans.
-18 (decreasing)
MEANS and MISCELLANEOUS SEQUENCES

ARITHMETIC AND GEOMETRIC MEAN

ARITHMETIC MEAN – the mean of the terms between a_1 and a_n of an arithmetic progression
GEOMETRIC MEAN – the mean of the terms between a_1 and a_n of a geometric progression

Σx :
%4 74 √Πx

Π = Product of all the terms , Σ = sum of all the terms, n = number of terms

HARMONIC PROGRESSION – sequence of numbers in which their reciprocals forms an arithmetic progression.
HARMONIC MEAN - the means of the terms a_1 and a_n of a harmonic progression
FIBONACCI SEQUENCE OR FIBONACCI PROGRESSION - The Fibonacci Sequence is the series of numbers: 0, 1, 1, 2, 3, 5, 8, 13, 21, 34, ...The next number
is found by adding up the two numbers before it.

RELATIONSHIPS BETWEEN HARMONIC, ARITHMETIC AND GEOMETRIC MEAN:

74 %4 ⋅ <4

EXAMPLES:

1. Find 5 arithmetic means between 7 and 19.


SOLUTION:
There are 7 terms.

1 ; 19 7 7 1 , 2.

Add by two gives , the next arithmetic means are 9,11,13,15,and 17.

2. (ECE Board) Find the fourth term of the progression 1/2 , 0.2 , 0.125 …
SOLUTION:
1/2 , 1/5 , 1/8 are in harmonic progression. d = 3 , The fourth term is 1/11 . Ans.

3. Determine the harmonic mean if the arithmetic mean is 9 and the geometric mean is 15.
SOLUTION:

74 %4 ⋅ <4, 15 9 ⋅ <4 , <4 25 % &.

EXERCISES – Solve the following problems.

1. (CE Board) The geometric mean of two numbers is 8 while the arithmetic mean is 4 , the cube of harmonic mean is ____. Ans. 4096
2. (CE Board) The geometric mean and the arithmetic mean of the two numbers are 8 and 17, respectively. Find the bigger number. Ans. 32
3. Find the three terms of the geometric progression between 18 and 2/9. Ans. 6,2 and 2/3 . Can be interchanged using r =3.
4. Given the numbers 4 and 8. Find the harmonic mean. Ans. 16/3
5. Find the tenth term of the harmonic progression 1/3 , 1/7 , 1/11 … Ans. 1/39
6. Insert three arithmetic means between 44 and 92. Ans. 56,68,and 80
7. Insert harmonic means between 3 and 7. Ans. 63/17 and 63/13 \
8. The first and second terms of a harmonic progression are ‘a” and “b” , respectively. What is the third term? Ans. ab/2a – b
9. What is the geometric mean of 1,3,9,27,and 81 ? Ans. 9
10. Find the twelfth term of the Fibonacci Sequence. Ans. 89

MATHEMATICAL INDUCTION AND OTHER SERIES AND SEQUENCES


MATHEMATICAL INDUCTION AND SEQUENCES AND SERIES FORMULAS

EXAMPLES:

1. Find the sixth term of this sequence:

1 1

1⋅3 3⋅5

SOLUTION:

1 1 1
% &.
2 1 2 1 2 6 1 2 6 1 143

2. Find the total number of balls piled in a pyramid with square base just one ball in the top layer if there are 4 layers.
SOLUTION:
EXERCISES – Solve the following problems.

1. Find the sum of the cubes of the first four multiples of 5. Ans. 12,500
2. Determine the next two terms of the sequence 3,5,11,21,43,85, … Ans. 171 and 341
3. Find the sixth term of the sequence (n^2 – 2) /3 Ans. 34/3
4. (ECE Board) Find the 2020th digit of the decimal form of 2020/9999. Ans. Second zero
5. Calculate the fourth term of the sequence (-1/n)^n. Ans. 1/256
6. Find the value of x of the given equation x + 2x + 4x + 8x … + 256x = 1022. Ans. X = 2
7. The NBA has a 14 team bubble tournament for those teams failed to make playoffs. How many games will be playing? Ans. 91

FINAL ALGEBRA TOPIC ON SEPTEMBER 21

1. Binomial Theorem
2. Partial Fractions
REFERENCES

Algebra and Trigonometry by Barnett


Engineering Mathematics by Besavilla
https://www.math10.com/problems/arithmetic-progression-problems/easy/
College Algebra by Sullivan
http://mathematics.laerd.com/maths/geometric-progression-intro.php
https://www.varsitytutors.com/hotmath/hotmath_help/topics/geometric-series
https://www.superprof.co.uk/resources/academic/maths/calculus/functions/geometric-sequence-problems.html
Engineering Mathematics by Bird
1001 Solved Problems in Engineering Mathematics by Tiong and Rojas
https://www.cliffsnotes.com/study-guides/algebra/algebra-ii/sequences-and-series/arithmetic-series
Precalculus by Blitzer
College Algebra by Stitz
https://www.bths.edu/ourpages/auto/2018/4/17/55554754/arithmetic%20series%20worksheet.pdf
https://www.onlinemath4all.com/how-to-find-the-first-three-terms-of-a-geometric-sequence.html
https://www.math.ucdavis.edu/~kouba/CalcTwoDIRECTORY/summationsoldirectory/SummationSol.html
https://1.cdn.edl.io/5BOi0UCa8HfeY0Djbebk3E759xuWPUgjaNrTxMnBj4eyd43v.pdf
Introductory and Intermediate Algebra by Blitzer
https://nanopdf.com/download/worksheet-114-infinite-geometric-series-word_pdf
Precalculus by Openstax
https://www.slideshare.net/denmarmarasigan/arithmetic-sequences-and-arithmetic-means
https://www.mathsisfun.com/numbers/fibonacci-sequence.html
https://www.nagwa.com/en/worksheets/717148546101/
Engineering Mathematics by Gillesania
https://www.basd.net/cms/lib2/PA01001269/Centricity/ModuleInstance/1719/AMC_CH_12_Packet.pdf
https://lhsblogs.typepad.com/files/harmonic-sequences-answers.pdf
https://socratic.org/questions/how-do-you-find-the-three-arithmetic-means-between-44-and-92
https://www.mathsisfun.com/numbers/geometric-mean.html s
Schaum’s Outlines of College Algebra
https://www.mathsisfun.com/algebra/mathematical-induction.html
https://math.libretexts.org/Courses/Saint_Mary's_College_Notre_Dame_IN/SMC%3A_MATH_339_-
_Discrete_Mathematics_(Rohatgi)/Text/2%3A_Sequences/2.E%3A_Sequences_(Exercises)
https://www.mathsisfun.com/algebra/sequences-series.html
CIVIL ENGINEERING BOARD EXAMS PROBLEMS PHILIPPINES – SEPTEMBER 21,2020

BINOMIAL AND MULTINOMIAL THEOREM

BINOMIAL THEOREM

For non-negative integers n and r, with n ≥ r the expression nCr is called binomial coefficient and defined by

!
! !

Note: The nomenclature nCr stands for combinations. This will be discussed in Numerical Data Analysis (Probability and Statistics) .

When we write out the binomial expression (a + b)^n , where n is a positive integer, a number of patterns begin to appear. This will be discussed further on
Pascal’s Triangle.

Each expanded form of the binomial expression is a polynomial. The patterns are as follows:

1. The first term (a + b)^n is a^n. The exponents on a decrease by 1 in each successive term.
2. The exponents of b in the expansion (a +b)^n increase by 1 in each successive term. In the first term, the exponent on b is 0. (Because b^0 =1 ,
b is not necessary to be written.) The last term is always b^n.
3. The sum of the exponents on the variables in any term in the expansion of (a + b)^n is equal to n.
4. The number of terms in the polynomial expansion is one greater than the power of the binomial n. There are n + 1 terms in the expanded form
of (a + b)^n.

For any positive integer n ,

SUM OF THE EXPONENTS

SUM OF THE COEFFICIENTS – Just use x = y = 1 .

PASCAL’S TRIANGLE – introduced by Blaise Pascal in which consists of an array of numbers showing coefficients of the binomial expansion (a +b)^n.

MULTINOMIAL THEOREM
The multinomial theorem extends the binomial theorem. It describes the result of expanding a power of a multinomial. We will show how it works for a
trinomial. Multinomials with 4 or more terms are handled similarly.

STEPS ON SOLVING MULTINOMIAL THEOREMS

1. Start solving the with three nested summations with this formula :
2. Solve for the filter factor known as Kronecker delta function , ( δ_i,j) . If i and j have similar terms, use the filter factor of 1 , otherwise 0.)
3. Compute the multinomial coefficient:

!
! ! !

This is known as repeating permutations that will be discussed in Numerical Data Analysis.

BINOMINAL THEOREM WITH NEGATIVE AND FRACTIONAL POWERS

FOR RATIONAL AND NEGATIVE POWERS

EXAMPLES:

1. Expand (3m – n)^4


SOLUTION:

2. Find the third term of this binomial expansion:

5
2

SOLUTION:

3. Find the sum of the coefficients of this binomial expansion:

SOLUTION: Substitute to 1 :

1 3 1 256 .

4. Calculate the term of the following binomial expansion which is free of x :

SOLUTION:
Consider the terms of x:

N = 15 ,
!"# ! "! $
Use exponential equations:

2 15 1 1 1 0, 11, ' ( 11(ℎ (* +.

Then,
$
2
15 15 11 1 3075072. .

5. Find the number of terms in the expansion of (x + y +z)^4 ?


SOLUTION:

The number of terms in a multinomial theorem is represented by Stars and Bars Theorem:

1 '. '- (* +

k - terms in a polynomial n – exponent

4 3 1 4 15 (* + .

6. Find the 3rd term of this expansion: (x + y)^-6


SOLUTION:

3rd term
1 /!
2!

K – term indicated:

6 6 1
21 .
2!

EXERCISES – Solve the following binomial expansion problems.

1. Find the second term of the expansion:

√ 2

Ans. 1
2. Determine the 4th term of the expansion (x^2 – 2a)^5 Ans. – 80x4a3
3. Find the coefficient of (3x – 2)^8 . Ans. -108,864
4. Find the fifth term of (4a + 3b)^7 Ans. 181,440a^3 b^4
5. Find the middle term of this expansion:
$
3
3

Ans. 252

6. Find the coefficient of p^5 in the expansion of (p+2)^6. Ans. 192


7. Determine the number of terms in the expansion (a +b +c)^6. Ans. 28
8. Expand (2c + 3m)^4 .

. 160 960 2 + 2160 + 2160+2 81+

9. Find the fifth term of the expansion (3 + x)^7 Ans. 945x^4


10. Find the third term of the expansion :

√1
3

Ans. -x^2/9
11. Determine the coefficient of x^20 in the expansion of (1 – 2x^4)^7. Ans. -672
12. Find the number of terms in the expansion of (x + y + z)^4? Ans. 15
13. Find the term involving x^2 y^7 in the expansion (3x – 5y)^9 Ans. -23,312,500
14. What is the coefficient of the term free of x of the expansion (2x – 5y)^4 ? Ans. 625
15. Find the sum of the exponents in the expansion (2x – y)^5 Ans. 30

PARTIAL FRACTIONS

Partial fractions are expressions of breaking down a rational expression into distinct number of terms. This refers to reverse of adding or subtracting rational
algebraic expressions.

Two polynomials are equal to each to other if and only if the coefficients of terms of like degree are equal. For a polynomial of degree n > 0 with real
coefficients, there always exists a factorization involving only linear or quadratic or both factors with real coefficients in which the quadratic factors have
imaginary zeros.

Any proper fraction P(x)/D(x) reduced to lowest terms can be decomposed into the sum of partial fractions as follows :

These fundamentals can be discussed further in Integral Calculus.

EXAMPLES:
5 7
1.
2 3
SOLUTION:
7 5 7
2.
2 2 1

SOLUTION:

7 2
3.
2 1 1

SOLUTION:
2 1
4.
1
SOLUTION:

EXERCISES – Resolve into partial fractions.

44 2 2
1. .
34 44 1 34 1 4 1

1 1 1 1
2. .
1 1
2 2 1 11 32
3. . 5
5 6 2 3

11 3 2 5
4. .
2 3 1 3

2 2 5 2 1 3 1
5. .
1 1 1

3 1 2
6. .
1 2 1 2
2 1 1 1 1
7. .
2 2 3 2 2 5 2 1 10 2
2
3 6 7 2 3 2
8. .
2 2 2 2 2 2

2 5
9. . 5'( *670 89*
4
3 1 1
10. .
3 3 2 3 2 3

5 1 2 3
11. .
3 4 3 4

1 16 81
12. . 6
1 2 3 2 1 2 2 3

22 4 3
13. .
2 8 2 4
8 1 1
14. .
2 6 2 6

2 1 3 5
15. .
2 2 4 2 4 2

END OF ALGEBRA BRIDGING PROGRAM

NEXT TOPIC ON SEPTEMBER 25 – TRIGONOMETRY

Trigonometry Topics On September 25


1. Introduction and the Unit Circle
2. Basic Trigonometric Functions
3. Right Triangles
REFERENCES:

Introductory and Intermediate Algebra


Engineering Mathematics by Besavilla
http://mathonweb.com/help_ebook/html/expressions_8.htm
http://mkhometuition.co.uk/index_files/A2-50_Binomial_Series_Rational.pdf
https://www.nagwa.com/en/explainers/465192710741/
https://brilliant.org/wiki/multinomial-theorem/#multinomial-theorem-examples-number-of-terms
Algebra for College Students by Mark Dugopolski
https://wl.apsva.us/wp-content/uploads/sites/38/2016/08/Binomial-Theorem-Worksheet-Longer-2016-17-1.pdf
Glencoe’s Advanced Mathematical Concepts
https://www.hitbullseye.com/Binomial-Expansion-Examples.php
Engineering Mathematics by Bird
https://math.stackexchange.com/questions/1997341/binomial-theorem-for-fractional-powers
http://teachers.wrdsb.ca/jhuff/files/2016/02/Binomial-Expansion-Worksheet.pdf
Engineering Mathematics by Rojas and Tiong
http://www.mesacc.edu/~scotz47781/mat150/notes/part_fract_decomp/Partial_Frac_Decomp_Notes.pdf
https://www.nagwa.com/en/worksheets/918104207408/
https://www.onlinemath4all.com/partial-fractions-quadratic-numerator-and-denominator.html
https://www.purplemath.com/modules/partfrac2.htm
Calculus by James Stewart
Precalculus by Blitzer
https://www.pbte.edu.pk/text%20books/dae/math_113/Chapter_04.pdf
PART 2:
PLANE AND SPHERICAL
TRIGONOMETRY
CIVIL ENGINEERING BOARD EXAMS PROBLEMS PHILIPPINES – SEPTEMBER 25, 2020

TRIGONOMETRY

INTRODUCTION

Trigonometry (from Greek trigōnon, "triangle" and metron, "measure") is a branch of mathematics that studies relationships between side lengths and angles
of triangles. The field emerged in the Hellenistic world during the 3rd century BC from applications of geometry to astronomical studies. The Greeks focused
on the calculation of chords, while mathematicians in India created the earliest-known tables of values for trigonometric ratios (also called trigonometric
functions) such as sine.

Throughout history, trigonometry has been applied in areas such as geodesy, surveying, celestial mechanics, and navigation.Trigonometry is known for its
many identities,] which are equations used for rewriting trigonometrical expressions to solve equations, to find a more useful expression, or to discover new
relationships.

THE UNIT CIRCLE - circle with 1 as radius and given by the equation x^2 + y^2 = 1.

Notice that the point (x, y) on the unit circle can be written as (cos θ , sin θ) If we recall the unit circle coordinate values for special angles (Section 6.4), we
can now summarize the exact values for sine and cosine in the following illustration.

(x,y) = (cos θ , sin θ) , θ – central angle whose terminal side intersects the unit circle at (x , y)

TRIGONOMETRIC FUNCTIONS – Let (x,y) by any point on the unit circle. If θ is a real number that represents the distance from the point (1 , 0 ) along the
circumference to the point (x , y) , then

REFERENCE ANGLES – angles between 0 to 360° in standard position whose terminal side lies one of the four quadrants and does have angles between 0
to 90° .
COTERMINAL ANGLES – angles in standard position with the terminal side.
NEGATIVE ANGLES – angles rotating clockwise with negative values.

CONVERSION OF UNITS
1 radian = 57.296° , 2π = 360°

ARC LENGTH – if a central angle θ in a circle with radius r intercepts an arc on the circle of length s, then the arc length is given by
𝜋
𝑠 = 𝑟𝜃 ( ) (𝑓𝑜𝑟 𝑟𝑎𝑑𝑖𝑎𝑛𝑠)
180

𝑠 = 𝑟𝜃 (𝑓𝑜𝑟 𝑑𝑒𝑔𝑟𝑒𝑒𝑠)

For the arc length of the other curves, see solid mensuration and integral calculus.
SIGNS OF THE TRIGONOMETRIC FUNCTIONS

Q I – all values are positive


Q II – y is positive
Q III – r is positive
Q IV – x is positive
EXAMPLES:

1. A belt connect a pulley of 2 inch radius with a pulley of 5 inch radius. If the larger pulley turns through 10 radians, through how many radius will the
small pulley turn ?
SOLUTION:

2. Find the reference angle of 225° .


SOLUTION:

3. P( -3 , 4) is a point on the Cartesian plane with the origin. Determine the value of (1/2) csc θ.
SOLUTION:

4. Find the tan θ given W(x) = (3/5 , -4/5)


SOLUTION:

5. Evaluate tan t that corresponds to the point ( -0.737 , 0.675) on the unit circle.
SOLUTION: \

6. Evaluate cos (- 5π/6)


SOLUTION:
7. Calculate the length of an arc if the radius of an arc is 8 cm and the central angle is 40°?
SOLUTION:

EXERCISES – Answer the following questions.

1. You have asked your landscaper to create a garden area in an oddly-shaped corner of your back yard. Regarding the fencing on the two sides of
the corner as being radii, the corner's central angle is two-thirds of a circle. You'd like the landscaper to install a particular type of edging, of which
you have 14 2/3 feet, as the arc of a circle whose center is the pole where the two fences meet. What will be the depth of your new garden? Ans.
7 feet
2. If csc θ = 4/3 , find tan θ . Ans. ±1.134
3. Find the coterminal angle of 870º. Ans. 210º
4. Let θ be any angle in standard position whose terminal side contains point (-4 , -5) . Find sec θ. Ans. √41/4
5. Given the points (√(p^2 +q^2) , √2pq ) , find sec A. Ans. (p+q)/√2pq
6. Find sin π/4 . Ans. √2/2
7. Evaluate sin 34π/6 Ans. -√3 /2
8. If tan θ = 84/13 and the terminal side lies in Q III , find sin θ. Ans. -84/85
9. Find the indicated trigonometric expression:

sin 0 + sin 1 + sin 2 + ⋯ + sin 90


cos 0 + cos 1 + cos 2 + ⋯ + cos 90

10. Find the exact value of cos (-2π/3) Ans. -1/2


11. Given csc θ = 25/7 , find sec θ. Ans. 25/24
12. An electric winch is used to pull a boat out of the water onto a trailer. The winch winds the cable around a circular drum of diameter 5 inches.
Approximately how many times will the winch have to rotate in order to roll in 5 feet of cable? Ans. 1.9 times
13. Find the angle β coterminal with 19π/4. Ans. 3π/4
14. Evaluate sin (-9π/2) Ans. -1
15. Given the coordinates (9,0) , Find cot θ. Ans. ∞
16. A bicycle with tires 90 cm in diameter is travelling at 25 km/h. What is the angular velocity of the tire in radians per second? Ans. 15.43 rad/s
17. If tan θ = 3.8436 , find θ. Ans. 75º 35’
18. The terminal side contains the point ( -60,-11). Find sec θ. Ans. -61/60
19. Find the length of an arc of a circle of radius 5 cm subtending a central angle measuring 15º Ans. 5π/12
20. The point (-√3/2 , 1/2 ) is on the unit circle. Find cot t. Ans. -√3

RIGHT TRIANGLE TRIGONOMETRY

PYTHAGOREAN THEOREM – The sum of the squares of the legs of a right triangle is equivalent to the square of the longest side (hypotenuse).

SPECIAL RIGHT TRIANGLES


1. 45-45-90 or Isosceles Right Triangle
2. 30-60-90 Right Triangle

APPLICATIONS
1. Geometric Problems
2. Navigation or Bearing
BEARING – angle in degrees measured clockwise from north.
TRUE BEARING – direction to an object from a point measured from 0 to 360° clockwise from true north.
AZIMUTH – the angle of an object in the sky along the horizon (discussed further in Surveying)
3. Angle of elevation and Angle of Depression
ANGLE OF ELEVATION – angle between the horizontal and line from the object above from observer’s eye.
ANGLE OF DEPRESSION – angle between the horizontal and line from the object below from observer’s eye.
4. Forces (refer to Statics for more force related problems)
5. Miscellaneous
TRIGONOMETRIC RATIOS OF RIGHT TRIANGLES

EXAMPLES

1. In triangle PQR shown, find the lengths of PQ and QR.

SOLUTION:

2. Solve the right triangle ABC with a = 4.32 cm and b = 2.62 cm.
SOLUTION :

3. Triangle ABC is a right triangle with A = 60º and a = 50 , find b,


SOLUTION:
4. In designing a house an architect wishes to determine the amount of overhang of a roof so that it shades the entire south wall at noon during the
summer solstice when the angle of elevation of the sun is 81° as shown in the figure. Minimally, how much overhang should be provided for this
purpose?

5. You are standing on top of a building, looking at park in the distance. The angle of depression is 53°. If the building you are standing on is 100 feet
tall, how far away is the park?
SOLUTION

6. (ECE Board) A man finds the angle of elevation of the top of a tower to be 30 degrees. He then walks 85 m nearer the tower and found its angle of
elevation to be 60 degrees. What is the height of the tower?
SOLUTION:

7. A guy wire supports a tower. The wire forms an angle of 57 degrees with level ground. The wire is anchored to the ground 16.5m away from the
base of the tower. How long is the guy wire?
SOLUTION:

16.5 16.5
cos 57° = , ℎ= = 30.3 𝑚 𝐴𝑛𝑠.
ℎ cos 57°
8. A jet takes off bearing N 28º E and flies 5 miles and then makes a left turn and flies 12 miles farther. If the control tower operator wants to locate
the plane, what bearing should he use?
SOLUTION:

9. An object is pushed across a table top with a force of 16.6 N directed 32.7 degrees S of E. What are the x- and y-components of this force?

EXERCISES A – Find the indicated value of the following right triangles shown.

1. Ans. x = 21

2. Ans. θ = 60.255º
3. β = 35º and c = 17 inches, find b. Ans. b = 9.75
4. Ans. 8.6 meters
5. If β = 17.8° and c = 3.45 , find a. Ans. 3.28

6. Given c = 91.92 and a = 2.19 , find B in ΔABC. Ans. 88° 38’ 5”


7. Find csc θ on the triangle shown. Ans. 3

8. In the triangle shown , find cot B. Ans. 2

9. Find ∠P in the figure shown. Ans. 17º 27’ 28”

10. Find x. Ans. 15.3


EXERCISES B – Solve the following questions.

1. A 100 ft. building casts a 300 ft. shadow from the sun. What is the angle of depression? Ans. 18.43º
2. The elevation of a tower from two points, one due east of the tower and the other due west of it are 20º and 24º, respectively, and the two points of
observation are 300 m apart. Find the height of the tower to the nearest meter. Ans. 60 meters
3. A man lives in a house that borders a pasture. He decides to go to the grocery store to get some milk. He is trying to decide whether to drive along
the roads in his car or take his all-terrain vehicle (ATV) across the pasture. His car is faster than the ATV, but the distance the ATV would travel is
less than the distance he would travel in his car. If sin θ = 3/5 and cos θ = 4/5 and he drove his car along the streets, it would be 14 miles round trip.
How far would he have to go on his ATV round trip? Ans. 5 miles

4. If the satellite in a geostationary orbit (at 35,000 km) was only 30 meters long, about how accurate would the pointing of the dish have to be? Give
the answer in degrees to two significant digits. Ans. 4.9 × 10^-5 degrees
5. The 40 meter flag pole has a rope makes a 25° angle with the ground. How long is the rope? Ans. 94.640 meters
6. (CE Board) Find the height of the tree if the angle of elevation its top changes from 20° to 40° as the observer advances 75 ft towards its base.
Ans. 48.21 meters
7. A ladder 40 feet long may be so placed that it will reach a window 33 feet high on one side of the street, and by turning it over without moving its
foot it will reach a window 21 feet high on the other side. Find the breadth of the street. Ans. 56.65 feet
8. A 40 m high tower stands vertically on a hillside which makes an angle of 18º with the horizontal. A tree also stands vertically up the hill from the
tower. An observer on top of the tower finds the angle of depression of the top of the tree to 26º and the bottom of the tree to be 38°. Find the height
of the tree. Ans. 10.62 meters
9. To measure the height of the cloud cover at an airport, a worker shines a spotlight upward at an angle 75º from the horizontal. An observer 600 m
away measures the angle of elevation to the spot of light to be 45º. Find the height of the cloud cover. Ans. 473.205 m
10. The machine tool diagram on the right shows a symmetric V-block, in which one circular roller sits on top of a smaller circular roller. Each roller
touches both slanted sides of the V-block. Find the diameter of the large roller, given the information in the diagram. Ans. 2.768 meters

11. Three ships are situated as follows” A is 225 miles west of C while B , due south of C, bears S 25º 10’ E from A. What is the bearing of A from B?
Ans. N 25º W
12. A boat leaves the entrance to a harbor and travels 25 miles on a bearing of N 42° E. The captain then turns the boat 90° clockwise and travels 18
miles on a bearing of S 48° E. What is the bearing of the boat from the harbor entrance? Ans. N 77º E
13. If a jet airliner climbs at an angle of 15°30’ with a constant speed of 315 miles per hour, how long will it take (to the nearest minute) to reach an
altitude of 8.00 miles? Assume there is no wind. Ans. 5.702 mins.
14. If Vx = 68.1 kph and Vy = 89.3 kph, find the angle of the resultant of the velocity. Ans. 52.7°
15. If the flagpole that a golfer aims at on a green measures 5 feet from the ground to the top of the flag and a golfer measures a 1° angle from top to
bottom, how far (in horizontal distance) is the golfer from the flag? Ans. 286.44 feet
16. A Men In Black agent is standing at ground level, 18 meters across the street, aiming his laser gun at the alien. At what angle, in degrees, should
the agent shoot his laser gun? Ans. 87.67°
17. Towers A and B stand on a level ground. From the top of tower A which is 30 m. high the angle of elevation of the top of tower B is 48˚. From the
same point the angle of depression of the foot of tower B is 26˚. What is the height of tower B in meters? Ans. 98.3 meters
18. A telephone pole 30 ft high is to be guyed from its middle section with a guy wire making an angle of 45 degrees with the ground. Find the total
length of the guy wire if an additional three feet is to be provided for splicing. Ans. 24.21 feet
19. A surveyor measures the angle of elevation of the top of a perpendicular building as 19º. He moves 120 m nearer the building and finds the angle
of elevation is now 47º. Determine the height of the building. Ans. 60.85 meters
20. The Smith's bought a 6-ft-square sheet of plywood as a base for their electric train. Will the plywood fit in the back of their van? The opening of the
van is 44 inches high and 60 inches wide. Ans. 6.2 feet

NEXT TOPIC ON SEPTEMBER 28 , 2020

1. Law of Sines, Cosines and Tangents


2. Ambigous Case
REFERENCES

https://en.wikipedia.org/wiki/Trigonometry
Algebra and Trigonometry by Openstax
Algebra and Trigonometry by Cynthia Young
https://www.siyavula.com/read/maths/grade-10/trigonometry-part-1/05-trigonometry-07
College Algebra with Trigonometry by Barnett
https://mathbitsnotebook.com/Geometry/Circles/CRArcLengthRadian.html
https://www.purplemath.com/modules/sectors2.htm
Schaum’s Outlines of Trigonometry
https://www.anderson5.net/cms/lib02/SC01001931/Centricity/Domain/2137/Trig%20in%20the%20Coordinate%20Plane%20with%20Notes%201.pdf
Plane and Spherical Trigonometry by Wentworth
Precalculus by Blitzer
https://www.ms.uky.edu/~ma110/s.18/worksheets/m110ws15keyfa17.pdf
Engineering Mathematics Vol 1 by Besavilla
https://www.govst.edu/uploadedFiles/Academics/Colleges_and_Programs/CAS/Trigonometry_Short_Course_Tutorial_Lauren_Johnson.pdf
PreCalculus by Openstax
https://www.intmath.com/trigonometric-functions/8-applications-of-radians.php
Algebra and Trigonometry Problem Solvers
http://www.brainkart.com/article/Radian-Measure_33910/
https://www.bbc.co.uk/bitesize/guides/zqwhjty/revision/1#:~:text=In%20mathematics%2C%20a%20bearing%20is,usually%20written%20as%20030%C2%B0
https://www.thefreedictionary.com/true+bearing
Engineering Mathematics by Bird
MTAP Reviewers
https://en.wikibooks.org/wiki/High_School_Trigonometry/Applications_of_Right_Triangle_Trigonometry
https://www.landmarkoutreach.org/wp-content/uploads/Finding-Side-Lengths.pdf
https://www.carlisle.k12.ky.us/userfiles/936/Classes/3415/ExamView%20-%20Right%20Triangle%20Test%20Review.pdf
http://pmoss.pbworks.com/w/file/fetch/60102520/10
Glencoe’s Advanced Mathematical Concepts
https://www.neshaminy.org/cms/lib/PA01000466/Centricity/Domain/240/angle_of_dep_angle_of_elev_ws.pdf
Licensure Examinations for CE by Besavilla
Algebra and Trigonometry by James Stewart
https://math.libretexts.org/Bookshelves/Precalculus/Book%3A_Elementary_Trigonometry_(Corral)/01%3A_Right_Triangle_Trigonometry_Angles/1.03%3A_A
pplications_and_Solving_Right_Triangles
http://www.technology.heartland.edu/faculty/todds/tmat103/Vectors%20for%20TMAT%20103.pdf
https://www.khanacademy.org/math/geometry/hs-geo-trig/hs-geo-modeling-with-right-triangles/e/applying-right-triangles
Mathematics by Padilla
Higher Engineering Mathematics by Bird
CIVIL ENGINEERING BOARD EXAMS PROBLEMS PHILIPPINES – September 28,2020

OBLIQUE TRIANGLES

The oblique triangles are triangles without right angles. It can be isosceles or scalene (in terms of lengths) or it can be acute or obtuse (in terms of angles). Regular
triangles are also oblique triangles.

The following given should meet on solving oblique triangles

1. Two angles
2. One angle and another side
3. The two other sides

Here are the cases on solving oblique triangles :

FORMULAS ON SOLVING THE OBLIQUE TRIANGLES

LAW OF SINES

LAW OF COSINES
LAW OF TANGENTS

EXAMPLES:
1. Solve the triangle shown.

SOLUTION:

2.
SOLUTION:

3. Solve the ΔABC given that a = 5 , b = 3 , ∠C = 96° and find A – B.


SOLUTION:
4. In a triangle XYZ , ∠X = 51° , ∠Y = 67° and YZ = 15.2 cm. Find the remaining parts.
SOLUTION:

5. Solve the triangle in the figure shown.

SOLUTION :

6. A lighthouse A and buoy B are 20 miles apart, the buoy lying due east of the lighthouse. A ship sailing 8 mph leaves from A at the same time another
ship sailing 6 mph and leaves B on a course at unknown angle west of the smooth. It is known that the ships are nearest to each other after they sailed
1 18/37 hours, the distance between them is 30√111 / 37 miles. Find the course of the second ship from the first when they are nearest.
SOLUTION:
7. Two fire-lookout stations are 15 miles apart, with station A directly east of station B. Both stations spot a fire. The angular direction of the fire from station
B is N52°E and the angular direction of the fire from station A is N36°W. How far is the fire from station A?
SOLUTION :

8. In the parallelogram shown find the diagonal BD.

SOLUTION :

EXERCISES A – solve the indicated unknown in the oblique triangles.

1. Find BC. Ans. 8.34 m (CE Board)


2. Find AC. Ans. 25

3. α = 71.2° , b = 5.32 m and c = 5.03 m, find A. Ans. 6.03 m


4. For ΔABC, A = 30º , b = 4 and c = 5. Find B. Ans. 52.5°
5. Find BC. 33.1 yards

6. Solve for γ in the triangle shown. Ans. 15.665°

7. Calculate the side c in the figure shown. Ans. 11.8

8. Find angle α if a = 20 , b = 25 and c =18. Ans. 52.4°


9. Find side e in the ΔDEF given that EF = 35.0 mm ,DE 25 mm and ∠E = 64° Ans. 32.91 mm
10. If the triangle ABC, C = π/6 , b = √3 and a = 1 , find the third side. Ans. c =1

EXERCISES B – Solve the following questions.

1. A farmer has a triangular field with sides 120 yards, 170 yards, and 220 yards. Find the biggest angle opposite of the biggest side. Ans. 97.2°
2. The Tower of Pisa was originally built 56 meters tall. Because of poor soil in the foundation, it started to lean. At a distance 44 meters from the base of
the tower, the angle of elevation is 55° How much is the Tower of Pisa leaning away from the vertical position? Ans. 5 degrees.
3. Find the height of the tree if the shadow length is 157 feet and, relative to the horizontal, the hill slopes 11.0° and the angle of elevation of the sun is
42.0°. Ans. 108.81 feet
4. A pilot wishes to fly an airplane due East, but a strong wind blowing Southeast at 100 km/h keeps blowing the airplane off-course. If the airplane has a
cruising speed of 450 km/h, in what direction should the pilot fly to reach the destination? Ans. N 9° E
5. The Duquesne Incline is a cable car in Pittsburgh, Pennsylvania, which transports passengers up and down a mountain. The track used by the cable
car has an angle of elevation of 30°. The angle of elevation to the top of the track from a point that is horizontally 100 feet from the base of the track is
about 26.8°. Find the length of the track. Ans. 807.71 feet
6. (CE Board) A 40 m high tower stands vertically on a hillside which makes an angle of 18º with the horizontal. A tree also stands vertically up the hill from
the tower. An observer on top of the tower finds the angle of depression of the top of the tree to be 26º and the bottom of the tree to be 38°. Find the
height of the tree. Ans. 10.62 meters
7. In a parallelogram, the adjacent sides measure 22 inches and 40 inches. If the larger angle of the parallelogram measures 116º, find the length of the
longer diagonal, to the nearest tenth. Ans. 53.4 inches
8. Two forces of 115 lb and 215 lb acting on an object have a resultant of magnitude 275 lb. Find the angle between the directions in which the given
forces act. Ans. 70º 50’
9. To measure length of the lake, a baseline is established and measured to be 125 meters. Angles A and B are measured to be 41.6º and 124.3º ,
respectively. How long is the lake? Ans. 341 meters
10. In the figure shown, a radar ship is 30 miles off shore when a large fleet of ships leaves port at an angle of 43º. If the maximum range of the ship’s
radar is 20 miles, will the departing fleet be detected? Ans. No
11. A wall that is 1.4 m long has started to lean and now makes an angle of 80º with the ground. A 2 m board is jammed between the top of the wall and
the ground to prop up the wall. What angle does the board make with the ground? Ans. 44°
12. Suppose a soccer player runs up to a moving soccer ball located at 𝑨 and kicks the ball into the air. The diagram below shows the initial velocity of the
ball along the ground and the initial velocity and direction of the kick. What is the resultant velocity and angle of elevation of the soccer ball immediately
after it is kicked? Ans. 21.05 m/s and 16.92°

13. A man owns a triangular lot on the corner of two intersecting streets which intersects at an angle of 62º. The frontage of one street is 200 m and on the
other side is 150 m. Determine the perimeter of this lot. Ans. 535.29 m
14. A room 8 m wide has a span roof which slopes at 33° on one side and 40° on the other. Find the length of the roof slopes. Ans. 4.556 and 5.377 m
15. Two planes leave an airport at the same time and fly for two hours. Plane A flies in the direction of 165° at 385 km/h and plane B flies in the direction of
250° at 410 km/h. How far apart are the planes after two hours? Ans. 1074.8 km
16. (ECE Board) Points A and B 1000 m apart are plotted on a straight highway running east and west. From A , the bearing of a tower C is 32° N of W and
from B the bearing of C is 26° N of E . Approximate the shortest distance of tower C to the highway. Ans. 273.92 m
17. Circular tracts of land with diameters 900 meters, 700 meters and 600 meters are tangent to each other externally. There are houses directly in the
center of each circle. What is the smallest angle of the triangle connecting the houses? Ans. 49.46°
18. Suppose a boat leaves port, travels 10 miles, turns 20 degrees N of W , and travels another 8 miles. How far from port is the boat? Ans. 17.7 mi.
19. A plane flew due north at 500 mph for 3 hours. A second plane, starting at the same point and at the same time, flew southeast at an angle clockwise
from due north at 435 mph for 3 hours. At the end of the 3 hours, how far apart were the two planes? Ans. 2,709.891 miles
20. In a parallelogram given the two diagonals 5 and 6 and the angle that they form 49° 18’. Find the sides. Ans. 2.338 and 5.003

AMBIGOUS CASE OF TRIANGLES

AMBIGUOUS CASE SSA – Side Side Angle


AMBIGUOUS CASE USING QUADRATIC FORMULA’s DISCRIMINANT

One positive solution – one triangle


Two distinct positive solutions - two triangles
No positive solution – no triangle

CONSISTENCY OF SOLUTIONS OF OBLIQUE TRIANGLES USING MOLLWEIDE’S EQUATION

EXAMPLES:

1. Solve the triangle(s) if any for a = 27 inches , b = 28 inches and α = 110°

2. Find all solutions for a =4 , b = 3 and A = 112 º


SOLUTION:

3. Solve the triangles PQR given that QR = 36.5 mm , PR = 26.6 mm and ∠Q = 36° .
SOLUTION:
4. Can a triangle have the parts a =6 , b =7 , c = 9 , A = 55° B =60° and C = 65° ?
SOLUTION:

EXERCISES – Answer the following questions.

1. Determine that how many triangles can be formed in b = 30 , c = 20 and β = 70º Ans. No triangle
2. Determine the side c if any in triangle ABC given that A = 43° a = 81 and b = 62. Ans. 114.2
3. If A = 30° , a = 7 and b =16, how many triangles formed using quadratic formula? No triangle
4. If B = 14.4° , a = 8 and b = 3, solve the angle C if any. Ans. 27.1° and 124.1°
5. Solve ∠C if any for a = 31.5 , b = 51.8 and A = 33° 40’. Ans. 80° 40’ and 32°
6. How many triangles in the solution for β = 27.3° ,a = 244 cm and b = 135 cm ? Ans. Two
7. Determine the remaining side if any for the triangle a = 8.1 m , b = 8.3 m and α = 72° Ans. 4.4 and 0.74
8. How do you determine the number of triangles given a = 8 , b = 10 and A = 20°? Ans. Two
9. How many triangles can be formed in the figure shown? Ans. One

10. At certain times during the year, you can see Venus in the morning sky. The distance between Venus and the sun is approximately 67 million miles.
The distance between Earth and the sun is approximately 93 million miles. Estimate the distance between Venus and Earth if the observed angle
between the sun and Venus is 34°. Ans. 119 and 34.8 million miles

NEXT TOPICS ON OCTOBER 2,2020

1. Areas of the triangles


2. Bisectors and Inscribed and Circumscribed Triangles
REFERENCES

College Algebra and Trigonometry by Barnett


College Algebra and Trigonometry by Cynthia Young
https://byjus.com/maths/law-of-tangents/
Engineering Mathematics by Bird
Algebra and Trigonometry by Openstax
Engineering Mathematics by Besavilla
Licensure Examinations by Besavilla
https://www.varsitytutors.com/act_math-help/how-to-find-the-length-of-the-diagonal-of-a-parallelogram
https://www.chino.k12.ca.us/site/handlers/filedownload.ashx?moduleinstanceid=28577&dataid=54110&FileName=Law%20of%20Sines%20and%20Cosines%20
Review%20Worksheet.pdf
Trigonometry by Michael Corral
https://lindblomeagles.org/ourpages/auto/2016/4/29/45784385/5_2%20preactice%20_Law%20of%20sines%20and%20cosines_.pdf
Precalculus by Cynthia Young
https://www.mathsisfun.com/algebra/trig-sine-law.html
Pre Calculus by Openstax
https://www.math-only-math.com/law-of-tangents.html
https://faculty.atu.edu/mfinan/1203/Lecture25.pdf
http://jongarvin.com/up/MPM2D/slides/apps_sc_laws_handout.pdf
Glencoe’s Advanced Mathematical Concepts
https://mathbitsnotebook.com/Geometry/TrigApps/TALawCosinesPractice.html
Scahum’s Outlines of College Mathematics
https://www.tvusd.k12.ca.us/site/handlers/filedownload.ashx?moduleinstanceid=26113&dataid=9986&FileName=Ch%208%20Applications%20of%20Trigonomet
ry.pdf
Plane and Spherical Trigonometry by Wentworth
https://www.highskills.ca/wp-content/uploads/sites/83/2017/11/MCF3M-MNF-Day-4-Applications-of-the-Sine-Law.pdf
http://anna-kuczynska.weebly.com/uploads/1/8/5/0/18500966/trigonometry_085_-_section_t5_the_law_of_sines_and_cosines_and_its_applications.pdf
https://www.houstonisd.org/cms/lib2/TX01001591/Centricity/Domain/21026/PreCalc_Unit_6_Soln.pdf
Algebra and Trigonometry by Openstax
http://faculty.madisoncollege.edu/alehnen/Trigonometry/Mollweide_Formulas.pdf
http://www.nabla.hr/GE-
AppTrigonomB1.htm#:~:text=Example%3A%20Determine%20length%20of%20sides,and%20angle%20g%20%3D113%C2%B0.&text=Example%3A%20Given%
20is%20the%20sum,and%20angles%20of%20the%20triangle.
https://math.libretexts.org/Bookshelves/Precalculus/Book%3A_Elementary_Trigonometry_(Corral)/02%3A_General_Triangles/2.03%3A_The_Law_of_Tangents
https://mathbitsnotebook.com/Geometry/TrigApps/TAUsingLawCosines.html
https://yoshiwarabooks.org/trig/The-Law-of-Cosines.html
Schaum’s Outlines of Trigonometry
https://socratic.org/questions/how-do-you-determine-the-number-of-possible-triangles-and-find-the-measure-of-th#498376
CIVIL ENGINEERING BOARD EXAMS PROBLEMS PHILIPPINES – October 2,2020

AREAS OF TRIANGLES

GIVEN THE HEIGHT AND THE BASE (GENERAL FORMULA)

1
𝐴= 𝑏ℎ
2

GIVEN THE SIDES AND AN INCLUDED ANGLE

GIVEN ALL THREE SIDES (HERON’S FORMULA)

GIVEN TWO ANGLES AND INCLUDED SIDE

EXAMPLES

1. Solve the area of the triangle DEF given that EF = 35 mm , DE = 25 mm and ∠E = 64°
SOLUTION:

2. Farmer Jones owns a triangular piece of land. The length of the fence AB is 150 m. The length of the fence BC is 231 m. The angle between fence
AB and fence BC is 123º. How much land does Farmer Jones own?
SOLUTION:

3. The area of the triangle is 8,346 sq m and two of its interior angles are 37° 25’ and 56° 17’. What is the length of the largest side?
SOLUTION:
4. Find the area of the triangle given that a = 5 m , b = 7 m and c = 10 m.
SOLUTION:

5. A parking lot is to have the shape of a parallelogram that has adjacent sides measuring 200 feet and 260 feet. The acute angle between two adjacent
sides is What is the area of the parking lot?
SOLUTION:
Using the formula and parallelogram have two congruent triangles each.

𝐴 = 𝑎𝑏 sin 𝜃 = 2[200(265) sin 65°] = 96,068.625 𝑓𝑡. 𝐴𝑛𝑠.

6. (ECE Board) Two triangles have equal bases. The altitude of one triangle is 3 units more than its base and the altitude of the other triangles is 3 units
less than its base. Find the altitudes if the areas of the triangles differ by 21 units.
SOLUTION:

EXERCISES – Answer the following questions.

1. Suppose in a right triangle one of the legs is of length 5 and the angle formed by the hypotenuse and this leg is 28°. What is the area? Ans. 6.646
2. A triangle has two known sides, namely AB = 40 m, AC = 60 m. Point D is on line AB such that AD = 25 m. Point E is on the line AC such that the
area ADE is one third of the area of ABC. How far is E from A? Ans. 32 cm
3. Before she goes camping, La Verne has to buy a tent pole to replace the one she lost on her last outing. If the area of the front of the tent is 22 square
feet and the base of the tent has the dimensions 8 feet on its base, how tall must the pole be? Ans. 44/8 feet
4. Find the area of the triangle with a = 12 yards, b = 16 yards, and c = 24 yards. Ans. 85 sq yards
5. The perimeter of an isosceles triangle is 100 cm. If the base is 36 cm, find the length of the equal sides. Ans. 32 cm
6. A Chicago city developer wants to construct a building consisting of artist’s lofts on a triangular lot bordered by Rush Street, Wabash Avenue, and
Pearson Street. The frontage along Rush Street is approximately 62.4 meters, along Wabash Avenue it is approximately 43.5 meters, and along
Pearson Street it is approximately 34.1 meters. How many square meters are available to the developer? Ans. 711.4 m^2
7. If AB = 27 , CD = 20 and the area of the triangle ADC = 240 , what is the area of the quadrilateral ABCD? Ans. 1128

8. A triangle has an area of 90 square cm. Find the length of the base if the corresponding base is 3 cm more then the height. Ans. 15 cm
9. A woman hikes 503 m, turns and jogs 415 m, turns again, and runs 365 m returning to her starting point. What is the area of the triangle formed by
her path? Ans. 74,600 m^3
10. (CE Board) If the midpoints of the sides of a certain triangle are connected, the area of the triangle thus formed is what part of the area of the original
triangle? Ans. 1/4
11. In the figure below, ABCD is a rectangle of length of length 60 and width 30. The length of of BF and GC are respectively equal to 10 and 20. Find
the area of the shaded region. Ans. 1350
12. Triangle XYZ has base angles X = 52º and Z = 60º , distance XZ = 400 m long. A line AB which is 20 m long is laid out parallel to XZ. Compute the
area ABXZ. Ans. 44,162 m^2
13. To find the length AB of a small lake, a surveyor measured angle ACB to be 96°, AC to be 91 yards, and BC to be 71 yards. What is the approximate
area he surveyed? Ans. 3212.803 yards^2
14. A building site is in the form of a quadrilateral as shown and its area is 1510 m^2. Determine the length of the perimeter of the site. Ans. 163.364 m

15. Calculate the area of the regular octagon if each side is 20 mm and the width across the flats is 48.3 mm. Ans. 1932 mm
16. Find the area of the triangle in the figure shown. Ans. 27.9 miles^2

17. A triangle blank of equal sides is to punch in a copper plate, the area of the blank should be 24 sq. cm find the side. Ans. 7.4 cm
18. A triangular sail has a perimeter of 25 m. Side a is 2 m shorter than twice side b, and side c is 3 m longer than side b. Find the area of the sail.
Ans. 23.419 m^2
19. Find the area of a rhombus to the nearest integer if the side has length 30 and an angle measures 55° Ans. 737.24
20. The area of the inner triangle ABC is 1/4 that of the triangle ADE. AB = 45 m, AD = 65 m , AE = 90 m and DE = 80 m. Point B is along line AD and
point C is along line AE. Find the length of segment CE. Ans. 57.5 m.

BISECTORS AND TRIANGLE RELATIONSHIPS


PROPERTIES OF TRIANGLES

MEDIAN OF A TRIANGLE – line drawn from one vertex to the midpoint of its opposite side. The medians of a triangle intersect at a common point known as
CENTROID .
ALTITUDES OF A TRIANGLE – line drawn from one vertex perpendicular to its opposite side. The altitudes intersected known as ORTHOCENTER.

ANGLE BISECTOR OF A TRIANGLE – line drawn from one vertex to the opposite bisecting the angle included between the two other sides. The bisectors
intersect at a point known as INCENTER.

PERPENDICULAR BISECTORS OF A TRIANGLE line drawn from the midpoint of a triangle perpendicular about its opposite side. The perpendicular bisectors
intersect is known as CIRCUMCENTER.

EULER LINE - line going through several important triangle centers, including the orthocenter, circumcenter, centroid, and center of the nine point circle. This
also determines if the triangle is equilateral.
CIRCLE – TRIANGLE RELATIONSHIPS
CIRCLE CIRCUMSCRIBED ABOUT A TRIANGLE – circle is circumscribed about a triangle if it passes through the vertices of the triangle.

CIRCLE INSCRIBED IN A TRIANGLE – circle is inscribed in a triangle if it is tangent to the three sides of the triangle.

CIRCLES ESCRIBED ABOUT A TRIANGLE – circle is escribed about a triangle if it is tangent to one side and to the prolongation of the other two sides. A
triangle has three escribed circles.

EXAMPLES:
1. P is the centroid of the ΔQRS and RP = 10. Find RT.
SOLUTION :
2. (ECE Board) If an equilateral triangle is circumscribed about a circle of radius 10 cm, determine the side of the triangle.
SOLUTION:

EXERCISES – Answer the following questions

1. Find PV if PW = 7. Ans. 7

2. Find the length of the median to the longest side of the triangle whose sides are 40,50, and 70, respectively. Ans. 28.723
3. The area of a triangle ABC is 65.59 cm and its perimeter is 48 cm. Compute the radius of the inscribed circle. Ans. 2.71 cm
4. Find the radius r of the inscribed for the triangle ABC given that a = 2 , b = 3 and c =4. Ans. 0.645
5. The ΔABC’s sides are AB = 6cm, BC = 4cm and AC = 5.5cm. locate its orthocenter of the longest side. Ans. 3.566 cm
6. In the figure shown , ND = 5x -1 and NE = 2x +11 . Find NF. Ans. 19

7. (CE Board) What is the radius of the circle circumscribing an isosceles right triangle having an area of 162 cm^2? Ans. 12.728 cm
8. Given triangle ABC whose angles are A = 40° , B = 95° and side b = 30 cm. Find the length of the bisector of angle C. Ans. 21.74 cm

NEXT TOPICS ON OCTOBER 5, 2020 :

1. Inverse Trigonometric Functions


2. Graphs of Trigonometric Functions
3. Hyperbolic Functions
REFERENCES

Algebra and Trigonometry by Cynthia Young


Engineering Mathematics by Besavilla
Higher Engineering Mathematics by Bird
Precalculus by Cynthia Young
https://www.onlinemath4all.com/area-of-triangle-word-problems.html
Engineering Mathematics by Rojas
http://www.algebralab.org/lessons/lesson.aspx?file=Trigonometry_WORD-AreaPerimeterTriangles.xml
https://www.onlinemathlearning.com/word-problems-area-triangle.html
Pre Calculus by Blitzer
https://www.hitbullseye.com/Triangle-Problems.php
Algebra and Trigonometry by Openstax
Glencoe’s Advanced Mathematical Concepts
https://www.analyzemath.com/middle_school_math/grade_8/triangles.html
Scahum’s Outlines of Trigonometry
Licensure Examinations by Besavilla
https://www.analyzemath.com/Geometry/area-of-triangles-problems-with-solutions.html
https://cpb-us-e1.wpmucdn.com/cobblearning.net/dist/d/1007/files/2013/03/Assignment-2-Areas-of-Oblique-Triangles-Odd-Answers-2kadldx.pdf
https://www.pbte.edu.pk/text%20books/dae/math_113/Chapter_10.pdf
https://www.somerset.k12.ky.us/userfiles/103/Word%20Problems%20Perimeter%20and%20Age.pdf
Schaum’s Outlines of Geometry
Engineering Mathematics by Gillesania
https://www.mathsisfun.com/definitions/circumcircle-of-triangle.html
https://brilliant.org/wiki/euler-line/
https://www.murrieta.k12.ca.us/cms/lib5/CA01000508/Centricity/Domain/1830/T4.6.pdf
https://cdn.kutasoftware.com/Worksheets/Geo/5-Angle%20Bisectors%20of%20Triangles.pdf
https://math.libretexts.org/Bookshelves/Precalculus/Book%3A_Elementary_Trigonometry_(Corral)/02%3A_General_Triangles/2.05%3A_Circumscribed_and_I
nscribed_Circles
CIVIL ENGINEERING BOARD EXAMS PROBLEMS PHILIPPINES - OCTOBER 5 ,2020

INVERSE TRIGONOMETRIC AND HYPERBOLIC FUNCTIONS

INVERSE TRIGONOMETRIC FUNCTIONS

HYPERBOLIC FUNCTIONS – functions associated with the hyperbolas. These can be used in catenary problems in statics. Hyperbolas will be discussed
in Analytic Geometry.
EXAMPLES :

1. Solve sin-1 (-√2 /2)


SOLUTION:

2. cos (cos-1 (√3 /2))


SOLUTION:

3. tanh ln x
SOLUTION:

4. cosh 2.4625
SOLUTION:

EXERCISES – Evaluate the following inverse trigonometric and hyperbolic functions.

1. tanh 0.65 𝐴𝑛𝑠. 0.5717 9. sinh ln 8 𝐴𝑛𝑠. 3.9375

5 119 1 4 8
2. cos (2 sin−1 ( )) 𝐴𝑛𝑠. 10. sin−1 + cos−1 − tan−1 𝐴𝑛𝑠. 14.707
13 169 3 5 9

−5𝜋
3. csc −1 −2 𝐴𝑛𝑠. 11. sec(sec −1 0) 𝐴𝑛𝑠. ∞
6
3
4. tanh ln 2 𝐴𝑛𝑠. 12. sin−1 0.97 𝐴𝑛𝑠. 75.93°
5

−1 5
5. cot (sin−1 ( ) ) 𝐴𝑛𝑠. 2√2 13. cosh ln 3 𝐴𝑛𝑠.
3 3

13𝜋
6. sin−1 (cos ( )) 𝐴𝑛𝑠. 60° 14. cos(sin−1 0.6) 𝐴𝑛𝑠. 0.8
6
41 9
7. cot (sec −1 ( )) 𝐴𝑛𝑠. 15. cos −1 −0.5 𝐴𝑛𝑠. 120°
9 40

8. csc −1 √2 𝐴𝑛𝑠. 45°

GRAPHS OF TRIGONOMETRIC FUNCTIONS

PERIODIC FUNCTIONS

ODD AND EVEN FUNCTIONS

EVEN FUNCTIONS – functions that are symmetric with respect to y-axis


ODD FUNCTIONS – functions that are symmetric with respect to x-axis

SINE FUNCTIONS

COSINE FUNCTIONS
TANGENT FUNCTIONS

COTANGENT FUNCTIONS
SECANT FUNCTIONS

COSECANT FUNCTIONS

GENERAL EQUATION OF THE GRAPHS OF TRIGONOMETRIC FUNCTIONS

𝑦 = 𝐴 sin(𝐵𝑥 − 𝐶) + 𝐷

AMPLITUDE – vertical height from center to the peak of the graph


PERIOD – length from one peak to another.
PHASE SHIFT – distance of the horizontal or vertical position moved
FREQUENCY - reciprocal of period

|A| - amplitude , B – cycles from 0 to 2π (frequency) period = 2π/B (π/B for non Sine or Cosine functions) D - vertical shift C – phase or horizontal shift
Frequency – 1/period . phase shift = -C/B
EXAMPLES

1. Find the period of y = sin 2x.


SOLUTION:

2. What is the phase shift of the graph y = sin (π + θ) ?


SOLUTION:

3. Determine the frequency of the trigonometric function y = 3 sin(100t + 1)


Ans. A = 3 , amplitude is 3.

4. Determine the amplitude of the graph y = 2 sin [4(x- 0.5) +3]


SOLUTION:
EXERCISES - Answer the following questions.

1. Find the period of the function y = 2 – 3sin 2π/3 x . Ans. 3


2. Find the amplitude of the graph y = 3sin(2x) . Ans. 3
3. Determine the period of the trigonometric function y = sec (x – π/2) Ans. 2π
4. What is the period of the function y = 2 sin x? Ans. 2π
5. Two frequencies of sound are played on an instrument governed by the equation n(t) = 8 cos(20πt)cos(1000πt). What is the period of this
function? Ans. 1/10
6. Determine the magnitude of the phase shift for f(x) = sin(x + π/6) – 2. Ans. π/6 to the left.
7. Determine the period of this trigonometric function:
𝑥 𝜋
𝑦 = tan ( − )
3 3

Ans. 3π
8. Find the amplitude of the function y = 5sin(A + 30°) from ) 0 to 360° . Ans. 5
9. Find the frequency of the trigonometric function f(x) = 2 tan 4x. Ans. 2/π
10. Determine the amplitude of the function y = -3 cos (πx/2) . Ans. 3
11. State the period of the trigonometric function y = 1 – tan(x - π/2) . Ans. π
12. Find the period of the function y = tan (x – π/3) Ans. π
13. Find the amplitude of the function y = cos (x – π/2). Ans. 1
14. Determine the frequency of the y = sec πx Ans. No Solution
15. Find the period of this function y = 5sin(2x) . Ans. π

NEXT TOPICS ON OCTOBER 9, 2020:

1. Trigonometric Identities
2. Sum and Difference Formulas
3. Product to Sum and Sum to Product Formulas
4. Double and Half Angle Formulas
5. Power Reducing Formulas
6. Cofunction Formulas
REFERENCES

1. Algebra and Trigonometry by Barnett


2. Higher Engineering Mathematics by Bird
3. Calculus by James Stewart
4. Advanced Mathematical Concepts by Glencoe
5. https://tutorial.math.lamar.edu/extras/algebratrigreview/inversetrig.aspx
6. http://www.phengkimving.com/calc_of_one_real_var/07_the_exp_and_log_func/07_06_the_hyp_func.htm
7. Scahum’s Outlines of Trigonometry
8. https://s3.amazonaws.com/calculus-worksheets/calculus-2-tutor/Calculus+2+Tutor+-+Worksheet+3+-+Hyperbolic+Functions.pdf
9. Precalculus by Blitzer
10. https://courses.lumenlearning.com/suny-osalgebratrig/chapter/inverse-trigonometric-functions/
11. Algebra and Trigonometry by Cynthia Young
12. http://nebula2.deanza.edu/~bert/2009Fall/1bhf.pdf
13. https://www.sac.edu/FacultyStaff/HomePages/MajidKashi/PDF/MATH_180/Sec_1.4.1.pdf
14. https://mathbitsnotebook.com/Algebra2/TrigGraphs/TGsinusoidal.html#:~:text=Sinusoids%20are%20considered%20to%20be%20the%20genera
l%20form%20of%20the%20sine%20function.&text=y%20%3D%20A%20sin(Bx),affects%20the%20amplitude%20(height).
15. https://www.mathsisfun.com/algebra/amplitude-period-frequency-phase-shift.html
16. https://math.libretexts.org/Bookshelves/Precalculus/Book%3A_Elementary_Trigonometry_(Corral)/05%3A_Graphing_and_Inverse_Functions/5.
02%3A_Properties_of_Graphs_of_Trigonometric_Functions
17. Algebra and Trigonometry by Openstax
18. Engineering Mathematics by Bird
19. https://www.classzone.com/eservices/home/pdf/student/LA214AAD.pdf
20. Precalculus by Cynthia Young
21. https://www.varsitytutors.com/hotmath/hotmath_help/topics/graphing-tangent-function
22. https://www.shsu.edu/~kws006/Precalculus/4.5_Graphs_of_Six_Trig_Fns._files/S%26Z%2010.5.pdf
23.
CIVIL ENGINEERING BOARD EXAMS PROBLEMS PHILIPPINES October 9, 2020

TRIGONOMETRIC IDENTITIES

PYTHAGOREAN AND BASIC IDENTITES

EXAMPLES:

1. Find the simplified form:

1 + sin 𝑥 cos 𝑥
+
cos 𝑥 1 + sin 𝑥

SOLUTION:

2. Simplify cos θ + sin θ tan θ


SOLUTION:
3. If csc θ = 4/3 , find tan θ.
SOLUTION:

4. Simplify the identity:

1 + cot 𝜃
1 + tan 𝜃

SOLUTION:

EXERCISES – Answer the following questions.

1. Simplify (1 + cot)^2 + (1 – cot)^2 . Ans. 2 csc θ


2. If sec u = -3/2 and is in Q III , find cot u. Ans. 2/√5
3. Find sin θ if tan = 3/4 and cos < 0 . Ans. -3/5
4. Simplify : (1 + sin x)/(1 – sin x) + (1 – sin x)/(1 + sin x) . Ans. 4tan x sec x
5. Evaluate: sin ( - π/3 ) . Ans. √3 /2
6. If A = 3 sin x + 4 cos x and B = 3 cos x + 4 cos x , find A^2 + B^2 . Ans. 25
7. Find sec^2 in the figure shown. Ans. 25/16

8. Find csc t – sin t . Ans. Cot^2 t/csc t


9. Simplify: sin^2 x (1 + cot^2 x) . Ans. 1
10. Find this expression when tan θ = -4/3 and in Q II. Ans. 23/5
sin 𝜃 + cos 𝜃 − tan 𝜃
sec 𝜃 + csc 𝜃 − cot 𝜃

11. Find tan^2 x / (tan^2 x + 1 ) . Ans. Sin^2 x


12. Determine sin m (csc m – sin m) . Ans. Cos^2 m
13. The terminal side of an angle θ in standard position passes through the point (2 , 5). Calculate cos^2 θ. Ans. 4/29
14. Find sec x √(1 – sin^2) . Ans. 1
15. Evaluate tan^2 x – sin^2 x. Ans. Tan^2 x sin^2 x
HALF AND DOUBLE ANGLE FORMULAS

EXAMPLES:
1. Find the exact value of 165° .
SOLUTION:

2. Find cos 2x if sin x = √5/5.


SOLUTION:
3. Determine (sin θ + cos θ)^2.
SOLUTION:

4. Find (1 – cos x)/2


SOLUTION:

EXERCISES – Answer the following questions.

1. Find the exact value of sin 22.5° . Ans.


2. Evaluate tan 2π . Ans. 0
3. Evaluate cot 2x + csc 2x . Ans. Cot x
4. Simplify : tan x + cos x. Ans. 2 csc 2x
5. Find the exact value of x in the figure shown. Ans. x = 2√3 ft

6. Find the exact value of cot 15° . Ans. √3 +2


7. Evaluate the following:

(tan 2𝑥)(1 + tan 𝑥)


tan 𝑥

Ans. 2/(1 – tan x)


8. Determine the value of sec 165° Ans. √2 - √6
9. In the Figure shown , determine the value of x. Ans. 7.29167

10. Simplify the following:

[cos(2𝑥) − sin(2𝑥)][cos 2𝑥 + sin(2𝑥)]

Ans. cos 4x
11. Simplify cos 2x /(sin x + cos x). Ans. cos x – sin x
12. Find sin 87° cos 87° . Ans. 1/2 sin 174°
13. Reduced to simplified form: (cos x)( 4 sin x – 8 sin^3 x) Ans. sin 4x
14. If sin θ = 3/5 , and θ is in QII , find cot 2θ. Ans. -7/24
15. If cos x = 3/5 and 3π/2 < x < 2π , find tan (x/2). Ans. -1/2

SUM AND DIFFERENCE FORMULAS

EXAMPLES:

1. Rewrite the following:

tan 37° + tan 68°


1 − tan 37° tan 68°

SOLUTION:

2. Simplify cos (y – π) + sin (y + π / 2) .


SOLUTION:
EXERCISES – Answer the following questions.

1. Simplify : sin (x – 45) Ans.


2. Determine the simplest form of this expression:

3𝜋
csc ( + 𝐴)
2

Ans. – sec A
3. Find tan (x – y) if sin x = 8/17 and cos y = 3/5 . Ans. -36/77
4. Find the exact value:

5𝜋 𝜋
cos ( − )
4 6

−√6 − √2
𝐴𝑛𝑠.
4

5. Rewrite as a sum or difference: sin 2x sin 3x + cos 2x cos 3x. Ans. cos (2x – 3x)
6. In the given equation shown,
𝜋 𝜋
sin−1 (3𝑥 − 4𝑦) = ; cos −1 (𝑥 − 𝑦) =
2 3

Find the value of x. Ans. x = 1


7. Find sin (u – v) given that sin u = - 3/5 with π < u < 3π/2 and cos v = 12/13 with 0 < v < π/2. Ans. -16/65
8. Find the values of cos (a – b) given that sin a = 8/17 and tan b = 5/12 , both are in Q I. Ans. 220/221
9. Simplify sin (A + B) if sin A = 3/5 and cos B = 8/17 , both are acute angles. Ans. 84/85

10. Find the exact value of tan (α + β ) if sin α = -1/3 and cos β = - 1/4 and the terminal side of α lies in Q II. Ans.

PRODUCT TO SUM AND SUM TO PRODUCT FORMULAS


EXAMPLES:

1. Rewrite sin 7θ – sin 3θ as a product.


SOLUTION :

2. Rewrite as product to sum: 2 cos (7x/2) cos (3x/2)

3. Express as a sum or difference: sin 55 sin 40


SOLUTION:

EXERCISES – Answer the following questions.

1. Find the product to sum forn of cos 2x cos 3x. Ans. 1/2 cos x + 1/2 cos 5x
2. Express as the product: cos (8x) – cos (4x) Ans. – 2 sin 6x sin 2x
3. Simplify as a product: cos 3t + cos t . Ans. 2 cos 2t cos t
4. Simplify:

cos 4𝑡 − cos 2𝑡
sin 4𝑡 + sin 2𝑡

Ans. -tan t
5. Express as a sum or difference: 2/5 sin 6θ cos (-9θ) Ans. -1/5sin(3θ) + 1/5 sin 15θ
6. Simplify the following: 2 sin 45 cos 15. Ans. (√3 +1)/2
7. Evaluate in cosine form: sin 40° + sin 20° . Ans. cos 10°
8. Rewrite as a sum or difference: 16 sin 16x sin 11x Ans. 8 (cos 5x – cos 27x)
9. Simplify the following:

cos 6𝑥 + cos 2𝑥
sin 6𝑥 + sin 2𝑥

Ans. cot 4x
10. Convert the product cos 4x cos 3x to a sum. Ans. 1/2 [cos 7x+ cos x]
POWER REDUCING, NEGATIVE, CONFUNCTION AND MISCELLANEOUS FORMULAS
POWER REDUCING FORMULAS

NEGATIVE ANGLES FORMULAS

sin(−𝑥) = − sin 𝑥

cos(−𝑥) = cos 𝑥

tan(−𝑥) = − tan 𝑥

COFUNCTION IDENTITES

MISCELLANEOUS

EXAMPLES:

1. If tan θ = 1.3 , what is the value of cot (90° - θ) ?


SOLUTION:

2. Find the value of sin (-45°) .


SOLUTION:

−√2
sin(−𝑥) = − sin 𝑥 , sin(−45°) = − sin 45° = 𝐴𝑛𝑠.
2
3. Evaluate this expression:

1
sin 2𝑥 [1 − cos 4𝑥]
2

SOLUTION:

EXERCISES – Answer the following questions

1. If cos x = 4/5 , find versine. Ans. 1/5


2. Determine the cofunction of sin θ = 3/5 . Ans. cos θ = 3/5
3. Determine the value of tan (-45°) . Ans. -1
4. Evaluate the following:
𝜋
tan ( − 𝑥) sin 𝑥
2

Ans. cos x
5. Write this function in terms of cofunction: tan x . Ans cot (90 – x)
6. Determine the value of csc (-150° ) . Ans. -2
7. Find the coversine θ if tan θ = 5/12 and in Q III. Ans. -8/13
8. Find the cofunction of tan 3x. Ans. cot (90 – 3x)
9. Evaluate the following:

csc(90° − 𝑥) csc 𝑥

Ans. 2 csc 2x
10. Find sin^ 4x if cos 2x = 12/13. Ans. 1/676

TOPICS TO BE DISCUSSED ON OCTOBER 16, 2020:

1. Trigonometric Equations
2. Spherical Trigonometry
REFERENCES

1. College Algebra with Trigonometry by Barnett


2. https://www.varsitytutors.com/hotmath/hotmath_help/topics/basic-trigonometric-identities
3. Glencoe’s Advanced Mathematical Concepts
4. Engineering Mathematics by Bird
5. https://mathbitsnotebook.com/Algebra2/TrigConcepts/TCPythIden.html
6. MTAP Reviewers
7. https://mslc.osu.edu/sites/default/files/Trig%20Identities%20Handout.pdf
8. Engineering Mathematics by Besavilla
9. https://www.shmoop.com/unit-circle/negative-angles-examples.html
10. Precalculus Demystified
11. Precalculus by Blitzer
12. Schaum’s Outlines of Trigonometry
13. http://www.teaching.martahidegkuti.com/shared/lnotes/5_trig/identities/identities1/trigidentities1.pdf
14. Algebra and Trigonometry by Cynthia Young
15. https://www.cliffsnotes.com/study-guides/trigonometry/trigonometric-identities/double-angle-and-half-angle-identities
16. Algebra and Trigonometry by Openstax
17. https://www.mathworksheets4kids.com/trigonometry/identities/double-half-angle/double-angle-1.pdf
18. https://www.onlinemathlearning.com/double-angle-formula.html
19. http://mrcobb.net/wp-content/uploads/2015/01/8.5-Half-Angle-Formulas-MAC-1114-Trigonometry.pdf
20. https://www.classzone.com/eservices/home/pdf/teacher/LA214GAD.pdf
21. https://www.classzone.com/eservices/home/pdf/student/LA214FAD.pdf
22. Schaum’s Outlines of College Mathematics
23. https://mida39.files.wordpress.com/2008/11/worksheet-2-sum-and-difference-formulas.pdf
24. http://www.nabla.hr/TF-TrigFunctionsC5.htm
25. https://vignonoussa.files.wordpress.com/2010/08/product-to-sum-and-sum-to-product.pdf
26. https://s3.amazonaws.com/precalculus-worksheets/trig-precalc-vol2-worksheets/Trig+and+PreCalculus+Tutor+-+Vol+2+-+Worksheet+14+-
+Product-to-Sum+Trig+Identities.pdf
27. https://precalculusfinal.weebly.com/17-power-reducing-
identities.html#:~:text=Power%20reduction%20formulas%20can%20be,or%20cos%20%5E2%20a%20).
28. https://blogs.scientificamerican.com/roots-of-unity/10-secret-trig-functions-your-math-teachers-never-taught-you/
29. https://www.softschools.com/math/trigonometry/power_reduction_formula/
30.
PART 3:
ANALYTIC GEOMETRY
CIVIL ENGINEERING BOARD EXAMS PROBLEMS PHILIPPINES – OCTOBER 19, 2020

ANALYTIC GEOMETRY - known as coordinate geometry or Cartesian geometry, is the study of geometry using a coordinate system.

Analytic geometry is used in physics and engineering, and also in aviation, rocketry, space science, and spaceflight. It is the foundation of most modern fields
of geometry, including algebraic, differential, discrete and computational geometry.

Usually the Cartesian coordinate system is applied to manipulate equations for planes, straight lines, and squares, often in two and sometimes three dimensions.
Geometrically, one studies the Euclidean plane (two dimensions) and Euclidean space (three dimensions). As taught in school books, analytic geometry can be
explained more simply: it is concerned with defining and representing geometrical shapes in a numerical way and extracting numerical information from shapes'
numerical definitions and representations.

LINES – these are single dimension figure which indicates the length and determined by two points in a Cartesian coordinate plane.

CARTESIAN COORDINATE SYSTEM

DISTANCE BETWEEN TWO POINTS

EQUATIONS OF A LINE

𝐴𝑥 + 𝐵𝑦 + 𝐶 = 0 → 𝐺𝑒𝑛𝑒𝑟𝑎𝑙 𝑒𝑞𝑢𝑎𝑡𝑖𝑜𝑛

𝑦 − 𝑦1 = 𝑚(𝑥 − 𝑥1 ) → 𝑃𝑜𝑖𝑛𝑡 𝑠𝑙𝑜𝑝𝑒 𝑓𝑜𝑟𝑚 ; 𝑚 = 𝑠𝑙𝑜𝑝𝑒 𝑜𝑓 𝑡ℎ𝑒 𝑐𝑢𝑟𝑣𝑒


𝑥 𝑦
+ = 1 → 𝐼𝑛𝑡𝑒𝑟𝑐𝑒𝑝𝑡 𝑓𝑜𝑟𝑚 ; 𝑎 = 𝑥 − 𝑖𝑛𝑡𝑒𝑟𝑐𝑒𝑝𝑡 𝑎𝑛𝑑 𝑏 = 𝑦 − 𝑖𝑛𝑡𝑒𝑟𝑐𝑒𝑝𝑡
𝑎 𝑏
𝑦 − 𝑦1 𝑦2 − 𝑦1
= → 𝑇𝑤𝑜 𝑝𝑜𝑖𝑛𝑡 𝑓𝑜𝑟𝑚
𝑥 − 𝑥1 𝑥2 − 𝑥1

𝐴𝑥 + 𝐵𝑦 = 𝐶 → 𝑆𝑡𝑎𝑛𝑑𝑎𝑟𝑑 𝐹𝑜𝑟𝑚

𝑝 = 𝐴 cos 𝜃 + 𝐵 sin 𝜃 → 𝑃𝑜𝑙𝑎𝑟 𝑜𝑟 𝑁𝑜𝑟𝑚𝑎𝑙 𝐹𝑜𝑟𝑚

SLOPE OF THE LINE MIDPOINT OF A SEGMENT


DIVISION OF LINE SEGMENTS

DISTANCE FROM A POINT= TO A LINE.

DISTANCE BETWEET TWO PARALLEL LINES: RELATIONSHIP BETWEEN PERPENDICULAR AND PARALEL LINES:

ANGLE BETWEEN TWO LINES: AREA OF THE POLYGON USING COORDINATES (Shoelace Method):

Note: For the slope and the angles between curves, see Differential Calculus.
EXAMPLES:
1. Find the distance between two points (-3,2) and (3 , 5) .
SOLUTION:

2. Find the slope of the graph 3x + y + 1 = 0 .


SOLUTION:

m = 1/2 . Ans.

3. Find the slope of the line passing through ( -3, -1) and (-2 , 4) .
SOLUTION:

4. Find the midpoint of the line segment that joins points P(-3,3) and Q (1,0).
SOLUTION:
5. A point divides internally line segment joining (8,9) and (-7,4) in the ratio 2:3. Find the coordinates.
SOLUTION:

6. Find the distance between P (4,5) to a line 8x + 5y = 20


SOLUTION:

7. Find the angle between the slopes of 1/2 and 2.


SOLUTION:

8. Determine the kind of a triangle given the points A (1,3) , B (-1, 2) and C (5,3).
SOLUTION:
9. Assume the graph has 1 unit each grid, determine the line of the graph shown.

SOLUTION:

10. Find the area of the quadrilateral given the points (5,2) , (4,3) , (2,4) and (-8 , -1).
Using the formula:

1 𝑥1 𝑥2 𝑥3 𝑥4 𝑥1
𝐴= [ 𝑦1 ]
2 𝑦1 𝑦2 𝑦3 𝑦4

1 5 4 2 −8 5
𝐴= [ ]
2 2 3 4 −1 2

𝐴 = 18. 𝐴𝑛𝑠.

EXERCISES – Answer the following questions.

1. Find line in standard form given that P (0 ,9) and m = -2. Ans. 2x + y = 9
2. Find the area covered given the points (1,4) , (7,0) , (5 , -3) and (-1 , 1). Ans. 26
3. Find the equation of the line which cuts off an intercept 3 on the positive direction of x-axis and an intercept 5 on the negative direction of y-axis. Ans.
5x – 3y = 15
4. A and B are the vertices of the base of an isosceles triangle ABC. A is at point (1,2) and B at (4,1). Compute the area of the triangle given the ordinate
of 4. Ans. 10.5
5. Find the acute angle between the two lines 2x + 4y – 5 = 0 and 7x – 3y + 2 = 0 . Ans. 52.594°
6. Find the equation of the line parallel to 4x + 3y = 12 and passing through (−12, 4) Ans. 4x +3y + 36 = 0
7. Calculate the shortest distance between the parallel line y = 1/2 x and y = 1/2 x – 5 . Ans. 4.47
8. Write the equation of the line in slope intercept form if the line passes through (3,0) and perpendicular to the line y = 3x +1. Ans. y = -1/3 x + 3
9. (ECE Board)
10. Determine the coordinates of the point which is three fifths of the way from the point (2, -5) to the point (-3 , 5). Ans. (-1,1)
11. If the coordinates of 𝐴 and 𝐵 are (5, 5) and (−1, −4) respectively, find the coordinates of the point 𝐶 that divides 𝐴𝐵 internally in the ratio 2 : 1.
Ans. (1, -1)
12. Find the distance from the point (-6,8) to the line y = -3x +10. Ans. 6.3246
13. If M (1, 1) is the midpoint of the line segment joining A (3, 1) and B (x, y), find the coordinates of B. Ans. (5,3)
14. Find the distance between two lines 5x + 3y + 6 = 0 and 5x + 3y – 6 = 0. Ans. 12/√34
15. Given the vertices A (1,4) , B(3,6) , C(6,3) and D(4,1) , determine the perimeter. Ans. 14.14
16. Find the equation of line through point (3,2) and making angle 45° with the line x-2y = 3. Ans. 3x – y – 0 and x + 3y – 9 = 0 .
17. Determine the distance between the points (1.1 , 2.2) and (3.3 , 4.4) . Ans. 3.1113
18. Consider the points (-1,2) and (2, -3) , find the point which divides AB internally in the ratio 3:1. Ans. (5/4, -7/4)
19. Without graphing, find the gradient of the equation 3y = -6x +2. Ans. -2
20. Find the point whose coordinates are equal and equidistant from the points (-2 , 3) and (1,4) . Ans. (-0.5 , 0.5)
CIRCLES – sets of all points in a plane that are equidistant from a center or the fixed point. The fixed distance to any point on the circle is known to be radius.

CIRCLES IN ANALYTIC GEOMETRY

(h , k) - coordinates of the center of the circle.

STANDARD FORM OF THE CIRCLE

GENERAL FORM OF THE CIRCLE

In solving circles, the following cases should be met:


1. Three points given (general form)
2. Center (h,k) and the radius r. (standard form)

Using general form:

RADICAL AXIS

1. If the given circles intersect at two points, the radical axis passes through the intersection points. The radical axis is always perpendicular to the line
joining the center of the given circles.
2. If the given circles are tangent to each other, the radical axis is tangent to each other. The radical axis is always perpendicular to the line joining the
centers of the given circles.

3. If the given circles have no common point, the radical axis is between the given circles. The radical axis is always perpendicular to the line joining
the centers of a given circles.

Note: Recall the non linear systems of equations.

EXAMPLES:

1. (CE Board) A circle has the equation x^2 + y^2 – 6x + 12y + 9 = 0. Find the radius of the circle.
SOLUTION:

2. Rewrite in standard form: x^2 + y^2 + 4x – 6y – 23 = 0 .


SOLUTION:
3. Given the circles x^2 + y^2 – 4x – 6y = 3 and x2 + y2 + 6x +4y – 7 = 0 , find their intersections.

4. Find the equation of the circle that passes through the points (1 , -6) , (2,1) and (5,2).
EXERCISES – Answer the following questions.

1. Write the equation of the circle the have radius of 3√5 and center (-5, 4). Ans. (x + 5)2 + (y – 4)2 = 45
2. A square OABC is drawn with vertices as shown. Find the equation of the circle that can be drawn inside the square. Ans. x2 + y2 – 4y + 2 = 0.

3. Calculate the equation of the circle that passes through the point (0, −3), whose radius is √5 and whose center is on the angle bisector of the first
and third quadrants. Ans. x2 + y2 + 2x + 2y – 3 = 0 and x2 + y2 + 4x + 4y + 3 = 0
4. (ECE Board) The diameter of the circle described by 9x2 + 9y2 = 16 is ________. Ans. 8/3
5. Find the equation of the circle in standard form with radius 4 and center at C (-3 , 6). Ans. (x + 3)2 + (y – 6)2 = 16
6. The coordinates of three points on a circle are (0, 6), (6, 4) and (–8, 2). Determine the center of the circle. Ans. (0 , -4)
7. Calculate the radius of the circle given the equation x2 + y2 + 14x – 8y + 56 = 0 . Ans. r = 3
8. Calculate the equation of the circle in standard form given the center (0 , 13) and area of 25π. Ans. x2 + (y – 13)2 = 25
9. (CE Board) Find the area enclosed by the circle x2 - 10x + 4y + y2 = 196. Ans. 225π
10. The equation x2 + y2 – 6x + 4y = d describes a circle. Solve for d if the radius is 6 units. Ans. 23
11. Identify the area of the circle (x - 5)2 + (y + 2)2 = 4. Ans. 4π
12. The end of a diameter of the circle are (-3, 4) and (6,2). What are the coordinates of the center? Ans. (1.5 , 3)
13. Find the equation of the circle that passes through the origin and has its center at (-4,3) . Ans. (x + 4)2 + (y – 3)2 = 25
14. Find the equation of the circle in general form with (1,8) and (5, -6) as the endpoints of the diameter. Ans. x2 + y2 – 6x – 2y – 48 = 0
15. A circle passes through the point (5,7) and has its center (2,3) . Find its equation. Ans. (x – 2)2 + (y – 3)2 = 25

TOPICS ON OCTOBER 23, 2020:

1. Parabolas
2. Ellipses
3. Hyperbolas
REFERENCES

1. Analytic Geometry by Rainville


2. Engineering Mathematics by Gillesania
3. Engineering Mathematics by Bird
4. https://www.chilimath.com/lessons/intermediate-algebra/distance-formula/
5. Intermediate Algebra by Elayn Martin Guy
6. https://www.math-only-math.com/division-of-line-segment.html
7. https://www.tpub.com/math2/5.htm
8. Glencoe’s Advanced Mathematical Concepts
9. Scahum’s Outlines of Precalculus
10. https://calcworkshop.com/graphing-linear-equations/point-slope-form/
11. https://www.exeter.k12.pa.us/cms/lib6/PA01000700/Centricity/Domain/118/Day%209%20-%20Point%20Slope%20Homework%20complete.pdf
12. https://www.math-only-math.com/straight-line-in-intercept-form.html
13. Engineering Mathematics Vol. 1 by Besavilla
14. Analytic Geometry by AC Burdette
15. MMC Questionnaires
16. https://www.ck12.org/geometry/Distance-Between-Parallel-Lines/lesson/Distance-Between-Parallel-Lines-GEOM/
17. Algebra and Trigonometry by Cynthia Young
18. Engineering Mathematics by Excel Review Center
19. https://www.scevmath.org/uploads/2/8/9/4/28940543/3.4_distance_from_a_point_to_a_line.pdf
20. College Algebra with Trigonometry by Barnett
21. https://www.toppr.com/guides/maths/straight-lines/distance-of-point-from-a-line/
22. https://www.engageny.org/sites/default/files/downloadable-resources/2014/Aug/geometry-m4-topic-c-lesson-10-teacher.pdf
23. https://www.mathstopia.net/coordinate-geometry/angle-two-lines
24. https://www.cuemath.com/geometry/coordinate-geometry-internal-division/
25. https://www.math-only-math.com/circle-passing-through-three-given-points.html
26. http://www.nabla.hr/PC-CircleAndLineCnt3.htm
27. Introductory and Intermediate Algebra by Blitzer
28. Euclid Math Contest Questionnaires
29. https://www.superprof.co.uk/resources/academic/maths/analytical-geometry/conics/equation-of-a-circle-problems.html#chapter_exercise-12
30. 1,001 Geometry Problems
31. https://tutorial.math.lamar.edu/Solutions/Alg/Circles/Prob6.aspx
32. https://cdn.kutasoftware.com/Worksheets/Geo/11-Equations%20of%20Circles.pdf
33. https://mathforums.com/threads/difficult-question-equation-of-a-circle.180/
34. https://www.mathwarehouse.com/sheets/algebra/equation-of-circle-worksheet.php
CIVIL ENGINEERING BOARD EXAMS PROBLEMS PHILIPPINES - October 26, 2020

CONIC SECTIONS

CONIC SECTIONS - In mathematics, a conic section (or simply conic) is a curve obtained as the intersection of the surface of a cone with a plane. The three
types of conic section are the hyperbola, the parabola, and the ellipse; the circle is a special case of the ellipse, though historically it was sometimes called a
fourth type. The ancient Greek mathematicians studied conic sections, culminating around 200 BC with Apollonius of Perga's systematic work on their
properties.

Types of Conic Sections

1. Parabolas
2. Ellipses
3. Hyperbolas

PARABOLAS – set of all points in a plane that are equidistant from a fixed line (known as DIRECTRIX) and a fixed point (known as the FOCUS) that is not
on the line.

VERTEX – The uppermost , lowermost , leftmost or rightmost point of the parabola


AXIS OF SYMMETRY – line passing through the focus and perpendicular to the directrix.

LATUS RECTUM – chord passing through the focus and parallel to the directrix or perpendicular to axis.

GENERAL EQUATION OF A PARABOLA:

GENERAL FORM: STANDARD FORM:

Conditions in solving the parabola:


1. Three points along the parabola and an axis (either vertical or horizontal) - General Form
2. Vertex (h,k) , distance from the vertex to focus a and axis – Standard Form
3. Vertex (h,k) and location of focus – Standard Form

ECCENTRICITY – ratio of its distance from the focus and from the directrix. Parabola’s eccentricity is equal to 1.
EXAMPLES:
1. Find the vertex of the parabola x = y2 + 4y – 5 .
SOLUTION:

2. Write the equation of the directrix of the parabola y2 – 8x – 2y + 17 = 0


SOLUTION:
3. Determine the directrix of a parabola having the origin as its vertex, the y axis as its axis of symmetry and passes through the point (-10, -5).
SOLUTION:

4. The parabolic arch in the concrete bridge in the figure must have a clearance of 50 feet above the water and span a distance of 200 feet. Find the
equation of the parabola after inserting a coordinate system with the origin at the vertex of the parabola and the vertical axis (pointing upward)
along the axis of the parabola.
SOLUTION:

EXERCISES - Answer the following questions.

1. Consider the equation y2 = 8x +48. Find the axis of symmetry. Ans. y = 0


2. Find the focus of the parabola 16y + 11 = x^2 + 11. Ans. (0,4)
3. The equation of the parabola is x2 = 4y – 8 . Find the distance of the vertex from the point 3x + 4y + 2 = 0 . Ans. 2
4. Find the vertex of the function G(x) = (x – 3)2 . Ans. (3,0)
5. Determine the length of the latus rectum of the curve x2 + 8y = 0 . Ans. 8
6. Given the focus (7,0) and directrix x = -7, determine the equation of a parabola in standard form. Ans. y2 = 28x
7. Write in standard form: V(0,0) and directrix y = -1. Ans. x2 = 4y
8. A parabola has its vertex at the origin and the given is the focus (25,0). Find the length of the latus rectum. Ans. 100
9. If an automobile head light reflector is cut by a plane through its axis, the section is a parabola having the light as the focus. If the light is 18 mm
from the vertex, and the diameter of the light is 250 mm. Find the depth of the headlight. Ans. 217 mm
10. An arch in a memorial park, having a parabolic shape, has a height of 25 feet and a base width of 30 feet. Find an equation which models this
shape, using the x-axis to represent the ground. State the focus. Ans. (0 , 91/4)
ELLIPSES

An ellipse is the set of all points in a plane whose distances of two fixed points (foci) is constant . The midpoint of the segment connecting the foci
is the center of the ellipse.

EQUATIONS OF THE ELLIPSE

STANDARD FORM GENERAL FORM

ELEMENTS OF ELLIPSE:

Conditions in Solving an Ellipse:


1. Four points along the ellipse – general form
2. Center (h,k) , semi-major axis a and semi-minor axis b – standard form

Solving for the center of the ellipse in general form:


EXAMPLES:
1. Find the center of this ellipse:

(𝑥 − 1)2 (𝑦 + 2)2
+ =1
4 9

SOLUTION:

2. Find the eccentricity of the ellipse in the figure shown.

SOLUTION:

3. Of the nine planetary orbits in our solar system, Pluto’s has the greatest eccentricity, 0.248. Astronomers have determined that the orbit is about
29.646 AU (astronomical units) from the sun at its closest point to the sun (perihelion). The length of the semi-major axis is about 39.482 AU. 1
AU is the average distance between the sun and Earth, about 9.3 10^7 miles. Find the distance of Pluto from the sun at its farthest point.
SOLUTION:
EXERCISES – Answer the following questions.

1. Write the equation of an ellipse centered at the origin, with a =8, b = 5, and the major axis on the y-axis. Ans. x2/25 + y2/64 = 1
2. Evaluate the eccentricity of the equation 7x2 = 35 – 5y2 . Ans. sqrt (2/7)
3. Find the center of the ellipse x2 /9 + y2 /36 = 1 . Ans. (0,0)
4. The distance between the foci of an ellipse is 5. If its eccentricity is 0.5. what is the distance between the directrices? Ans. 20
5. The moon travels about Earth in an elliptical orbit with Earth at one focus, as shown. The major and minor axes of the orbit have lengths of 768,800
kilometers and 767,640 kilometers, respectively. Find the greatest and smallest distances (the apogee and perigee), respectively from Earth’s
center to the moon’s center. Ans. Greatest – 405,508 km. and Closest – 363,292 km.
6. (CE Board) The equation of the ellipse is 16x2 + 25y2 – 128x – 150y +381 = 0. Find the length of the minor axis. Ans. 4
7. What is the standard form equation of the ellipse that has vertices ( ± 8,0) and foci (± 5,0)? Ans. x2/64 + y2 /39 = 1
8. Find the focus of the ellipse 2x2 + y2 = 10. Ans. (0 , ± √5 )
9. An asteroid has elliptical orbit with the sun at one focus. Its distance from the sun ranges from 18 to 182 (in million miles.) Write and equation of
the orbit of the asteroid. Ans. x2 / 10000 + y2/ 3276 = 1
10. A longitudinal section (that includes the two vertices and the center) of an official Wilson NFL football is an ellipse. The longitudinal section is
approximately 11 inches long and 7 inches wide. Write an equation governing the elliptical longitudinal section. Ans. x2/5.52 + y2/3.52 = 1

HYPERBOLAS

A hyperbola is the set of all points in a plane, the difference of whose distances from two fixed points is a positive constant. These two fixed points
are called foci. The hyperbola has two separate curves called branches. The two points where the hyperbola intersects the line joining the foci are called
vertices. The line segment joining the vertices is called the transverse axis of the hyperbola. The midpoint of the transverse axis is called the center.

EQUATIONS OF HYPERBOLA

GENERAL FORM STANDARD FORM

For the hyperbola ∓ Ax2 ± Cy2 + Dx + Ey + F = 0:


ELEMENTS OF HYPERBOLA EQUATIONS OF ASYMPTOTES

EXAMPLES:
1. Find the y – intercepts of the hyperbola 4y2 – 9x2 = 36
SOLUTION:

2. What is the length of the latus rectum of the hyperbola x2/25 – y2 /64 = 1 ?
SOLUTION:

3. The design layout of a cooling tower is shown in the figure. The tower stands 179.6 meters tall. The diameter of the top is 72 meters. At their
closest, the sides of the tower are 60 meters apart. Find the equation of the hyperbola that models the sides of the cooling tower. Assume that the
center of the hyperbola—indicated by the intersection of dashed perpendicular lines in the figure—is the origin of the coordinate plane. Round final
values to four decimal places.
SOLUTION:

EXERCISES – Answer the following questions.


1. Identify the foci of the hyperbola y2/49 – x2/32 = 1 . Ans. (0,9)
2. Label the asymptote of the hyperbola 9x2 – 4y2 – 36x – 40y - 388 = 0 . Ans. y = ±3/2 (x – 2) – 5
3. Locate the center of the hyperbola (y + 4)2 /36 - (x – 2)2/25 = 1 . Ans. (2, -4)
4. Stations A and B are 150 kilometers apart and send a simultaneous radio signal to the ship. The signal from B arrives 0.0003 seconds before the
signal from A. If the signal travels 300,000 kilometers per second, find the equation of the hyperbola on which the ship is positioned.
Ans. x2/2025 – y2/3600 = 1
5. The length of the latus rectum of a hyperbola is equal to 18 and the distance between the foci is 12. Compute the equation of the asymptote of the
hyperbola. Ans. y = √3 x
6. Determine the vertices of the hyperbola (x + 7)2 /36 – (y + 4)2/64 = 1. Ans. ( - 13, -4) and ( - 1 , -4)
7. Find the equation of the hyperbola with the distance between foci having equal to 18 and between directrices having equal to 2. Ans.
8x2 – y2 = 72
8. Locate the foci of the hyperbola (x + 2)2/9 – (y – 5)2/49 = 1. Ans. (-2 , ± √58)
9. (CE Board) The equilateral hyperbola xy = 8 and has the x-axis and y-axis as asymptote. Compute the eccentricity of the hyperbola. Ans. 1.414
10. Cross section of a Nuclear cooling tower is in the shape of a hyperbola with equation (x^2/302) - (y^2/442) = 1 . The tower is 150 m tall and the
distance from the top of the tower to the centre of the hyperbola is half the distance from the base of the tower to the centre of the hyperbola. Find
the diameter of the top and base of the tower. Ans. 45.41 and 74.45 m

NEXT TOPICS ON October 30, 2020:

1. Determination of Conics, Tangent and Normal to Conics


2. Rotation and Translation of Axes
3. Parametric Equations
REFERENCES:

1. Introductory and Intermediate Algebra by Blitzer


2. Engineering Mathematics by Gillesania
3. https://en.wikipedia.org/wiki/Conic_section
4. https://www.onlinemath4all.com/finding-the-vertex-focus-directrix-and-latus-rectum-of-the-parabola.html
5. https://www.algebra.com/algebra/homework/Quadratic-relations-and-conic-sections/Quadratic-relations-and-conic-
sections.faq.question.856050.html
6. Algebra and Trigonometry by Barnett
7. Glencoe’s Advanced Mathematical Concepts
8. https://www.easyteacherworksheets.com/pages/pdf/math/geometry/parabolas/54.html
9. Engineering Mathematics by Besavilla
10. Intermediate Algebra by Elayn Martin
11. Analytic Geometry by Rainville
12. https://mi01000971.schoolwires.net/cms/lib/MI01000971/Centricity/Domain/433/Parabolas%20WS%20D1.pdf
13. Schaum’s Outlines of College Mathematics
14. https://www.purplemath.com/modules/parabola3.htm
15. https://www.mathwarehouse.com/ellipse/eccentricity-of-ellipse.php#problem4
16. https://www.cbsd.org/cms/lib/PA01916442/Centricity/Domain/2023/333202_1003_744-752.pdf
17. Algebra and Trigonometry by Openstax
18. https://www.emathzone.com/tutorials/geometry/applications-of-ellipse.html
19. Algebra and Trigonometry by Cynthia Young
20. https://www.mathopolis.com/questions/q.html?id=838&t=mif&qs=835_3336_836_3337_837_3338_838_3339_9068_9069&site=1&ref=2f67656f
6d657472792f6879706572626f6c612e68746d6c&title=4879706572626f6c61
21. http://www.opentextbookstore.com/precalc/2/Precalc9-2.pdf
22. Precalculus Demystified
23. Analytic Geometry by Rainville
24. http://www.phschool.com/atschool/ap_misc/dwfk_precalc/pdfs/8e/Ch8_Section3.pdf
25.
CIVIL ENGINEERING BOARD EXAMS PROBLEMS PHILIPPINES – October 30, 2020

IDENTIFICATION OF CONIC SECTIONS AND NORMAL AND TANGENT TO CONICS

GENERAL EQUATION OF THE CONICS

IDENTIFYING CONIC SECTIONS BY ECCENTRICITY (e) :


1. Circles – e = 0
2. Parabolas - e = 1
3. Ellipses – 0 < e < 1
4. Hyperbolas – e > 1

TANGENTS AND NORMAL TO CONICS

Unlike other curves, the tangent to any conic will pass only through one point. The following substitutions to be used in solving the tangents and normal to
conics:

For other functions, you may refer to differential calculus. Sub normal – means the distance from point of tangency to x-axis.

Cases:

1. Finding the equation of the tangent given the point on the conic – replace variables in the conic and substitute for (x`1 , y1) the given point.
2. Finding the equation of the tangent that passes through a given point outside the conic – apply necessary replacements of variables leaving x1 and
y1 unknown. Another equation relating x1 and y1 can be found by substituting (x1 , y1) to the equation of the conic. By expressing y1 in terms of x1 in
either equation and substituting the other equation, a quadratic equation is derived in the form Ax 1 + Bx1 + C = 0. With (x1, y1) known, the tangent is
solvable.
3. Finding the equation of the tangent given the slope m of the tangent – use slope intercept form for the tangent line with m and b unknown. Since
the line and the conic crosses, substitute this value of y to the y value in the conics resulting to quadratic equation. Since tangent passes through
one distinct point, solve using the discriminant formula. With b known and m given, the tangent can now be solved.
Note: For the steps on the other cases in non-conic sections, refer to differential calculus.

EXAMPLES

1. Identify the conic section 6y2 + 3x – 4y – 12 = 0


SOLUTION:

2. Identify the conic section 2x2 + 5y2 – 8x + 25y + 115 = 0


SOLUTION:

3. Find an equation of the circle with center (1, -5) and tangent to the line 3x + 4y = 8 .
SOLUTION:
4. Find the equations of tangent and normal to the parabola x2 + 6x + 4y + 5 = 0 at (1, -3)
SOLUTION:

EXERCISES – Answer the following questions.


1. Determine the conic section 2x2 + 2y2 = 10. Ans. Circle
2. A curve has an equation of x^2 = 16y . Compute the equation of the tangent at (4,1) . Ans. x = 2y + 2
3. Find the tangent line of the parabola y = x2 + 2x at (-3,3) . Ans. 4x +y + 9 = 0
4. Identify the conic section 4x2 + y2 + 2y – 15 = 0. Ans. Ellipse
5. Identify the conic section y2/20 – (x+5)2/5 = 1. Ans. Hyperbola
6. What is the conic section of the curve 4x2 + 6y2 = 36. Ans. Ellipse
7. Find the normal line of the curve y = x2 – x – 6. Ans. 5y = -x + 3
8. What conic section is 4x2 = 25y2 + 100? Ans. Hyperbola
9. Classify this conic section: x2 + 2y2 + 4x + 2y – 27 = 0 . Ans. Ellipse
10. Determine the equation of the tangent of the circle x2 + 16y = 32 – 4x – y2. Ans. 3x + 4y – 12 = 0

ROATION AND TRANSLATION OF AXES AND DIAMETER OF CONICS

TRANSLATION AND ROTATION OF AXES

Translation of axes occurs when the new coordinate axes have the same direction as and are parallel to the original coordinate axes.

Rotation of axes occurs when the origin is kept fixed and the x’ and y’ axes are obtained by rotating the x and y axes counterclockwise through an angle θ
DIAMETER OF CONICS

Diameter – locus of the midpoints of a system of parallel chords.

PARABOLA

ELLIPSES

HYPERBOLAS
*Conjugate diameter – two diameters of an ellipse or hyperbola are conjugate if each conic bisects the chords parallel to the other.

Polar and pole – If tangents AB and AC are drawn tangent to a conic, from A (x1, y1) external to the conic , then the line through the points of tangency B and
C is called the POLAR of the point A with respect to the conic. Conversely, if a line is drawn cutting the conic B and C and tangents constructed at these points
intersects at A, then A is called the POLE of the with the respect to the conic.

EQUATIONS OF THE POLAR

A. ELLIPSE
B. PARABOLAS

C. HYPERBOLAS

EXAMPLES

1. Transform xy = -2 using rotation of axes through 45° .


SOLUTION:

2. A circle (x – 4)2 + (y + 1)2 = 36 is translated to (4, -1) , find the equation of the curved in the translated system.
SOLUTION:
3. (CE Board) A parabola has an equation y2 = 8x. Find the equation of the diameter of parabola which bisect chords parallel to the line x – y = 4.
SOLUTION:

4. The equation of the ellipse is given as 16x2 + 36y2 = 576. Find the equation of the polar of the given point (4, -6) with respect to the ellipse.
SOLUTION:

EXERCISES - Answer the following questions.


1. Write the equation xy = 1 in terms of rotated system when θ = 45° . Ans. x’2 - y’2 = 2
2. Given is the polar 4x + y + 12 = 0 of the parabola y2 = 4x , find the coordinates of the pole. Ans. (3 , 1/2 )
3. An ellipse has an equation equal to 9x2 + 25y2 = 225. Determine the equation of the polar of the point (2,-3) with respect to the ellipse.
Ans. 6x – 25y = 225
4. Transform this conic into standard translated form: y2 – 6y – 4x + 1 = 0 . Ans. y’2 = 4x’
5. Transform this conic into standard rotated form: x2 + y = 0 when θ = 60° . Ans. x’2 - 2√3 x’y’ + 3y’2 + 2√3 x’ + 2y’ = 0
6. Write an equation of the translated conic in general form: x2 + y2 = 7 at (3,2) . Ans. x’2 + y’2 - 6x’ – 4y’ + 6 = 0
7. A parabola has an equation y2 = 8x. Find the equation of the tangent to the parabola having a slope parallel to line x – y = 4. Ans. x – y + 2 = 0
8. Find a new representation of the equation 2x2 – xy + 2y2 – 30 = 0 when it is rotated to 45° . Ans. x’2 /20 + y’2 = 1

PARAMETRIC EQUATIONS

The variable t is called a parameter and does not appear on the graph. Equations are called parametric equations because both x and y are expressed in terms
of the parameter t. Eliminating the parameter is the most common method in solving parametric equations.

EXAMPLES:

1
1. 𝑥 = √𝑡 𝑎𝑛𝑑 𝑦 = 𝑡 + 1
2
SOLUTION:
2. 𝑥(𝑡) = √𝑡 + 2 , 𝑦(𝑡) = log 𝑡
SOLUTION:

EXERCISES - Eliminate the parameters on the following equations. If trigonometric equations are given, use the interval between 0 and 2π .

3
1. 𝑥 = 3𝑡 2 − 4 , 𝑦 = 2𝑡 𝐴𝑛𝑠. 𝑥 = 𝑦 2 − 4 5. 𝑥 = 𝑡 , 𝑦 = 𝑡 2 𝐴𝑛𝑠. 𝑥 2 = 𝑦
4

2. 𝑥(𝑡) = 𝑡 2 + 1 , 𝑦(𝑡) = 2 + 𝑡 𝐴𝑛𝑠. 𝑥 = 𝑦 2 − 4𝑦 + 5 6. 𝑥(𝑡) = 4 cos 𝑡 , 𝑦(𝑡) = 3 sin 𝑡 𝐴𝑛𝑠. 9𝑥 2 + 16𝑦 2 = 144

𝑥 2 𝑦2
3. 𝑥 = 3 sin 𝑡 , 𝑦 = −4 cos 𝑡 𝐴𝑛𝑠. + =1 7. 𝑥(𝑡) = 𝑒 −𝑡 , 𝑦(𝑡) = 3𝑒 𝑡 𝐴𝑛𝑠. 𝑥𝑦 = 3
9 16

4. 𝑥 = tan 3𝑡 , 𝑦 = 2 + sec 3𝑡 𝐻𝑖𝑛𝑡: 𝑈𝑠𝑒 𝑃𝑦ℎ𝑡𝑎𝑔𝑜𝑟𝑒𝑎𝑛 𝑖𝑑𝑒𝑛𝑡𝑖𝑡𝑖𝑒𝑠. 𝐴𝑛𝑠. 𝑥 2 + 1 = (𝑦 − 2)2

Next (Final) Topics on November 6, 2020:

1. Polar Coordinates and Conversion of Rectangular to Polar and Vice Versa


2. Solid Analytic Geometry
REFERENCES:

1. Engineering Mathematics by GIllesania


2. https://math.libretexts.org/Bookshelves/Calculus/Book%3A_Calculus_(OpenStax)/11%3A_Parametric_Equations_and_Polar_Coordinates/11.5%
3A_Conic_Sections
3. https://www.ck12.org/c/analysis/classifying-conic-sections/lesson/Classifying-Conic-Sections-ALG-II/
4. https://www.purplemath.com/modules/circle3.htm
5. http://www.brainkart.com/article/Tangents-and-Normals-to-Conics_39171/
6. Algebra and Trigonometry by Cynthia Young
7. Algebra with Trigonometry by Barnett
8. https://www.lavc.edu/math/library/math125/Worksheets/classifyingconics2.pdf
9. Engineering Mathematics
10. https://flexbooks.ck12.org/cbook/ck-12-precalculus-concepts-2.0/section/9.1/primary/lesson/general-form-of-a-conic-pcalc
11. Analytic Geometry by Rainville
12. https://rancho-hs.enschool.org/ourpages/auto/2013/6/3/28343346/4S%20WS%2011.pdf
13. https://iblog.dearbornschools.org/singley/wp-content/uploads/sites/781/2018/05/translated-conics-resource.pdf
14. https://www.mathalino.com/tag/reviewer/diameter-parabola
15. https://fl01000126.schoolwires.net/cms/lib/FL01000126/Centricity/Domain/261/Ch9_Section4.pdf
16. https://www.toppr.com/ask/question/given-is-the-polar-4xy120-of-the-parabola-y24x-find-the-coordinates-of-the-pole/
17. https://cdn.kutasoftware.com/Worksheets/Precalc/10%20-%20Rotations%20of%20Conic%20Sections.pdf
18. Glencoe’s Advanced Mathematical Concepts
19. https://courses.lumenlearning.com/precalctwo/chapter/rotation-of-axes/
20. Precalculus by Blitzer
21. https://courses.lumenlearning.com/suny-osalgebratrig/chapter/parametric-equations/
22. https://people.richland.edu/james/lecture/m116/conics/parametric.html
23. https://math.libretexts.org/Bookshelves/Algebra/Book%3A_Algebra_and_Trigonometry_(OpenStax)/10%3A_Further_Applications_of_Trigonometr
y/10.07%3A_Parametric_Equations
24. https://tutorial.math.lamar.edu/problems/calcii/parametriceqn.aspx
25. Analytic Geometry by Burdette
26. https://openstax.org/books/precalculus/pages/8-6-parametric-equations
27. Algebra and Trigonometry by Openstax
CIVIL ENGINEERING BOARD EXAMS PROBLEMS PHILIPPINES – November 6, 2020

POLAR COORDINATES AND EQUATIONS

To form a polar coordinate system, start with a fixed point and call it the pole or origin. From this point draw a half line, or ray (usually horizontal and to the
right) and call this line the polar axis. In this system, the location of a point is expressed by its distance r from a fixed point and its angle from a fixed line.

Sign convention

1. θ is positive for counterclockwise and negative for clockwise.


2. r is positive for laid offs at terminal side and negative for laid offs at prologation through O from the terminal side θ .

DISTANCE BETWEEN TWO POINTS

CONVERSION OF RECTANGULAR TO POLAR COORDINATES AND VICE VERSA.

POLAR CURVES AND RECTANGULAR CURVES


EXAMPLES:
1. Convert (-4 , 1.077) to rectangular coordinates. (1.077 radians)
SOLUTION:

2. Rewrite to rectangular form: r = 3/(1 – 2cos θ)


SOLUTION:
3. Convert ( -1 , √3) to polar coordinates.
SOLUTION:

4. Find the polar equation of the circle whose rectangular equation is x2 + y2 – 8x + 6y – 2 = 0 .


SOLUTION:

5. Transform to polar equation: xy = 5


SOLUTION:

𝑥 = cos 𝜃 , 𝑦 = sin 𝜃

1
cos 𝜃 sin 𝜃 = 5(1) ; sin 2𝜃 = 5(sin2 𝜃 + cos2 𝜃)
2

sin 2𝜃 = 10 sin2 𝜃 + 10 cos2 𝜃 → 𝐴𝑛𝑠.

6. Convert to rectangular coordinates: (-5 , 90° )


SOLUTION :

EXERCISES – Answer the following questions.

1. Transform to rectangular form: r2sin 2θ = 6. Ans. xy = 3


2. Transform into polar coordinates: (2, 2). Ans. (2√2 , π/4)
3. Convert to polar form: (x +1)2 + y2 = 1 Ans. r = - 2cos θ
4. Find the polar coordinates of R( - 8, -12) . Ans. (14.42 , 4.12)
5. Write the polar equation of r = 2 csc θ . Ans. y = 2
6. The polar equation of an ellipse is equal to r2(4 sin2 θ + 9 cos2 θ) = 36. Compute its area. Hint (A = πab) Ans. 6π
7. A surveyor identifies a landmark at the point with polar coordinates (325, 70°) . Find the rectangular coordinates. Ans. (111.157 , 305.4)
8. Find the distance between (9, -45°) and ( -4 , 70°) . Ans. 8.16
9. Convert to rectangular form: r(sin θ – 3 cos θ) = 2. Ans. 3x = y – 2
10. Convert to polar form : x2 + y2 = 25. Ans. r = ± 5
11. Convert to rectangular form: r = 4cos θ. Ans. x2 - 4x + y2 = 0
12. A surveyor mapping out the land where a new housing development will be built identifies a landmark 𝟐𝟐𝟑 feet away and 𝟒𝟓° left of center. A
second landmark is 𝟒𝟏𝟖 feet away and 𝟕𝟎° right of center. Find the distance between two points. Ans. 550 feet
13. Find the rectangular form of r = 4. Ans. x2 + y2 = 16
14. Rewrite the Cartesian equation x2 + y2 = 6y as a polar equation. Ans. r = 6 sin θ
15. Convert ( -4 , 2π/3) into Cartesian Coordinates. Ans. (2 , -2√3 )
SOLID ANALYTIC GEOMETRY

SPACE COORDINATE SYSTEM – There three coordinate system used in solid analytic geometry:

1. RECTANGULAR COORDINATES – a point P(x , y , z) in space is fixed by its three distance x, y, and z from the coordinate planes.

2. CYLINDRICAL COORDINATES – A point P in space may be imagined as being on the surface of a cylinder perpendicular to the XY- plane. P(r, θ
,z) is fixed by its distance z from the xy- plane and by the polar coordinates (r , θ) of the projection of P on the XY-plane.

3. SPHERICAL COORDINATES – A point P in space may be imagined as being on the surface of a sphere with center at the origin O and radius r.
P(r, ϕ , θ) is fixed by its distance r from O, the angle ϕ between OP and z-axis , and the angle θ which is the angle between the x axis and the
projection OP on the xy- plane.

DISTANCE BETWEEN TWO POINTS


CONVERSION FACTORS : FROM RECTANGULAR AND VICE VERSA.

TO CYLINDRICAL : TO SPHERICAL:

DISTANCE FROM A POINT TO A PLANE: GENERAL EQUATION OF THE PLANE

EQUATION OF THE PLANE (INTERCEPT FORM) ANGLE BETWEEN TWO PLANES

PERPENDICULAR DISTANCE BETWEEN TWO PLANES

POINT OF INTERSECTION OF THE MEDIANS


SPACE COORDINATE SYSTEM COSINE FORMULAS (More useful when we take three dimensional forces in Engineering Mechanics)

SOLID ANALYTIC GEOMETRY GRAPHS

EXAMPLES
1. Calculate the distance between A (0,2,0) and B(7,2,-1) .
SOLUTION:
2. What is the distance between the point P(1,2,3) and the plane 2x + 2y – 3y + 3 = 0?
SOLUTION:

3. Convert to rectangular coordinates: (4 , 2π/3 , -2)


SOLUTION:

4. Determine the angles of the radius vector of the point (3,-2,5) that forms with the coordinate axes.
SOLUTION:

5. Find the direction cosines of a point having a coordinates of (2, 3, -6).


SOLUTION:

6. Convert the point (-1 , 1, -√2) to spherical coordinates.


SOLUTION:
EXERCISES – Answer the following questions.

1. Convert from cylindrical to spherical coordinates: (1, π/2 , 1) (Hint: Convert first to rectangular form) Ans. (√2 , π/2 , π/4)
2. Find the distance from a point (4, -4, 3) to the plane 2x – 2y + 5z + 8 = 0. Ans. 6.8
3. If 1/2 , 1/√2 , cos γ are the direction cosines of the vector, find γ . Ans. No solution
4. A given sphere has the equation x2 + y2 + z2 + 4x – 6y – 10z + 13 = 0 . Find the radius. Ans. 5
5. Determine the direction cosines of the normal to the plane x + y + z = 1. Ans. cos α = cos β = cos γ = 1/√3
6. Calculate the distance of the planes 2x – y – 2z + 5 = 0 and 4x – 2y – 2z + 15 = 0. Ans. 0.833
7. Find the direction cosines of the line passing through points ( -2, 4, -5) and (1, 2, 3). Ans. cos α = 3/√77 , cos β = -2/√77 , cos γ = 8/√77
8. The vertices of a triangle are (1,1,0) , (1, 0 , 1) and ( 0, 1, 1) . Find the point of intersection of the medians of the triangle. Ans. (2/3 , 2/3, 2/3)
9. Find the midpoint of the points (5, 12, 10) and (3, 0 , -1). Ans. (4,6,4.5)
10. Find the angle between two planes x – 2y +z = 0 and 2x + 3y – 2z = 0. Ans. 126.448º
11. Find the distance between the given plane 2x + 4y – 4x – 6 = 0 and P(0,3,6). Ans. 3
12. Convert (1, -1 , -√2) to spherical form. Ans. (2, 7π/4 , 3π/4)
13. Find the equation of the sphere x2 + y2 + z2 = 4 in cylindrical form. Ans. r2 + z2 = 4
14. Determine the cos γ for the point (2 , 3 , 4) . Ans. 4/√29
15. Calculate the distance between points (6, 11, 3) and (4, 6, 12). Ans. 10.5

END OF ANALYTIC GEOMETRY

Next Topics on November 9, 2020 – Solid Mensuration

1. Squares and Rectangles


2. Parallelograms , Trapezoids and Other Quadrilaterals
REFERENCES

1. College Algebra with Trigonometry by Barnett


2. Engineering Mathematics by Gillesania
3. Algebra and Trigonometry by Openstax
4. College Algebra and Trigonometry by Cynthia Young
5. Analytic Geomtry by Burdette
6. https://www.lcps.org/cms/lib/VA01000195/Centricity/Domain/10387/8.1%20Worksheet%20Polar%20Coordinates%20KEY.pdf
7. Glencoe’s Advanced Mathematical Concepts
8. Engineering Mathematics Vol. 1 by Besavilla
9. https://www.ck12.org/book/ck-12-trigonometry-concepts/section/6.2/
10. https://math.libretexts.org/Bookshelves/Algebra/Book%3A_Algebra_and_Trigonometry_(OpenStax)/10%3A_Further_Applications_of_Trigonometr
y/10.05%3A_Polar_Coordinates_-_Graphs
11. https://precalculuscoach.com/wp-content/uploads/2017/11/9-1-Word-Problems-Polar-Coordinates.pdf
12. https://tutorial.math.lamar.edu/classes/calcii/polarcoordinates.aspx
13. Analytic Geometry by Rainville
14. https://www.sangakoo.com/en/unit/distance-between-two-points-in-space
15. http://mathonline.wikidot.com/the-distance-between-a-plane-and-a-point
16. https://math.libretexts.org/Bookshelves/Calculus/Book%3A_Calculus_(OpenStax)/12%3A_Vectors_in_Space/12.7%3A_Cylindrical_and_Spherica
l_Coordinates
17. https://tutorial.math.lamar.edu/classes/calciii/CylindricalCoords.aspx
18. https://tutorial.math.lamar.edu/classes/calciii/SphericalCoords.aspx
19. https://www.math.utah.edu/lectures/math2210/9PostNotes.pdf
20. https://www.onlinemath4all.com/direction-cosines-word-problems-in-vectors.html
21. https://www.superprof.co.uk/resources/academic/maths/analytical-geometry/distance/distance-between-a-point-and-a-plane.html
22. https://www.teachoo.com/3559/756/Example-3----Find-direction-cosines-of-line-passing-through/category/Examples/
23. https://math.stackexchange.com/questions/1528909/find-the-angle-between-two-planes-using-their-normal-vectors
24. https://mosaic.math.tamu.edu/~glahodny/Math251/Section%2013.9.pdf
25. https://www.whitman.edu/mathematics/calculus_late_online/section14.06.html
26. https://slideplayer.com/slide/13460064/
27.
PART 4:
PLANE AND SOLID
GEOMETRY
CIVIL ENGINEERING BOARD EXAM PROBLEMS PHILIPPPINES – November 9, 2020

SOLID MENSURATION

Solid mensuration is another name for solid geometry, which is a field in mathematics where measurements of three-dimensional shapes are studied. Length,
width and height are the three most important measurements in solid mensuration. With those three values, many other values, such as surface area and
volume, can be determined.

PLANE FIGURES (Two Dimensional) - Triangles, Squares, Polygons, Circles, Ellipses, Quadrilaterals
SOLID FIGURES (Three Dimensional) – Prisms, Parallelpipeds , Pyramids, Cones, Cylinders, , Spheres

Note: For the Triangle topics, see trigonometry .

SQUARES AND RECTANGLES

SQUARES – quadrilaterals with four equal sides


RECTANGLES – quadrilaterals with a pair of equal parallel sides perpendicular to other pair.

AREAS AND PERIMETERS:

EXAMPLES:
1. Length of a side of a square field is 275 m. What will be cost of levelling the field at a rate of 10 cent per square metre?
SOLUTION:

2. The official distance between home plate and second base in a baseball diamond is 120 ft. Find the area of the official ball diamond and the distances
between the bases. (The official ball diamond is in the form of a square.)
SOLUTION:
3. A piece of wire is enclosed in a shape of a square whose area is 169 sq cm. it is then reshaped to enclose a rectangle whose length is 15 cm. find
the area of the rectangle.
SOLUTION:

4. An American football field is a rectangle with a perimeter of 1040 feet. The length is 200 feet more than the width. Find the area of the rectangular
field.
SOLUTION:

Let 200 + w = length.

𝑃 = 2𝑙 + 2𝑤 , 1040 = 2𝑙 + 2𝑤 ; 1040 = 2(200 + 𝑤) + 2𝑤 ; 𝑤 = 160 𝑓𝑡

𝑙 = 160 + 200 = 360 , 𝑡ℎ𝑒𝑛; 𝐴 = 𝑙𝑤 = 360 ∗ 160 = 57,600 𝑓𝑡 2 . → 𝐴𝑛𝑠.

5. A contractor is going to re-tile a rectangular section of the kitchen floor. If the floor is 6ft x 3ft, and he is going to use square tiles with a side of 9in.
How many tiles will be needed?
SOLUTION:

6. The length of a rectangle is 8 cm more than its width. If the length is decreased by 9 and the width is tripled, the area is increased by 50%. What was
the area of the original rectangle?
SOLUTION:

EXERCISES - Solve the following problems.


1. If the perimeter of the squares are in the ratio 2:5 , find the ratio of the areas. Ans. 4:25
2. Two frames are needed with the same perimeter: one frame in the shape of a square and one in the shape of a regular pentagon. Each side of the
square is 7 inches longer than each side of the pentagon. Find the area of the glass frame that can be fit if the wood framing needs to be 2 inches
width. Ans. 961 in2
3. The perimeter of a rectangular concrete slab is 82 feet and its area is 364 square feet. What is the length of the longer side of the slab? Ans. 28 ft
4. The length of a rectangular garden is 4 m longer than the width. If the perimeter is 52 m, what is the area? Ans. 165 m.
5. Find the area of the figure shown. Ans. 164 m2

6. A rectangular open-topped box is to be constructed out of 20-inch-square sheets of thin cardboard by cutting x-inch squares out of each corner and
bending the sides up. After that procedure, the area of the cardboard of the bottom section is 256 in2 . Find the size of the corners removed. Ans. 2
inches
7. The length of a rectangle is 6 cm and the width is 4 cm. If the length is greater by 2 cm, what should the width be so that the new rectangle have the
same area as the first one? Ans. 3 cm
8. The length of a rectangular flower garden is 6 feet more than its width. A walkway 3 feet wide surrounds the outside of the garden. The total area of
the walkway itself is 288 square feet. Find the dimensions of the garden. Ans. 18 x 24 feet
9. The perimeter of a rectangular outdoor patio is 54 ft. The length is 3 ft greater than the width. Find its area. Ans. 180 ft3
10. Find the diagonal of a rectangle with an area of 100 square yards and length of 10 yards. Ans. 10√2 yards
11. A 50 m cable is divided into two parts and formed into squares. If the sum of the areas is 100 m2 , find the difference in length. Ans. 24.5 m.
12. A gardener has a 30 foot by 20 foot rectangular plot of ground. She wants to build a brick walkway of uniform width on the border of the plot. If the
gardener wants to have 400 square feet of ground left for planting, how wide (to two decimal places) should she build the walkway? Ans. 5 feet
13. A baseball diamond is a square defined by home plate and the three bases. The sides of the square each measure 90 feet. What is the distance from
the pitcher's mound (in the center of the square) to home plate to the nearest foot? Ans. 45√2
14. A 1200 square meter large rectangular garden is enclosed with 150 meters of fencing. Find the dimensions of the garden Ans. 23.139 x 51.861 m
15. The figure shows a square floor plan with a smaller square area that will accommodate a combination fountain and pool. The floor with the fountain-
pool area removed has an area of 21 square meters and a perimeter of 24 meters. Find the dimensions of the square that will accommodate the
pool. Ans. 5 meter square

PARALLELOGRAMS, RHOMBUSES, AND OTHER QUADRILATERALS

PARALLELOGRAMS – quadrilateral with two pairs of parallel sides . These parallel sides are equal; diagonals bisected to each other.
RHOMBUS - quadrilateral with four congruent sides having two pairs of parallel sides; diagonals bisected perpendicularly.
TRAPEZOID – quadrilateral with one pair of parallel sides.
*Isosceles Trapezoid – trapezoid with equal non parallel sides.
KITE - a quadrilateral whose four sides can be grouped into two pairs of equal-length sides that are adjacent to each other.
DART – a concave dart
QUADRILATERAL – polygon with four sides
CYCLIC QUADRILATERAL - quadrilateral whose vertices lie on the circumference of a circle.
PTOLEY’S THEOREM – product of diagonals equals the sum of the products of the opposite sides
EXAMPLES:

1. The research department of a flooring company is considering a new flooring design that contains parallelograms. The area of a parallelogram with
base b and height h is bh. Find the area of a parallelogram with base 10 centimeters and height 8.2 centimeters.
SOLUTION:

2. Find the area of the cyclic quadrilateral whose sides are 7, 5 , 4, and 10 cm.
SOLUTION:

3. The side of a rhombus is 15 cm and the length of the longer diagonal is 24.6 cm. Find the measures of the angles and its area.
SOLUTION:

4. The bases of a trapezoid are 100m and 160m, and its altitude equal to 12m. it is to be divided into two equal parts by a line parallel to the base. find
the length of the dividing line.
SOLUTION

5. A quadrilateral ABCD have sides equal to AB = 12 m, BC = 20 m , CD = 8 cm and DA = 16.97 m. If the sum of the two opposite angles is 225° , find
the area of the quadrilateral.
SOLUTION:
6. The length of the shorter diagonal of the kite ABCD is 16. Find the area of the kite shown.

SOLUTION:

Area: 1/2 (d1d2) = 1/2 (21)(16) = 168. Ans.

EXERCISES – Answer the following questions.


1. A mirror is made of two congruent parallelograms as shown in the diagram. The parallelograms have a combined area of 9 1/3 square yards. The
height of each parallelogram is 1 1/3 yards. How long is the base of each parallelogram ? Ans. 2.67 inches

2. The area of a trapezium is 165 cm² and its height is 10 cm. If one of the parallel sides is double of the other, find the two parallel sides. Ans. 11 and
22 cm
3. In trapezoid ABCD, sides AB and CD are equal . The perimeter of this trapezoid is _____ . Ans. 34

4. One of the diagonals of the Rhombus is 12 inches. If the area of the Rhombus is 132 sq in, find the measure of the acute angle between the sides of
the rhombus . Ans. 28.61°
5. ABCD is a rhombus. AB = 2x + 1 , DC = 3x – 11 and AD = x + 13. Find the area if the included acute angle is equal to 65° . Ans. 566.4424
6. If non-parallel sides of an isosceles trapezoid are prolonged, an equilateral triangle with sides of 6 cm would be formed. Knowing that the trapezoid
is half the height of the triangle, calculate the area of the trapezoid. Ans. 11.70 cm2
7. A cyclic quadrilateral has an area of 12 cm2 . The sides of the quadrilateral AB = 3 cm, CD = 4 cm and DA = 4 cm. Find the missing side.
Ans. 3 cm
8. The vertical end of the trough has width 4.4 ft at the top , width 3.2 ft at the bottom, and depth 5.5 ft . Find the area. Ans. 13.3 ft
9. In a circular grassy plot, a quadrilateral shaped with its corners touching the boundary of the plot is to be paves with bricks. Find the area of the
quadrilateral when the sides of the quadrilateral are 36 ,77,75 and 40 m. Ans. 2886 m2
10. The lengths of the diagonals of a parallelogram are 20 in. and 30 in. They intersect at 35°. Find the lengths of the parallelogram’s sides.
Ans. 8.902 inches and 23.890 inches
11. Vinny has a garden that is shaped like a trapezoid. The trapezoid has a height of 3.4 yards and the bases are 8.2 and 5.6 yards. How much will be
available to plant? Ans. 23.46 yards2
12. The floor of building consists of 2000 tiles which are rhombus shaped and each of its diagonals are 40 cm and 25 cm in length. Find the total cost of
polishing the floor, if the cost per m² is $5. Ans. $ 500
13. The area of the rhombus is 132 cm2 . If its shorter diagonal is 12 cm, find the length of the longer diagonal. Ans. 22 cm
14. A trapezoidal lot ABCD has sides BC parallel to AD. The angle between the side BA and AD is 70° while that angle between side CD and DA is 58°
Side AD = 300 m and that of BC = 200 m. How far is the location of one end of the dividing line from a along AB? Ans. 48.48 m
15. PHIL is a kite. If ZH = 40 cm, IP = 18 cm, and m∠PLZ = 37°, what is the perimeter of the kite? Z is the point of intersection of its diagonals.
Ans. 112 cm.

TOPICS ON NOVEMBER 13,2020


1. Circles
2. Polygons and Similar Areas
REFERENCES

1. https://www.reference.com/world-view/solid-mensuration-a47c951b2f6d7508
2. https://www.math-only-math.com/word-problems-on-area-of-a-square.html
3. Engineering Mathematics Vol. 1 By Gillesania
4. Engineering Mathematica Vol. 1 by Besavilla
5. https://www.algebra.com/algebra/homework/word/geometry/Geometry_Word_Problems.faq.question.792712.html
6. Introductory and Intermediate Algebra by Blitzer
7. https://www.varsitytutors.com/high_school_math-help/how-to-find-the-area-of-a-rectangle
8. https://www.onlinemath4all.com/perimeter-of-square.html
9. Intermediate Algebra by Elayn Martin
10. https://www.onlinemathlearning.com/word-problems-area.html
11. MTAP Reviewers
12. Engineering Mathematics by Bird
13. Algebra with Trigonometry by Barnett
14. https://www.math10.com/problems/area-square-rectangle/easy/
15. Algebra and Trigonometry by Openstax
16. Multiple Choice Questions in Engineering Mathematics by Padilla
17. https://mathbitsnotebook.com/Geometry/Quadrilaterals/QDAppliedProblemPractice.html
18. Algebra and Trigonometry by Cynthia Young
19. Precalculus by Blitzer
20. Intermediate Algebra by Blitzer
21. Glencoe’s Advanced Mathematical Concepts
22. https://www.algebra.com/algebra/homework/Surface-area/Surface-area.faq.question.29176.html
23. https://www.onlinemath4all.com/area-of-parallelogram.html
24. https://www.math-only-math.com/area-of-trapezium.html
25. American Mathematics Contest
26. http://aldentech.wnyric.org/webshare/frizzo/Geometry/Chapter10.pdf
27. https://www.superprof.co.uk/resources/academic/maths/geometry/plane/trapezoid-problems.html#chapter_solution-of-exercise-6
28. Solid Mensuration by Kern and Bland
29. https://www.emathzone.com/tutorials/geometry/area-of-cyclic-quadrilateral.html
30. https://courses.lumenlearning.com/prealgebra/chapter/using-the-properties-of-trapezoids-to-solve-problems/
31. https://www.math-only-math.com/area-of-rhombus.html
32. https://www.shmoop.com/quadrilaterals/kites-examples.html

LINKS TO STUDY:

Squares and Rectangles


1. https://www.youtube.com/results?search_query=tagalog+area+of+square&page&utm_source=opensearch
2. https://www.youtube.com/results?search_query=rectangles+mensuration+

Parallelograms, Trapezoids, and other Quadrilaterals


1. https://www.youtube.com/results?search_query=tagalog+area+trapezoid+
2. https://www.youtube.com/results?search_query=dividing+line+trapezoid+
3. https://www.youtube.com/results?search_query=tagalog+area+parallelogram
4. https://www.youtube.com/results?search_query=tagalog+area+rhombus+
5. https://www.youtube.com/results?search_query=brahmagupta+formula
6. https://www.youtube.com/results?search_query=area+irregular+quadrilateral+
7. https://www.youtube.com/results?search_query=area+cyclic+quadrilateral+
CIVL ENGINEERING BOARD EXAMS PROBLEMS PHILIPPINES – November 13, 2020

CIRCLES , SECTORS AND SECANTS

Circles – set of point that contains a fixed point called a center.

Circle Theorems
1. Through three points not is a straight line one circle, and only one , can be drawn.

2. A tangent to a circle is perpendicular to the radius at the point of tangency, and conversely.

3. The tangents to a circle drawn from an external point are equal, and make equal angles with the line joining the point to the center.

4. An inscribed arc is measured by one half the intercepted arc.

5. An angle inscribed in a semicircle is a right angle. Thus, if a right triangle is inscribed in a circle, the hypotenuse is also the diameter.

6. An angle formed by two chords interesting within the circle is measured by half the sum of the intercepted arcs.
7. If two chords intersect in a circle, the product of the segments of one is equal to the product of the segments of the other.
8. An angle included by a tangent and a chord drawn from the point of contact is measured by half the intercepted arc.

9. An angle formed by two secants, two tangents , or a tangent and a secant , drawn to a circle from an external point, is measured by half the
difference of two intercepted arcs.
10. If from the point outside the circle a secant and a tangent are drawn, the tangent is the mean proportional between the whole segment and its
external segment.

11. A perpendicular from a point on the circumference to a diameter of a circle is a mean proportional between the segment of the diameter.

12. The circumferences of two circles are in the same ratio as their radii and the arcs of the two circles subtended by equal central angles are in the
same ratio as their radii.

CIRCLES, SEGMENTS, SECTORS AND OTHER FORMULAS


ELLIPSES , SPANDRELS AND PARABOLIC FIGURES
RADIUS OF CIRCLES
1. Circle circumscribed about a triangle - A circle is circumscribed about a triangle if it passes through the vertices of the triangle.

2. Circle inscribe in a triangle – A circle is inscribe in a triangle if it is tangent to three sides of a triangle.

3. Circle escribed about a Triangle – A circle is escribed about a triangle if it is tangent to one side and to the prolongation of the other two sides. A
triangle has three escribed circles.

Note: For other areas of the ungiven figures especially curves, see Integral Calculus.

EXAMPLES:
1. Calculate the angles in the figure shown.

SOLUTION:
2. Two perpendicular chords both 5 cm from the center of the circle divide the circle into four parts. If the radius of the circle is 13 cm, find the area of
the smallest part in the figure shown.
SOLUTION:

3. The ellipse has horizontal radius 9.5 cm and vertical radius 5.5 cm. Find the area of ellipse.
SOLUTION:

4. A hollow shaft (known as annulus) has an outside diameter of 5.45 cm and an inside diameter of 2.25 cm. Calculate the cross-sectional area of the
shaft.
SOLUTION:

5. A circular dog pen has a circumference of 78.5 feet. Estimate how many hunting dogs could be safely kept if each dog needs 60 feet2 of room.
SOLUTION:
Compute first the radius of a dog pen:

78.5
𝐶 = 2𝜋𝑟 ; 78.5 = 2𝜋𝑟 ; = 𝑟 ; 𝑟 = 12.494 𝑓𝑡.
2𝜋

Then, compute the area:


𝐴 = 𝜋𝑟 2 = 12.4942 𝜋 = 490.4027 𝑓𝑡 2

Compute the number of dogs needed:

𝑃𝑒𝑛 𝑎𝑟𝑒𝑎 490.427


𝐷𝑜𝑔𝑠 = = = 8.17 , 𝑠𝑎𝑦 8 𝑑𝑜𝑔𝑠. 𝐴𝑛𝑠.
𝑛𝑒𝑒𝑑𝑒𝑑 𝑎𝑟𝑒𝑎 𝑜𝑓 𝑒𝑎𝑐ℎ 𝑑𝑜𝑔 60

6. A lawn sprinkler is located at the corner of the yard is set to rotate through 90° and project water out 30 feet. Find the area watered by the lawn?
SOLUTION:
EXERCISES – Solve the following problems

1. A semi-circle 14 cm radius is formed from a piece of wire. If it is bent into a rectangle whose length is 1 cm more than its width, find the area of the
rectangle. Ans. 323.57 cm2
2. Find the area of the largest circle which can be cut from a square of edge 4 inches. What area of the material wasted? Ans. 3.434 in2
3. The length of the common chord of two circles intersecting each other is 48 cm. The distance and the radius of one circle is 25 cm. Compute the
common area of the two circles. Ans. 1191.36 cm2
4. Find the value of x and y in the figure shown. Ans. x = 90° and y = 70°

5. Two pulleys 10 m between centers are linked by a non-crossing belt. The larger pulley is 10 m in diameter and the smaller pulley is 5 m in diameter.
Find the length of the common external tangent. Ans. 9.68 m
6. AB and CD are chords of the circle that intersect at P as shown. Find x. Ans. 10

7. The base of an isosceles triangle is 16 in. and the altitude of the triangle is 15 in. Find the radius of the inscribed circle. Ans. 4.8 inches
8. The area of a triangle ABC is 65.59 cm. and its perimeter is 48 cm. Compute the radius of the inscribed circle. Ans. 2.71 cm
9. Three circles with radii 4, 5, 6cm respectively are tangent to each other externally. Find the largest angle of the triangle formed by radii.
Ans. 70.53°
10. If a chord 24 cm long is 5 cm from the center of a circle, how long is a chord 10 cm from the center? Ans. 16.61 cm
11. Elliptipool is a billiards game that use an elliptically-shaped pool table with only one pocket in the surface. A cue ball and a target ball are used in
play. The object of the game is to strike the target ball with the cue ball so that the target ball rolls into the pocket after one bounce off the side.
Suppose the cue ball and target ball can be placed anywhere on the half of the table opposite the pocket. The pool table shown at the right is 4 feet
wide and 6 feet long. If the hole is 2 inches in diameter, find the area of pool table needed for the green portion? Ans. 23π
12. On a certain vehicle, one windshield wiper is 60 cm long, and is afixed to a swing arm which is 72 cm long from pivot point to wiper-blade tip. If the
swing arm turns through 105°, what area of the windshield, to the nearest square centimeter, is swept by the wiper blade? Ans. 1512π cm2
13. How many plants spaced every 6 inches are needed to surround a circular garden with a 30 foot radius? Ans. 377 plants
14. A flood light in a football stadium can spread its illumination over an angle of 45° to a distance of 55 m. Determine the maximum area that can be
lit. Ans. 1188 m2
15. In the figure shown, PA and PB are tangents to the circle. Find the length of OP given that PB = 7 cm. Ans. 7.66 cm

POLYGONS, AREAS OF IRREGULAR FIGURES AND SIMILAR AREAS

POLYGON – plane figure with having number of sides .

Types based on angles :


Convex – polygons in which no side, when extended , will pass inside the polygon and does have angles less than 180°
Concave – polygons in which at least one of the angles having greater than 180° .

Types based on sides:


Regular – having equal sides and angles
Irregular – distinct sides and angles

Theorems:
1. The sum of the angles of a convex polygon of “n” sides is 2(n -2) right angles.
2. The exterior angles of a polygon, made by producing each of its sides in succession are equal together equal to 360°
3. Homologous parts of congruent figures are equal.
POLYGON NOMENCLATURE

SUM OF INTERIOR ANGLES

𝜃 = 180° (𝑛 − 2); 𝑛 − 𝑠𝑖𝑑𝑒𝑠

Sum of exterior angles – 360°

Diagonals

𝑛(𝑛 − 2)
𝐷=
3
REGULAR POLYGONS

AREA AND PERIMETER OF REGULAR POLYGON INSCRIBED IN A CIRCLE.

AREA AND PERIMETER OF A POLYGON CIRCUMSCRIBING A CIRCLE

SIMILAR AREAS

(𝑥1 )2 𝐴1
2
=
(𝑥2 ) 𝐴2

OTHER AREAS
1. Composite figures – by solving the combined areas
2. Trapezoidal Rule – using segments as composite trapezoids

Warning: This formula is not applicable for different intervals. And this topic is more easier to understand on Integral Calculus.
3. Simpson’s One Third Rule – More accurate calculation especially in curved sides. Odd number of offsets and similar common intervals are the
requirements in order to use this method.

EXAMPLES:

1. The perimeters of two similar triangles is in the ratio 3:4. The sum of their areas is 75 cm2. Find the area of each triangle.
SOLUTION:

2. Calculate the amount of paint needed to paint the front of this building knowing that 0.5 kg of paint is needed per square meter as shown.

SOLUTION:

Note: The triangles at sides are considered at the side views.

3. A grassy plot has the shape of a regular hexagon each side 100 m. Within the plot and along its sides a foot path is made 4 m wide all around. Find
the area of the grassy lot left within.
SOLUTION:
4. If an interior angle of a regular polygon measures 108° , find the number of sides.
SOLUTION:

Therefore , it is a pentagon.

5. A regular pentagon is inscribed in a circle with diameter 8.34 centimeters. The apothem of a regular polygon is the measure of a line segment from
the center of the polygon to the midpoint of one of its sides. Find the apothem of the pentagon.
SOLUTION:

6. Approximate the area of f(x) = x2 +1 on the interval [0,3] with n = 6 using Simpson’s Rule.
SOLUTION:
Tabulate the values first:

This will be much easier when we will use integral calculus.

EXERCISES – Solve the following problems.

1. A regular hexagon is inscribed in a circle with a radius 6.4 cm long. Find its apothem. Ans. 5.5 cm
2. Hardwood flooring costs P 100 per square meter. How much will it cost to cover the auditorium with that flooring? Ans. P 245,663.06

3. Find the perimeter of the pentagon inscribed in a circle of radius 12.6 meters. Ans. 74.061 m
4. How many triangles are formed by 7 points, no three of which are collinear? Hint: Use combinations. Ans. 35
5. Each interior angle of a regular polygon measures 160° . How many diagonals that can be formed? Ans. 135
6. A map has the scale of 1 inch: 15 miles. If the city has 300 square miles on actual, find the area on the map. Ans. 1.3333 in2
7. Find the number of sides of a polygon that has 54 diagonals. Ans. 12 sides
8. In the arrow shaped polygon shown , determine the area. AB̅ ⊥ AG̅ . Ans. 300

9. The sides of the pentagon is in the ratio 2:5 . If the smaller area is equal to 27.5 cm2, find the area of the larger one. Ans. 171.875 cm2
10. A unit hexagon is composed of a regular hexagon of side length 1 and its equilateral triangular extensions, forming a hexagram or six-sided star.
What is the ratio of the area of the extensions to the area of the original hexagon? Ans. 1/2
11. Determine the number of diagonals in a 20-gon. Ans. 170
12. The area of a hexagon inscribed in a circle is 166.28 cm2. If the circle is also inscribed in a square, find the total perimeter of the hexagon.
Ans. 48 cm
13. Norman window has the shape of a rectangle with a semicircle attached at the top. The diameter of the semicircle is equal to the width of the
rectangle. The window has a perimeter of 12 feet and its height is twice the radius. Find the area of the window. Ans. 16.492 ft2
14. If a regular polygon has 27 diagonals, what figure is this? Ans. Nonagon
15. A circle of radius 6 cm is inscribed in a 5-sided regular polygon (pentagon), find the length of one side of the pentagon. Ans. 8.7 cm

NEXT TOPICS ON NOVEMBER 16,2020:

1. Cubes and Rectangular Parallelpiped (Rectangular Prism)


2. Prisms and Cavaleri’s Theorem
3. Cylinders
REFERENCES :
1. Engineering Mathematics by Besavilla
2. Engineering Mathematics by Gillesania
3. Precalculus by Blitzer
4. https://www.bbc.co.uk/bitesize/guides/zcsgdxs/revision/9
5. https://www.web-formulas.com/Math_Formulas/Geometry_Area_of_Ellipse.aspx
6. Intermediate Algebra by Elayn Martin
7. Solid Mensuration by Kern and Bland
8. Scahum’s Outlines of Geometry
9. https://www.mathopolis.com/questions/q.html?id=15600&t=mif&qs=15593_15594_15595_15596_15597_15598_15599_15600&site=1&ref=2f676
56f6d657472792f636972636c652d696e746572736563742d63686f7264732e68746d6c&title=496e74657273656374696e672043686f7264732054
68656f72656d
10. http://www.math-principles.com/2014/04/circle-inscribed-triangle-problems.html
11. https://socratic.org/questions/three-circles-with-radii-4-5-6cm-respectively-are-tangent-to-each-other-external
12. MTAP Questionnaires
13. Glencoe’s Advanced Mathematical Concepts
14. https://www.purplemath.com/modules/sectors2.htm
15. Introductory and Intermediate Algebra by Blitzer
16. Engineering Mathematics by Bird
17. https://en.wikipedia.org/wiki/Polygon#:~:text=Individual%20polygons%20are%20named%20(and,quadrilateral%20and%20nonagon%20are%20ex
ceptions.
18. Surveying for Licensure and Geodetic Examinations by Besavilla
19. https://www.superprof.co.uk/resources/academic/maths/geometry/plane/area-word-problems.html#chapter_exercise-3
20. https://www.pbte.edu.pk/text%20books/dae/math_113/Chapter_12.pdf
21. https://www.onlinemath4all.com/sum-of-interior-angles-of-a-polygon.html
22. Intermediate Algebra by Kauffman
23. Contest Problem Book of AIME
24. https://www.onlinemathlearning.com/scale-factor.html
25. https://byjus.com/diagonal-of-polygon-formula/
26. Algebra and Trigonometry by Cynthia Young
27. UPCAT Reviewers
CIVIL ENGINEERING BOARD EXAMS PROBLEMS PHILIPPINES – November 16, 2020

CUBES AND RECTANGULAR PRISMS (RECTANGULAR PARALLELPIPED)

Cube – solid figure with having six square shaped faces.


Prism – solid figure with polygonal base. The number of faces may vary.

Cavalieri’s Theorem - Suppose two regions in three-space (solids) are included between two parallel planes. If every plane parallel to these two planes
intersects both regions in cross-sections of equal area, then the two regions have equal volumes.

FORMULAS

PRISMS

EXAMPLES:

1. A building contractor is to dig a foundation 4 yards long, 3 yards wide and 2 yards deep for a toll booth’s foundation. The contractor pays P 1,000
per load for trucks to remove the dirt. Each truck holds 6 cubic yards. What is the cost to the contractor to have all the dirt hauled away?
SOLUTION:
Compute first the volume needed :

𝑉 = 𝑙𝑤ℎ = 4 ∗ 3 ∗ 2 = 24 𝑦𝑑3

Then, divide number of trucks needed:

24 𝑦𝑑3
𝑇𝑟𝑢𝑐𝑘𝑠 = = 4 𝑡𝑟𝑢𝑐𝑘𝑠
𝑦𝑑3
6
𝑡𝑟𝑢𝑐𝑘

Total cost:

𝑃ℎ𝑝 1000
𝐶𝑜𝑠𝑡 = 𝑡𝑟𝑢𝑐𝑘𝑠 ∗ 𝑢𝑛𝑖𝑡 𝑐𝑜𝑠𝑡 = 4 ∗ = 𝑃ℎ𝑝 4,000. 𝐴𝑛𝑠.
𝑡𝑟𝑢𝑐𝑘
2. The length of each edge of the cube is increased by 10% , by what percent is the volume increased?
SOLUTION:

3. Each end of a glass prism is a triangle with a height that is 1 inch shorter than twice the base. If the area of a triangular part is 60 in2, determine the
dimensions and the volume of the prism if the length is 10 inches.
SOLUTION:

Compute the dimensions of the triangle:

1 𝑥(2𝑥 − 1)
𝐴 = 𝑏ℎ ; 60 =
2 2

Then cross multiply 60 by 2 and simplify the right hand side:

120 = 2𝑥 2 − 𝑥

Rearranging the terms:

2𝑥 2 − 𝑥 − 120 = 0 → 𝑆𝑜𝑙𝑣𝑒 𝑣𝑖𝑎 𝑞𝑢𝑎𝑑𝑟𝑎𝑡𝑖𝑐 𝑓𝑜𝑟𝑚𝑢𝑙𝑎

Note: multiplying both negative signs are positive and negative values is not allowed when solving geometric quantities.

−𝑏 ± √𝑏 2 − 4𝑎𝑐 1 ± √12 − 4(2)(−120) 1 ± √481 1 + 31


𝑥= = = = = 8.
2𝑎 2(2) 4 4

Substituting the dimensions:

2𝑥 − 1 = 2(8) − 1 = 15 ; 𝐷𝑖𝑚𝑒𝑛𝑠𝑖𝑜𝑛𝑠 𝑎𝑟𝑒 8 𝑎𝑛𝑑 15 𝑖𝑛𝑐ℎ𝑒𝑠. → 𝐴𝑛𝑠.

Computing the volume:

𝑉 = 𝐵ℎ = 60 ∗ 10 = 600 𝑖𝑛3 → 𝐴𝑛𝑠.

4. A fish tank is 40 cm long, 15 cm broad and 10 cm high. What is its weight when filled with seawater? Weight factor or specific gravity of a seawater
is 1.03 grams/cm3
SOLUTION:

Weight = Sp gr* V = 6000*1.03 = 6180 grams/cm3 → Ans.


5. Given a 4 cm3 cube divided into 1 cm edge , how many faces were painted at two sides?
SOLUTION:

By inspecting the cubes thoroughly, there are 4*4*4 = 64 pcs of cubes in total.

When we say painted two sides, only two sides are painted. By checking the number of cubes painted twice at the edges (not corners) ,
At the top we found 8 pieces we as well as at the bottom. At the sides , we have 8 also, combining these faces (8+8+8) makes 24 pieces of cubes
in which two faces painted.

6. (CE Board) A trough has an open top 0.30 m by 6 m and closed vertical ends which are equilateral triangles 30 cm on each side. It is filled with
water to half its depth. Find volume of the water in cubic meters.
SOLUTION:
The volume we refer is the water. So, the volume will be

1 2
𝑉 = 𝐴𝑏 𝐿 , 𝐴= 𝑎 sin 60°
2

EXERCISES – Answer the following questions.

1. The total surface area of a cube is 216 in2 , find the volume of the cube. Ans. 216 in3
2. Given a 4 x 3 x 2 cube, how many cubes were painted at one side? Ans. 4
3. Lisa uses a sheet of paper to make a rectangular box with a dimensions 12 cm x 8 cm x 18 cm. Each 50 cm2 costs P 15. How much will it cost if
she will make 20 boxes? Ans. P 5,472
4. Find the volume and surface area of a rectangular box that is 12 in., 6 in., and 3 in., high. Ans. 252 in2 - surface area and 216 in3 - volume
5. A flat square piece of cardboard is used to construct an open box. Cutting a 1 ft by 1 ft square off of each corner and folding up the edges will yield
an open box. If the desired volume of the box is 9 cubic feet, what is the length of the edge of the original square piece of cardboard?
Ans. 5 inches.
6. The glass has 600 ml of tea, which represents 80% of the volume of the glass. If you put twenty regular ice cubes of 2 cm in the tea, how many ml
of tea are poured? Ans. 10 mL
7. A solid cube has side length 3 inches. A 2-inch by 2-inch square hole is cut into the center of each face. The edges of each cut are parallel to the
edges of the cube, and each hole goes all the way through the cube. What is the volume, in cubic inches, of the remaining solid? Ans. 7 in3
8. A fish tank is 50 cm long, 30 cm wide and 40 cm high. It contains water up to a height of 28 cm. How much water is needed to fill the tank to a height
of 35 cm? Ans. 10,500 cm3
9. A dam 100 ft long has a cross section which is a trapezoid whose altitude is 16 feet and whose upper base is 5 ft. If the angles at the lower base
are 50° and 65° , find the volume of the material the dam contains. Ans. 24,710 ft3
10. A closet is 6 ft wide, 5 ft deep and 8 ft tall. In the closet, there is a suitcase that is 2 feet wide, 3 feet long and 4 feet tall. How much room is left in
the closet? Ans. 216 ft3
11. A right, rectangular prism has a surface area of 28 square meters. Its width is twice its length, its height is four times its length. Find the diagonal
distance of the prism. Ans. √21 m
12. One edge of a regular hexahedron is 24 cm long. If the edge is 50% increased, how much the percentage will the volume increase? Ans. 237.5%
13. Two cubes of volume 664 m3 each are joined end to end. Find the surface area of the formed cuboid (known as rectangular prism) Ans. 761.10 cm2
14. A quart of stain covers 100 ft2 . How many quarts you should buy to stain the wheelchair ramp as shown? 1 ft 3 = 29.221 quarts. Ans. 3804.82
quarts

15. A candy box is to be made out piece of cardboard that measures 8 by 12 inches. Squares, 1 inch on each side , will be cut from each corner, then
the ends and sides will be folded down. Find the gross weight of the candy box if candy contains 45 ounces and carton weighs 0.10 ounces per
square inch. Assume that the candy box is wrapped with cellophane. Ans. 54.2 ounces
CYLINDERS – solid figure in which surface generated by a straight line intersecting and moving along a closed plane curve (directrix) , while remaining parallel
to a fixed straight line (the axis) that is not on or parallel to the plane of the directrix.

For slanted cylinder, use Cavalieri’s theorem.


TRUNCATED CYLINDER

1
𝑉= (ℎ + ℎ2 )𝜋𝑟 2
2 1

EXAMPLES:

1. A cylindrical carton is used as a container to put oatmeal before selling it to customers. The diameter of the base of the carton is 10 centimeters
and the height of the carton is 18 centimeters.
SOLUTION:

2. The can is to have a volume of 12 fl oz , approximately 22 cubic inches. Express the surface area of the can , A, in square inches as a function of
its radius, r, in inches.
SOLUTION:
3. Emily brought the lollipops jar near the store, the jar is in a cylindrical shaped and has a radius 2.4 mm and height 7.5 mm. What is the surface area
of the jar?
SOLUTION:
Surface Area:

𝐴 = 2𝜋𝑟 2 + 2𝜋𝑟ℎ

𝐴 = 2𝜋(2.4)2 + 2𝜋(2.4)(7.5) = 149.288 𝑚𝑚2

EXERCISES – Answer the following questions.

1. The battery is in the shape of a cylinder and has a weight of 22.419 grams and a radius of 5 mm. Find the length of the battery when unit weight of
cadmium is equal to 0.00865 g/mm3 . Ans. 33 mm
2. Find the waste in making the largest possible cylindrical rod from a bar of iron 3 ft long which has a square cross section whose diagonal is 6 inches
square. Ans. 139.06 cu in.
3. John is considering buying a right cylindrical water container. The container needs to be insulated on the top, bottom, and side. The store has two
containers to choose from. Container #1 has a height of 120 cm and a radius of 30 cm. Container #2 has a height of 100 cm and a radius of 35 cm.
If John wants to use the least amount of insulation (assuming constant thickness of insulation), which container should he buy? Ans. Container 1
4. The radius of the cylinder is reduced by 4% and its height increased by 2%. Determine the approximate change in volume. Ans. -6%
5. There are 24 cans of soda and they are arranged in 4 rows of 6. Each can has a radius of 4 cm and a height of 15 cm. These cans are to be packed
snuggly into a case with no extra space on the top, bottom, or sides. Calculate how much empty space will not be occupied by a can in the box.
Ans. 4944.426 cm3
6. A cylinder with radius 2 inches and height 3 inches has its radius quadrupled. How many times greater is the volume of the larger cylinder than the
small cylinder? Ans. 16 times
7. The figure shows a section of a metal pipe. Given the material radius of the pipe is 2 cm, the external radius is 2.4 cm and the length of the pipe is
2.4 cm and the length of the pipe is 10 cm. Find the volume of the metal used. Ans. 55.3 cm3

8. Find the diameter of the right circular cylinder if its lateral area is equal to its volume. Ans. 4
9. A cylindrical can that is four inches tall and has a radius of 1.5 inches can hold 10¢ worth of soda. Assuming that the value of the contents is
proportional to the size (volume) of the can, what would be the value of the soda contained in a can that is 8 inches tall with a radius of 3 inches?
Ans. 80 cents
10. What happens to the volume if you double the height? Ans. The volume will be doubled

NEXT TOPICS ON NOVEMBER 20,2020:

1. Pyramids
2. Cones
3. Frustums
REFERENCES

1. https://www.khanacademy.org/math/geometry/hs-geo-solids/xff63fac4:hs-geo-cavalieri-s-principle/a/cavalieri-s-principle-in-3d
2. Intermediate Algebra by Blitzer
3. https://socratic.org/questions/the-length-of-each-edge-of-a-cube-is-increased-by-10-by-what-percent-is-the-volu
4. Engineering Mathematics by Besavilla
5. Engineering Mathematics by Gillesania
6. https://www.math-only-math.com/cuboid.html
7. https://www.onlinemathlearning.com/surface-area-cube.html
8. https://www.henry.k12.va.us/cms/lib/VA01000023/Centricity/Domain/642/HW4%20SA%20of%20Rectangular%20Prisms.pdf
9. https://www.hackmath.net/en/math-problem/4806?tag_id=40_6
10. Algebra and Trigonometry by Cynthia Young
11. American Mathematics Contest Level 12
12. https://artofproblemsolving.com/wiki/index.php/2010_AMC_12A_Problems/Problem_9
13. http://ibkmartin.weebly.com/uploads/1/0/3/4/10343187/12.4_book.pdf
14. Solid Mensuration by Kern and Bland
15. https://www.k5learning.com/worksheets/math/grade-5-volume-word-problems-a.pdf
16. https://www.henry.k12.va.us/cms/lib/VA01000023/Centricity/Domain/642/HW4%20SA%20of%20Rectangular%20Prisms.pdf
17. https://www.varsitytutors.com/intermediate_geometry-help/how-to-find-the-diagonal-of-a-prism
18. https://www.toppr.com/ask/question/2-cubes-of-volumes-664-cm3-are-joined-end-to-end-fin-the-surface-area/
19. https://www.basic-mathematics.com/volume-of-cylinders.html
20. Precalculus by Blitzer
21. Intermediate Algebra by Elayn Martin
22. http://ion.uwinnipeg.ca/~jameis/MY%20course/my3stages/gr%208%20SA%20cylinder.pdf
23. http://www.algebralab.org/practice/practice.aspx?file=Word_VolCylnders.xml
24. https://www.onlinemathlearning.com/volume-of-a-cylinder.html
25. https://www.math.fsu.edu/~wooland/hm2ed/Part3Module9/Part3Module9.pdf
26. Explorations in College Algebra by Kime
CIVL ENGINEERING BOARD EXAMS PROBLEMS PHILIPPINES – November 20, 2020

PYRAMIDS AND FRUSTUMS

PYRAMIDS – polyhedron or solid figures with a polygonal base and triangular faces that meet to common point (vrertex)

FRUSTUMS - portion of the pyramid between the base and a cutting plane parallel to the base.

FORMULAS:

For other solids’ volumes, use the formula V = 1/3 bh

Frustums
EXAMPLES:

1. Felice bought the candle below for her friend’s birthday. The package says that the candle burns one hour for every 20 cubic centimeters of wax.
How many hours will it take the entire candle to burn?
SOLUTION:

2. Determine the total surface area of the 5 cm square pyramid if the perpendicular height is 12 cm.
SOLUTION:

3. The frustum of a pyramid has rectangular ends , and the sides of the base are 25 dm and 36 dm. If the area of the top face is 784 dm2 and the height
is 60 dm, find its volume.
SOLUTION:

4. Find the altitude of a frustum of a pyramid whose volume is 148 ft3 . The areas of the frustum are 9 ft3 and 36 ft3 .
SOLUTION:

5. The base of a right pyramid is a square of side 24 cm. and its height is 16 cm. Find the area of the slant surface.
SOLUTION:
EXERCISES – Answer the following questions.

1. A frustum of a pyramid has a lower base of 12 x 12 cm and an upper base of 8 cm x 8 cm. If the lateral edge is 18 cm., compute the total surface
area of the regular pyramid. Ans. 923.6 cm2
2. A rectangular hexagonal pyramid has a slant height of 4 cm and the length of each side the base 6 cm. Find the lateral area. Ans. 72 cm2
3. The height of a regular tetrahedron is 4√6 . Find the volume . Ans. 144√2 cm3
4. A hotel in Las Vegas is in the shape of a pyramid with square base. An architect designed a scale model . If the height is 9 inches less than its length
base and the volume if equal to 6300 cubic inches, find the lateral area. Ans. 749.2892 inches2
5. A triangular pyramid has a volume of 266 cubic feet and a base area of 42 square feet. What is its height? Ans. 19 feet
6. The frustum of a regular pyramid has an upper base 8 x 80 m and a lower base of 10 x 100 m with the altitude of 5 meters. Find the volume.
Ans. 4066.67 m3
7. A frustum of a regular pyramid has a lower base of 12 cm x 12 cm and an upper base of 8 x 8 cm. If the lateral edge is 18 cm , compute the volume
of the frustum. Ans. 1801.71 cm3
8. (CE Board ) the volume of the frustum of a regular triangular pyramid is 135 m3 . The lower base is an equilateral triangle with an upper edge of 9
m. The upper base is 8 m above the lower base. Find the upper base edge. Ans. 3 meters
9. Calculate the volume of the pillar (shaped in frustum of a pyramid) if the square have sides of 19 and 27 and a height of 48 . Ans,. 25,648 u3
10. Calculate the volume of the frustum of a square pyramid whose larger base is 24 cm , smaller base is 14 cm and the lateral edge of 13 cm.
Ans. 4029.43 m3
11. Gemma made a mobile to hang over her brother’s crib. She put each animal on the mobile in a clear plastic case. The shape of one case was an
equilateral triangular pyramid. Find the surface area of the pyramid. Ans. 13.7 inches.
12. A church spire is in the form of a regular hexagonal pyramid whose base edge is 8 ft and whose altitude is 75 ft is to be painted at a cost of 22 cents
per square yard. What is the total cost? Ans. $44.19.

CONES and FRUSTUMS OF A CONE

CONES – solid figure generated by a straight line , the generator, passing through a fixed point, the vertex, and moving along the a fixed curve , the directrix.

FORMULAS

𝐵ℎ
𝑉=
3

RIGHT CIRCULAR CONE


FRUSTUM OF A CONE

EXAMPLES:

1. A corn cob somewhat like a cone, has the radius of the broadest end as 2.1 cm and length as 20 cm. If each 1 cm2 of the surface of the cob has an
average of 4 grains , determine the number of grains in the entire cob.
SOLUTION:

2. A water storage tank is in the shape of a cone with pointed end down. If the radius is 14 ftt and the depth of the tank is 15 ft, approximate the volume
of the tank in cubic feet.
SOLUTION:
𝜋 2 𝜋
𝑉= 𝑟 ℎ= (14)2 (15) = 3078.7608 𝑓𝑡 3 → 𝐴𝑛𝑠.
3 3

3. A reservoir contains 28.54 m3 of water when full. The diameter of the upper base is 3.5 m, while the lower base is 2.5 m. Find the height if the reservoir
is in the form of the frustum of a right circular cone.
SOLUTION:
4. The bucket is in the shape of the frustum of a right circular cone as shown. Find the volume.

SOLUTION:

5. A cone can be formed from a 252° sector of a circle of radius 10 cm which is made of a cardboard . Find the weight of the candy could be filled if
each cubic centimeter is 2.5 grams.
SOLUTION:

Let 10 be the lateral edge . then converting the arc length to a circumference of a cone:

2𝜋 ∗ 252°
𝐶= (10) = 14𝜋 .
360°

𝐶𝑐𝑜𝑛𝑒 = 2𝜋𝑟 ; 14𝜋 = 2𝜋𝑟 ; 𝑟 = 7 𝑐𝑚.

Compute the height :

ℎ = √𝐿2 − 𝑟 2 = √102 − 72 = 7.1414

Solving the volume :


𝜋 2 𝜋
𝑉= 𝑟 ℎ = (7)2 (7.1414) = 366.4444 𝑐𝑚3
3 3

Compute the weight : 366.4444 cc * (2.5 g./1 cc) = 916.1109 grams.

EXERCISES - Solve the following questions.

1. A conical silo of radius 9 feet and height 14 feet releases cereals at its bottom at a constant rate of 20 cubic feet per minute. How long will it take for
the silo to be empty? Ans. 59 mins
2. A hole 10 cm in diameter is to be punched out from a right circular cone having a diameter of 16 cm. Height of the cone is 24 cm. Determine the
remaining volume after the hole is punched out. Ans. 508.94 cm3
3. An open plastic drum of height 63 cm with radii of lower and upper ends as 15 cm and 25 cm respectively is filled with gasoline. Find the cost of milk
which can completely fill the bucket at P 45 per liter and VAT rate of 12% is excluded. P 4074.84
4. A silo is shaped like a cone and contains wheat. The radius is 10 feet and the height is 15 feet. If the silo can release wheat from its bottom at the
rate of 25 cubic feet per minute, how long would it take for the silo to empty fully? Ans. 63 mins
5. The paper drinking cup is in the shape of a right circular cone. It is constructed 125 cm3 of paper with the height 10 cm. Determine the radius of the
cone. Ans. 3.7282 cm
6. A cone has a circular base of radius 10 cm and a slant height of 30 cm. Calculate the surface area. Ans. 1,256.6370 cm2
7. If radii of the circular ends of a frustum which is 45 cm high are 28 cm and 7 cm, find the volume. Ans. 48,490.4826 cm3
8. A circular piece of cardboard with a diameter 1 meter will be made into a conical nut 40 m high by cutting a sector off and joining the edges to form
a cone. Compute the angle subtended by the sector removed. Ans. 144°
9. When Mt St Helens erupted, it removed 0.4 mile radius and 0.25 mile height in a shape of a cone. Determine the present volume of that volcano
when it is 3 miles in radius and 1.83 miles in altitude. Ans. 17.2055 cubic miles
10. Find the volume of this frustum shown. Ans. 12733.9222 mm3

11. If 23 cubic meters of water are poured into a conical vessel, it reaches a depth of 12 cm. how much water must be added so that the depth reaches
18 cm? Ans. 54.625 m3
12. You have 10 gallons of lemonade to sell. Each cup has 8 cm diameter and 11 cm height. How many cups can be made? 1 gallon = 3785 cm3.
Ans. 206 cups
13. (EE Board) What is the volume of a frustum of a cone whose upper base is 15 cm in diameter and lower base of 10 cm in diameter with an altitude
of 25 cm? Ans. 3108.87 cm3

NEXT TOPICS ON NOVEMBER 23,2020

1. Spheres
2. Zones, Segments of a Sphere
3. Spherical Pyramids
References:

1. Engineering Mathematics by Gillesania


2. Engineering Mathematics by Besavilla
3. Engineering Mathematics by Bird
4. https://www.math-only-math.com/frustum-of-a-pyramid.html
5. https://owlcation.com/stem/Finding-the-Surface-Area-and-Volume-of-Frustums-of-a-Pyramid-and-Cone
6. https://www.math-only-math.com/problems-on-pyramid.html
7. Glencoe’s Advanced Mathematical Concepts
8. https://www.hackmath.net/en/math-problem/709?tag_id=231
9. https://www.superprof.co.uk/resources/academic/maths/geometry/solid/truncated-pyramid.html
10. Solid Mensuration by Kern and Bland
11. https://www.ask-math.com/surface-area-of-cone.html
12. https://www.hackmath.net/en/word-math-problems/frustum
13. Intermediate Algebra by Elayn Martin
14. https://www.emathzone.com/tutorials/geometry/frustum-of-a-cone.html
15. American Mathematics Contest
16. https://www.storyofmathematics.com/volume-of-a-cone
17. https://www.onlinemath4all.com/volume-of-a-cone-word-problems.html
18. College Algebra by Barnett
19. https://www.toppr.com/guides/maths/surface-areas-and-volumes/frustum-of-cone/
20. http://www.brainkart.com/article/Volume-of-frustum-of-a-cone_39426/
21. https://www.murrieta.k12.ca.us/cms/lib5/CA01000508/Centricity/Domain/1830/T9.5.pdf
22. http://www.west-jefferson.k12.oh.us/Downloads/Geometry%20Glencoe%202003%20chap13.pdf
23. https://www.bigideasmath.com/protected/content/ipe_cc/grade%207/07/g7_07_04.pdf
24. https://pinoybix.org/2016/09/what-is-the-volume-of-a-frustum-of-a-cone-01.html

YOUTUBE RECOMMENDED LINKS

Pyramids and Frustums of Pyramids


1. https://www.youtube.com/results?search_query=tagalog+volume+pyramid&page&utm_source=opensearch
2. https://www.youtube.com/results?search_query=tagalog+surface+area+pyramid
3. https://www.youtube.com/results?search_query=tagalog+volume+frustum+of+pyramid

Cones and Frustums of Cones


1. https://www.youtube.com/results?search_query=tagalog+volume+cone+
2. https://www.youtube.com/results?search_query=tagalog+lateral+area+cone+
3. https://www.youtube.com/results?search_query=tagalog+volume+frustum+of++cone+
CIVIL ENGINEERING BOARD EXAMS PROBLEMS PHILIPPINES - November 23, 2020

SPHERES

SPHERES - solid figure with set of points that are all at the same distance r from a given point in a three-dimensional space.

FORMULAS:

EXAMPLES:
1. If the surface area of the sphere is increased by 21% , find the percentage increase in volume.
SOLUTION:

2. The earth’s radius is about 3960 miles. 70% of them is water. Determine the area covered with water.
SOLUTION:

𝑆𝐴 = 4𝜋𝑟 2 = 4𝜋(3960)(0.7) = 34,833.9793 𝑚𝑖 2 . → 𝐴𝑛𝑠.

3. What happens to the volume if the radius is doubled?


SOLUTION:

4𝜋 3 4𝜋 4𝜋
𝑉= 𝑟 = (2𝑥)3 = 8𝑥 3
3 3 3

The volume will be eight times.

4. A metal ball 24 kg has its density of the metal equal to 8000 kg/m3 . Determine the radius of the metal ball.
SOLUTION:
Convert first into the volume of a metal:

1 𝑚3 100 𝑐𝑚 3
24 𝑘𝑔 ∗ ∗( ) = 3000 𝑐𝑚3
8000 𝑘𝑔 1𝑚

Solve the radius of the metal ball:

4
3000 = 𝜋𝑟 3 ; 𝑟 = 8.947 𝑐𝑚. → 𝐴𝑛𝑠.
3

5. Air is leaking from a spherical-shaped advertising balloon at the rate of 26 cubic feet per minute. If the radius of the ball is 7 feet, how long would it
take for the balloon to empty fully ?
SOLUTION:
Solve the volume first:

4𝜋 3 4𝜋
𝑉= 𝑟 = ∗ 73 = 1436.7550 𝑓𝑡 3
3 3

Then the time will take to empty the air:


1436.7550 ft3 * ( 1 min/ 26) = 55.2598 mins. → Ans.
EXERCISES – Solve the following questions.

1. What is the size of the surface of fitball with diameter 65 cm? Ans. 13273.229 cm3
2. A sphere and a cube have the same surface area. Find the ratio of their volumes with respect to the sphere. Ans. √6 : √π
3. The volume of the spherical ball is 5,000 cm3 , find the radius. Ans. 10.61 cm
4. The steel ball is in the shape of the sphere with a diameter of 12 mm. Find the weight of the ball if steel has a weight of 7.8 grams per cc.
Ans. 0.90477 grams
5. Two balls of ice 10 m diameter are melted down and recast into a conic ice with height equal to the radius of its base. Find the height of the cone.
Assume that all melted ice will be consumed. Ans. 10 meters
6. If thin chocolate coating on a large malted milk ball has increased its radius by 3 mm , find the increase in amount of liquid sugar needed if it contains
20%. Assume radius of a large malted milk ball is 100 mm. Ans. 1.018%
7. If the radius of the sphere is tripled, by what number is its volume multiplied? Ans. 27 times
8. The radius of the sphere is decreased by 5%. Find the decrease in surface area. 9.75%
9. A big cantaloupe has a diameter of 26 inches. How much surface area does its peel covered? Ans. 2123.7166 in2
10. The globe of earth ,made of Styrofoam , is in the shape of the sphere with 14 cm radius. Find the volume of the Styrofoam ball needed. Ans.
11,494.0403 cm3
11. The volume of a sphere is equal to the surface area. Find the radius. Ans. 3
12. Mars has an approximate diameter of 6.794 × 103 kilometers. Find the surface area of the gases covered. Ans. 1.45 × 108 sq km
13. A spherical tank has radius of 21 m. Find the capacity on it in liter to store water in it. Ans. 38.8 million liters

SECTORS, ZONES AND SEGMENTS OF SPHERES

SPHEREICAL SECTOR – portion of sphere defined by a conical boundary with apex at the center of the sphere.
SPHERICAL SEGMENT/ZONE OF SPHERE – solid portion of sphere cut by the pair of parallel planes
SPHERICAL LUNE – area on a sphere bounded by two half great circles which meet at antipoidal points
SPHERICAL WEDGE/UNGULA – portion of a sphere bounded by two plane semidisks and a spherical lune.

FORMULAS

Warning: If the given in the spherical segment is the base radius, use the formula involving two bases assuming the base radius of the other circle is zero.
EXAMPLES:

1. The central angle of the spherical wedge is 1 radian. Find the volume for 1 unit radius.
SOLUTION:

2. The sphere of radius 8 cm is cut by two parallel planes, one passing 2 cm from the center and the other 6 cm from the center. Find the area of the
zone.
SOLUTION:

EXERCISES – Answer the following questions.


1. The volume of the two spheres are in the ratio of 27:343 and their sum of the radii is 10 cm. Find the volume of the larger. Ans. 1436.76 cm3
2. A spherical wooden ball 15 cm in diameter sinks to a depth of 12 cm in a certain liquid. Find the area of exposed above the liquid. Ans. 141.37 cm3
3. Find the volume of the spherical wedge with the radius 8 meters and the dihedral angle of 45° . Ans. 268 m3
4. A wooden ball 11.15 in. in diameter sinks to a depth of 9.37 in. in water. Find the area of the wet surface. Ans. 328.2195 in2
5. (CE Board) What is the area of the spherical segment having a volume of 1470.265 cu. m. if the diameter of the sphere is 30 m? Ans. 565.5 m2
6. What is the surface of the spherical cap which has the height 40 cm and base radius as 30 cm? Ans. 82,809.52 m3
7. A domed stadium is in the shape of spherical segment with a base radius of 150 m. The dome must contain a volume of 3500000 m³. Determine the
height of the dome at its center. Ans. 88.6933 meters

CULMIINATING TOPICS ON NOVEMBER 27,2020 :

1. Miscellaneous Solid Figures


2. Similar Volumes and Surface Areas
3. Polyhedrons
References

1. https://www.google.com/search?q=wiki+spheres&oq=wiki+spheres&aqs=chrome..69i57j0i22i30i457j0i22i30l6.2103j0j7&sourceid=chrome&ie=UTF-
8
2. Engineering Mathematics by Gillesania
3. https://www.murrieta.k12.ca.us/cms/lib5/CA01000508/Centricity/Domain/1830/T9.6.pdf
4. College Algebra with Trigonometry by Barnett
5. Engineering Mathematics by Bird
6. https://www.hackmath.net/en/math-problem/3061?tag_id=42&result=1
7. LeHigh Math Contest
8. https://www.onlinemathlearning.com/volume-of-a-sphere.html
9. Intermediate Algebra by Elayn Martin
10. https://www.pbte.edu.pk/text%20books/dae/math_113/Chapter_19.pdf
11. Geometry The Easy Way
12. Higher Engineering Mathematics by Bird
13. https://courses.lumenlearning.com/prealgebra/chapter/finding-the-volume-and-surface-area-of-a-sphere/
14. Glencoe’s Advanced Mathematical Concepts
15. https://pinoybix.org/2016/09/find-the-volume-of-a-spherical-wedge-01.html
16. https://www.emathzone.com/tutorials/geometry/zone-or-frustum-of-a-sphere.html
17. Engineering Mathematics by Besavilla
18. https://byjus.com/spherical-wedge-and-spherical-lune-formula/
19. http://www.math-principles.com/search?updated-max=2013-08-23T03:39:00-07:00&max-results=2&reverse-paginate=true
20. https://pinoybix.org/2016/09/what-is-the-area-of-the-zone-of-a-spherical-segment-01.html
21. https://byjus.com/spherical-cap-volume-formulas/
22. https://www.hackmath.net/en/math-problem/2682?tag_id=131

YouTube links:

Spheres Surface Area and Volume :


1. https://www.youtube.com/results?search_query=tagalog+volume+of+sphere&page&utm_source=opensearch
2. https://www.youtube.com/results?search_query=tagalog+surface+area+of+sphere

Zones, Spherical Segments, Sectors:

1. https://www.youtube.com/results?search_query=spherical+wedges
2. https://www.youtube.com/results?search_query=spherical+lune+
3. https://www.youtube.com/results?search_query=spherical+zone+
4. https://www.youtube.com/results?search_query=spherical+zone+
5. https://www.youtube.com/results?search_query=zone+of+sphere+tagalog+
CIVIL ENGINEERING BOARD EXAMS PROBLEMS PHILIPPINES – November 27, 2020

POLYHEDRONS – these are solids whose faces are plane polygons.

REGULAR POLYGONS OR PLATONIC SOLIDS - those polyhedrons having identical faces.


KNOWN PLATONIC SOLIDS
1. Tetrahedron
2. Hexahedron
3. Octahedron
4. Dodecahedron
5. Icosahedron

FORMULAS:

SIMILAR SOLIDS

Two solids are similar if any two corresponding sides or planes are proportional. (e.g. spheres and cubes)

The areas of similar figures or similar faces have the same ratio as the squares of any two corresponding lines.

𝐴1 𝑥12
=
𝐴2 𝑥22

The volumes of similar solids have the same ratio as the cubes of the corresponding lines.

𝑉1 𝑥13
=
𝑉2 𝑥23

For height proportional (applicable for solids = 1/3 Bh )

𝑉1 ℎ13
=
𝑉2 ℎ23
COMPOSITE SOLIDS – these solids have two or more solids joined together. Volume is the determined by combining the solids.

EXAMPLES:
1. Determine the number of faces, vertices and edges of a triangular prism.
SOLUTION:

2. Two similar cylinders have the ratio of the areas of 16:25. Find the height of the larger cylinder if the smaller cylinder height is 24?
SOLUTION:

3. A vitamin is in the shape of a cylinder with a hemisphere at each end as shown. Find the total volume.

SOLUTION:
We have a sphere and a cylinder with diameter of 4 mm, radius is 2 mm.

4𝜋 3 4𝜋
𝑉𝑠𝑝ℎ = 𝑟 = (2)3 = 33.5103 𝑚𝑚3
3 3
For the cylinder:
𝑉𝑐𝑦𝑙 = 𝜋𝑟 2 ℎ = 𝜋(2)3 (15) = 376.9911 𝑚𝑚3

Lastly combining the volumes:

𝑉 = 𝑉𝑐𝑦𝑙 + 𝑉𝑠𝑝ℎ = 33.5103 + 376.9911 = 410.5014 𝑚𝑚3 → 𝐴𝑛𝑠.

EXERCISES - Answer the following questions.

1. Find the number of vertices of an octagonal prism. Ans. 16


2. A regular dodecahedron has an edge of 12 cm. Compute the volume of the dodecahedron. Ans. 13236.48 cm3
3. The lift power of a weather balloon is the amount of weight the balloon can lift. Find the missing measures in the table below, given that the ratio of
the lift powers is equal to the ratio of the volumes of the balloons. Ans. 1:8
4. A silo with right circular cylinder whose radius of 6 m and the height is twice the radius is surmounted by a hemisphere. Find the volume of the air
can be contained. Ans. 1809.557 m3
5. Tennis balls with 3 in. diameter are sold in cans of three. Find the volume not occupied by tennis balls. Ans. 21.21 in3
6. Two similar solids have the volumes 1500 and 2592. Find the ratio of the sides. Ans. 5:6
7. An oil tank is in the shape of hemisphere in both ends and a cylinder with volume equal to 11,000π cubic inches. Determine the radius of the tank.
Ans. 12.4 inches
8. Find the surface area of an octahedron with an edge of 5 cm. Ans. 86.60 cm2
9. Determine the number of edges in a regular icosahedron. Ans. 30
10. Raven ordered a new wooden nameplate for her office desk. The nameplate is made of a triangular prism attached to a 0.6 inch high base. The
base is in the shape of a rectangular prism. Find the volume of the entire wooden nameplate. Ans. 9.6 inches3
MISCELLANEOUS SOLIDS
Note: For Pappus Theorems, See Integral Calculus.

PARABOLOIDS – solid surface that has exactly one axis of symmetry and no center of symmetry. The term "paraboloid" is derived from parabola, which refers
to a conic section that has a similar property of symmetry.

PRISMATOIDS – solids such that the area of any section parallel to and distant y from a fixed plane can be expressed as a polynomial of y of degree not
higher than the third.

𝐴𝑦 = 𝑎𝑦 3 + 𝑏𝑦 2 + 𝑐𝑦 + 𝑑

Where a,b, and c are constants which may be positive, negative , or zero.
PRISMOIDAL FORMULA – One criteria of knowing if a certain solid is a prismatoid is that all section parallel to a certain base are all similar.

ELLPISOIDS - surface that may be obtained from a sphere by deforming it by means of directional scalings, or more generally, of an affine transformation.

PROLATE SPHEROID – formed by revolving the ellipse about its major axis.
OBLATE SPHEROID - is formed by revolving the ellipse about its minor axis. Earth is an example of oblate spheroid.

SPHERICAL POLYGONS – polygon on the surface of a sphere whose sides are arcs of great circles.

SPHERICAL PYRAMID VOLUME

For the additional volumes of the other figures, see Integral Calculus.

EXAMPLES:

1. The ellipsoid whose radii are given as a = 9 cm, b = 6 cm and c = 3 cm. Find the volume of an ellipsoid.
SOLUTION:

2. The volume of the truncated prism with an equilateral triangle as its horizontal base is equal to 3600 m3 . The vertical edges at each corners are 4,
6 and 8 , respectively. Find the one side of the base.
SOLUTION:
EXERCISES -Answer the following questions.

1. A truncated prism has a square horizontal base. The vertical edges at each corners are 4,5 , 6 and 7, respectively. If the volume of the truncated
prism is 8800 m3 , compute the one side of the base.
2. Find the volume of the ellipsoid if a = 2 in., b = 4 in., and c = 3 in. Ans. 100.48 in3

CONGRATULATIONS! You completed the Bridging Program in Algebra, Trigonometry, Analytic Geometry and Solid Mensuration

For more information about the Files, feel free to like and share our page

Civil Engineering Board Exams Problems Philippines


CE Board Exams PH Secondary

Thank you for trusting the Bridging Program and best of luck to your Engineering Journey!
REFERENCES:

1. Engineering Mathematics by Gillesania


2. https://www.ck12.org/geometry/Area-and-Volume-of-Similar-Solids/lesson/Area-and-Volume-of-Similar-Solids-BSC-GEOM/
3. Intermediate Algebra by Elayn Martin
4. https://www.ck12.org/geometry/solid-figures/lesson/Polyhedrons-BSC-
GEOM/#:~:text=Examples%20of%20polyhedrons%20include%20a,surfaces%20that%20are%20not%20polygons.
5. Engineering Mathematics by Besavilla
6. https://www.onlinemath4all.com/similar-solids.html
7. Algebra and Trigonometry by Cynthia Young
8. https://flexbooks.ck12.org/cbook/ck-12-interactive-middle-school-math-7-for-ccss/section/6.11/related/lesson/composite-solids-geom/
9. https://www.jcschools.us/cms/lib/MO01909951/Centricity/Domain/1927/Similar%20Solids%20Word%20Problems%20Finding%20Parts.pdf
10. Algebra and Trigonometry by Bennett
11. https://www.superprof.co.uk/resources/academic/maths/geometry/solid/regular-octahedron.html
12. https://en.wikipedia.org/wiki/Paraboloid
13. https://byjus.com/volume-of-an-ellipsoid-formula/
14. https://www.basic-mathematics.com/volume-of-an-ellipsoid.html
15.

You might also like